Você está na página 1de 103

UNIVERSIDADE DO ESTADO DO PAR

CENTRO DE CINCIAS SOCIAIS E EDUCAO


DEPARTAMENTO DE MATEMTICA, ESTATSTICA E INFORMTICA.
LICENCIATURA EM MATEMTICA MODALIDADE A DISTNCIA

DISCIPLINA: TEORIA DOS NMEROS

CONTUDO PROGRAMTICO:
INTRODUO AO ESTUDO DA TEORIA DOS NMEROS:
UNIDADE I NMEROS INTEIROS:
1.1 - Nmeros inteiros.
1.2 - Propiedades dos inteiros;
1.3 - Valor absoluto de um inteiro;
1.4 - Questes Resolvidas e Propostas.
UNIDADE II INDUO MATAMTICA:
2.1 - Elemento mnimo de um conjunto de inteiros.
2.2 - Princpio da boa ordenao;
2.3 - Princpio de induo finita;
2.4 - Induo matemtica;
2.5 - Exemplos de demonstrao por induo matemtica e outras formas de induo
matemtica;
2.6 - Questes Resolvidas e Propostas.
UNIDADE III DIVISIBILIDADE:
3.1 - Relao de divisibilidade em Z.
3.2 - Conjunto dos divisores de um inteiro;
3.3 - Divisores comuns de dois inteiros;
3.4 - Algoritmo da diviso;
3.5 - Paridade de um inteiro;
3.6 - Questes Resolvidas e Propostas.
UNIDADE IV MXIMO DIVISOR COMUM:
4.1 - Mximo divisor comum de dois inteiros.
4.2 - Existncia e unicidade do mdc;
4.3 - Inteiros primo entre si;
4.4 - Caracterizao do mdc de dois inteiros;
4.5 - Mdc de vrios inteiros;
4.6 - Questes Resolvidas e Propostas.
UNIDADE V ALGORITMO DE EUCLIDES MNIMO MLTIPLO COMUM:
5.1 - Algoritmo de EUCLIDES.
5.2 - Mltiplos comuns de dois inteiros;
5.3 - Mnimo mltiplo comum de dois inteiros;
5.4 - Relao entre o mdc e o mmc;
5.5 - Mmc de vrios inteiros;
5.6 - Questes Resolvidas e Propostas.
UNIDADE VI NMEROS PRIMOS:
6.1 - Nmeros primos e compostos.
6.2 - Teorema fundamental da Aritmtica;
6.3 - Formula que do primos;
6.4 - Crivo de ERATSTENES;
6.5 - Primos gmeos;
6.6 - Seqncias de inteiros consecutivos compostos;
6.7 - Conjectura de GOLDBACH;
6.8 - Mtodo de fatorao de FERMAT;
6.9 - Questes Resolvidas e Propostas.

UNIDADE VII EQUAES DIOFANTINAS LINEARES:


7.1 - Generalidades.
7.2 - Condio de existncia de soluo;
7.3 - Soluo da equao ax + by = c;
7.4 - Questes Resolvidas e Propostas.
UNIDADE VIII CONGRUNCIAS:
8.1 - Inteiros congruentes.
8.2 - Caracterizao de Inteiros congruentes;
8.3 - Propriedades das congruncias;
8.4 - Sistemas completos de restos;
8.5 - Questes Resolvidas e Propostas.
UNIDADE IX CONGRUNCIAS LINEARES:
9.1 - Generalidades.
9.2 - Condio de existncia de soluo;
9.3 - Soluo da congruncia a x b(mod. m);
9.4 - Resoluo de equaes diofantinas lineares por congruncias;
9.5 - Inverso de um inteiro;
9.6 - Questes Resolvidas e Propostas.
UNIDADE X SISTEMAS DE CONGRUNCIAS LINEARES:
10.1 - Generalidades.
10.2 - Teorema do resto chinez;
10.3 - Questes Resolvidas e Propostas.
UNIDADE XI TEOREMA DE FERMAT E WILSON:
11.1 - Teorema de Fermat.
11.2 - Teorema de Wilson;
11.3 - Questes Resolvidas e Propostas.
BIBLIOGRAFIA:

INTRODUO AO ESTUDO DA TEORIA DOS NMEROS


Embora existam diversos tipos de nmeros em Matemtica (reais, complexos, etc.), o nome
"Teoria dos Nmeros" tradicionalmente reservado para o estudo dos Nmeros Inteiros, isto , -3, 2, -1, 0, 1, 2, 3, ... Tambm usado o nome Aritmtico, proveniente de arithms, que em grego
significa nmero".
A Teoria dos Nmeros, a mais pura disciplina dentro da mais pura das Cincias a
Matemtica e tem uma longa histria, originando-se nas antigas civilizaes da humanidade.
Listamos primeiro alguns nomes famosos de matemticos que contribuiro para o estudo da teoria
dos nmeros:
Pitgoras (569-500 a. C.)
Euclides (_ 350 a. C.)
Eratstenes (276-196 a. C.)
Diofante (_ 250 d. C.)
Plutarco (_ 100 d. C.)
Marin Mersenne (1588-1648)
Pierre de Fermat (1601-1665)
Blaise Pascal (1623-1662)
Christian Goldbach (1690-1764)
Leonhard Euler (1707-1783)
Joseph Louis Lagrange (1736-1813)
John Wilson (1741-1793)
Adrien Marie Legendre (1752-1833)
Carl Friedrich Gauss (1777-1855)
Augustin Louis Cauchy (1789-1857)
Peter Gustav Dirichlet (1805-1859)
P. L. Tchebychef (1821-1894)
Frederick Nelson Cole (1861-1927)
Axel Thue (1863-1922)
Jacques Salomon Hadamard (1865-1963)
Charles de la Vall e Poussin (1866-1962)
Dentre outros....
A teoria dos nmeros veio a ocupar-se com uma classe mais vasta de problemas que
surgiram naturalmente do estudo dos nmeros inteiros. A teoria dos nmeros pode ser subdividida

em vrios campos, de acordo com os mtodos que so usados e das questes que so investigadas, a
saber:
1) Teoria elementar dos nmeros: utiliza somente os mtodos elementares da aritmtica para a
verificao e comprovao das propriedades essenciais do conjunto dos nmeros inteiros e em
particular as propriedades dos nmeros primos.
2) Teoria analtica dos nmeros: utiliza a anlise real e anlise complexa, especialmente para
estudar as propriedades dos nmeros primos.
3) Teoria algbrica dos nmeros: utiliza lgebra abstrata e estuda os nmeros algbricos.
4) Teoria geomtrica dos nmeros: utiliza mtodos geomtricos, algbricos e analticos.
Nesta notas faremos o estudo da primeira Teoria, um conceito chave em Teoria elementar
dos Nmeros o conceito de divisibilidade. Enquanto nos nmeros reais, por exemplo, pode-se
dividir qualquer nmero por outro (no nulo), obtendo como resultado um nmero real, nos inteiros
diferente. Um inteiro a s divisvel pelo inteiro b quando existir um inteiro c tal que a = bc.
Nesse caso, diz-se tambm que b um divisor de a, ou que b divide a, ou ainda que a mltiplo de
b. Por exemplo, 8 divisvel por 2, mas no por 3. Mesmo que a no seja divisvel por b, pode-se
sempre encontrar, de modo nico, inteiros c (quociente) e r (resto) tais que a = bc + r.
Todo inteiro a divisvel por 1, -1, a, -a. Estes so os divisores triviais de a. Um inteiro
dito primo quando s possui os divisores triviais. Um inteiro de valor absoluto maior que 1 e que
no seja primo (isto , possua divisores no triviais) dito composto. Por exemplo: So primos: 2, 2, 3, -3, 17, .... So compostos 6 = 2x3, -8 = (-2) x 4, ... Os nmeros 0, 1 e 1 no so primos nem
compostos. Euclides foi o primeiro a demonstrar que existe uma infinidade de nmeros primos.
O mximo divisor comum dos inteiros no nulos a e b tem a propriedade de ser mltiplo de
qualquer divisor comum de a e b e pode ser encontrado pelo algoritmo de Euclides. Quando o
mximo divisor comum de a e b for 1, ento seus nicos divisores comuns so 1 e 1. Nesse caso, a
e b so ditos primos entre si, ou relativamente primos. Por exemplo, 9 e 14 so primos entre si.
As propriedades mais cruciais dos nmeros inteiros, e que no tm similares nos reais ou nos
complexos, so o Princpio da Boa Ordenao, segundo o qual qualquer conjunto no vazio de
inteiros limitado inferiormente possui um elemento mnimo, e o Princpio de Induo, segundo o
qual se uma propriedade P(n), referente ao inteiro n, for verdadeira para n = a, e a veracidade de
P(n) acarretar a veracidade de P(n + 1), ento P(n) verdadeira para todo inteiro maior que ou igual
a a.
A partir das propriedades usuais da adio e da multiplicao de inteiros, da relao <, e do
Princpio da Boa Ordenao (ou do de Induo, que lhe equivalente), possvel construir toda a
Teoria dos Nmeros. Um de seus resultados mais importantes o Teorema Fundamental da

Aritmtica, segundo o qual todo inteiro (diferente de 0, 1 e 1) pode ser escrito de modo nico
como um produto de fatores primos.
Uma das caractersticas da Teoria dos Nmeros que ela inclui problemas extremamente
simples de enunciar e ao mesmo tempo incrivelmente difceis de resolver. Um exemplo a
conjectura de Feuerbach: "todo nmero par a soma de dois nmeros primos"; ningum at hoje
conseguiu decidir se isto verdadeiro ou falso. Outro exemplo o famoso ltimo Teorema de
Fermat: "Dado um inteiro n maior que 2, impossvel encontrar inteiros no nulos x, y, z tais que x n
+ yn = zn". Este teorema, enunciado no sculo XVII por Fermat, que s foi demonstrado em 1995,
por Wiles.
Gauss, o "prncipe dos matemticos", dizia que a Matemtica era a rainha das cincias, e a
Aritmtica, a rainha das Matemticas. Gauss desenvolveu muita a Teoria dos Nmeros. Aos 22
anos, em 1799, publicou em latim suas "Investigaes Aritmticas", onde introduziu o importante
conceito de congruncia para nmeros inteiros.
O matemtico ingls Hardy, grande especialista em Teoria dos Nmeros, orgulhava-se, em
1940, de que "nenhuma descoberta sua havia feito, nem provavelmente viria a fazer, direta ou
indiretamente, alguma diferena para o conforto da humanidade". No entanto, 50 anos depois, um
obscuro matemtico americano descobriria uma falha no recm lanado processador Pentium, ao
realizar clculos "inteis" sobre primos gmeos (nmeros primos que diferem de 2).
Mas j no prprio momento em que Hardy escrevia aquela frase, durante a segunda guerra
mundial, trs americanos desenvolviam um sistema de cdigo secreto, chamado SRA, baseado nas
dificuldades insuperveis para descobrir os fatores primos de um nmero muito grande. Criava-se
um novo ramo a Criptografia, a cincia dos cdigos, fortemente baseado em Teoria dos Nmeros.
Com o advento dos computadores e da computao algbrica, a Criptografia ganhou um novo
impulso. Neste momento, a proliferao de senhas bancrias e de cartes de crdito, bem como a
crescente necessidade de criptografar dados confidenciais que inundam a Internet, faz da
Criptografia um dos ramos mais em moda da Matemtica aplicada. E um dos mais teis, para
desespero pstumo de Hardy.

UNIDADE I NMEROS INTEIROS


1 - Introduo:
A noo de nmero est, atravs dos tempos, associada a todos os tipos de atividades
humanas. A primeira concepo de nmero data do perodo paleoltico. Poucos progressos se
fizeram neste campo at se dar a transio para o perodo neoltico, durante o qual j existia uma
atividade comercial importante entre diversas povoaes. Esta atividade promoveu a formao de
linguagens, cujas palavras exprimiam coisas muito concretas e poucas abstraes, mas onde j havia
lugar para alguns termos numricos simples. Estes termos numricos destinavam-se apenas a
estabelecer a distino entre um, dois e muitos. Depois de durante milnios ter utilizado os nmeros
para contar, medir, calcular, o homem comeou a especular sobre a natureza e propriedades dos
prprios nmeros. Desta curiosidade nasceu a Teoria dos Nmeros, um dos ramos mais profundos
da matemtica.
A Teoria dos Nmeros nasceu cerca de 600 anos antes de Cristo quando Pitgoras e os seus
discpulos comearam a estudar as propriedades dos nmeros inteiros. Os pitagricos rendiam
verdadeiro culto mstico ao conceito de nmero, considerando-o como essncia das coisas.
Acreditavam que tudo no universo estava relacionado com nmeros inteiros ou razes de nmeros
inteiros (em linguagem atual, nmeros racionais). Alis, na antiguidade a designao nmero
aplicava-se s aos inteiros maiores do que um. Esta crena foi profundamente abalada quando
usaram o Teorema de Pitgoras para calcular a medida da diagonal de um quadrado unitrio. Com
efeito, a diagonal divide o quadrado em dois tringulos retngulos issceles cujos catetos tm
comprimento um e assim, pelo teorema de Pitgoras, a medida da hipotenusa igual raiz quadrada
de dois, que no pode ser expresso como quociente de inteiros.
Ao descobrirem que a diagonal de um quadrado de lado 1 no era uma razo entre dois
inteiros (em linguagem atual, que a raiz quadrada de 2 um nmero irracional) os Pitagricos
consideraram quebrada a harmonia do universo, j que no podiam aceitar a raiz quadrada de dois
como um nmero, mas no podiam negar que esta raiz era a medida da diagonal de um quadrado
unitrio. Convencidos de que os deuses os castigariam caso divulgassem aquilo que lhes parecia
uma imperfeio divina, tentaram ocultar a sua descoberta. Segundo reza a lenda, o primeiro
membro da seita pitagrica que divulgou esta descoberta morreu afogado.
Este fato teve grandes repercusses na histria da cincia que se fizeram sentir at finais do
sculo XIX. De cada vez que as necessidades do clculo levavam a introduzir novos entes
numricos gerava-se uma enorme desconfiana sua volta, o que levava a atribuir-lhes designaes
curiosas. Assim, os nmeros irracionais eram designados por nmeros inexprimveis e por nmeros
incalculveis. Durante muitos sculos os nmeros reais (fracionarias ou racionais e irracionais)

foram apenas concebidos como medidas de grandezas e s nos finais do sculo XIX, principalmente
por obra dos matemticos alemes Dedekind e Cantor, se construiu uma teoria dos nmeros reais
independente da geometria.
1.1 - Nmeros Inteiros Noes Fundamentais:
Os nmeros inteiros ou apenas os inteiros so: , 3, 2, 2, 0,1, 2,3, .
Cujo conjunto representa-se pela letra , isto :

, 3, 2, 2, 0,1, 2,3, , deste conjunto

destacam-se os seguintes subconjuntos:


1) Conjunto dos inteiro no nulos ( 0) . = x

z/x

1, 2, 3, 4, .

2) Conjunto dos inteiro no negativos ( 0) . = x

z/x

0,1, 2,3, .

3) Conjunto dos inteiro no positivos ( 0) . = x

z/x

0, 1, 2, 3, .

4) Conjunto dos inteiro positivos ( 0) . = x

z/x

4) Conjunto dos inteiro positivos ( 0) . = x

z/x

1, 2,3, 4, .
1, 2, 3, 4, .

Os inteiros positivos so tambm denominados inteiros naturais e por isso o conjunto dos inteiros
positivos habitualmente designados pela letra = *+ .
1.2 - Propriedades dos Inteiros:
O conjunto Z dos inteiros munidos das operaes de adio (+) e multiplicao ( ) possui as
propriedades fundamentais que a seguir enumeramos, onde a, b e c so inteiros quaisquer, isto ,
elementos de z:
1) Lei comutativa para multiplicao e adio: a + b = b + a e ab = ba .
2) Lei associativa para multiplicao e adio: a + b + c = a + b + c

(ab)c = a(bc) .

3) Existncia da identidade para adio e multiplicao: 0 + a = a e a 1 = 1 a = a .


4) Existncia do inverso em relao adio, -a, para todo inteiro a: a + (- a) = (-a) + a = 0 .
5) Lei distributiva: a b + c = ab + ac .
6) Lei do cancelamento da multiplicao 0 a = 0 e se ab = 0, ento a = 0 ou b = 0 .
Tambm existe uma relao de ordem entre os inteiros, representada pelo sinal < (menor que)
que possui as seguintes propriedades:
7) Se a

0, ento a < 0 ou 0 < a .

8) Se a

b, e b < c ento, a < c .

9) Se a

b, ento a + c < b + c .

10) Se a

b, ento 0 < c, ento ac < bc .

11) Se a

b, ento c < 0, ento bc < ac .

12) (Lei da Tricotomia) Para quaisquer inteiros a e b, vale exatamente uma das seguintes
afirmaes: a < b, a = b ou a > b .
13) Suponha que a

b , e seja c um inteiro qualquer. Ento

a + c b+c
ac bc se c > 0, mas ac bc, se c < 0.

Destas propriedades podem ser deduzidas muitas outras propriedades dos inteiros.
1.3 - Valor absoluto de um Inteiro:
Definio: Chama-se valor absoluto de um inteiro a, o inteiro que se indica por a , tal que:

a se a
-a se a

o
o
a
a

A partir da definio de a , para todo inteiro a, temos:

1) a
2)

0e a
a

3) ab
4) a

0 se a = 0 .
a .

a b.

5) a + b

b.

ab .

a2

-a
a

Teoremas: Se a e b so dois inteiros, quaisquer ento:

a .
a
a2

Questes Resolvidas
01) Calcular a soma dos n primeiros inteiros positivos.
Soluo: Vamos escrever a soma dos n primeiros nmeros inteiros positivos em ordem crescente e
a mesma soma em ordem decrescente, temos:
S=1
S=n

+
2
+ n1

+ 3
+ n2

+ 4 + ........
+ n 3 + ........

Somando as duas igualdades:


2S = (n + 1) + (n + 1) + (n + 1) + (n + 1) + ..............

+ n3
+ 4

+n2 + n1
+ 3 + 2

+ n
+ 1

+ (n + 1) + (n + 1) + (n + 1) + (n + 1)

Observe que sero n parcelas iguais a (n + 1). Portanto, 2S = n(n + 1) S = n(n + 1)/2.
02) Calcular o inteiro positivo n, sabendo que 3n+2 2n+3 = 2592.
Soluo: Decompondo 2592, obtm-se 34.25. Portanto, n + 2 = 4 n = 2, ou 5 = n + 3 n = 2.
Pois a forma de decomposio em fatores primos nica.
03) Achar um inteiro positivo de dois algarismos que seja igual ao qudruplo da soma dos seus
algarismos.
Soluo: Um nmero de algarismos a e b, na base 10 expresso por 10a + b.
Portanto, 10a + b = 4(a + b) 6a = 3b b = 2a. Ou seja, qualquer nmero de dois algarismos,
onde o algarismo das unidades o dobro do algarismo das unidades. Assim, temos: 12, 24, 36, etc.
04) Achar o menor e o maior inteiro positivo de n algarismos.
Soluo: Menor: 1 algarismo igual a 1 e os demais (n 1) iguais a zero. Portanto, 1 x 10 n 1.
Maior: todos os n algarismos iguais a 9, ou 1 seguido de n zeros menos 1 1.10n 1
Observao: Considerando, n = 5. Menor 10000 = 1.105 1 = 1.104
Maior 99999 = 100000 1 = 1.105 1.
05) Achar todas as solues inteiras e positivas da equao: x2 y2 = 88.
Soluo: x2 y2 = 88 (x + y) (x y) = 88. Como x e y so inteiros positivos, (x + y) e (x y)
so dois nmeros inteiros cujo produto 88. Assim, (1) x + y = 88 e x y = 1; (2) x + y = 44 e x y
= 2; (3) x + y = 22 e x y = 4; (4) x + y = 11 e x y = 8. Cada par de duas equaes formam um
sistema. Para resolver o sistema basta somar as duas equaes, o que resultaria em 2x = soma dos
nmeros. Como essa soma tem que ser par (x inteiro), resulta apenas as possibilidades 2 e 3.
Portanto, 2x = 46 x = 23 e y = 44 23 = 21 ou 2x = 26 x = 13 e y = 22 13 = 9.
Resposta: (x = 23, y = 21) e (x = 13, y = 9).
06) O produto de um inteiro positivo de trs algarismos por 7 termina direita por 638. Achar esse
inteiro.

Soluo: Para facilitar o raciocnio, construamos a tabela de multiplicao por 7.


7 x 1 = 7, 7 x 2 = 14, 7 x 3 = 21, 7 x 4 = 28, 7 x 5 = 35, 7 x 6 = 42, 7 x 7 = 49, 7 x 8 = 56, 7 x 9 =
63. Como o algarismo das unidades 8, o nico valor possvel para o algarismos das unidades do
nmero 4. Ao efetuar a multiplicao do algarismo das unidades, que 4 por 8, vo duas unidades
para a casa das dezenas. Assim, o algarismo das dezenas deve ser tal que ao multiplicar por 7 e
somar 2, resulte num final igual a 3. Portanto, o algarismo das dezenas 3, pois 7 x 3 + 2 = 23. Da
mesma forma vo duas unidades para a casa das centenas. O algarismo a deve ser de forma que, ao
somar 2 (vindo das dezenas) resulte em 6. Portanto, deve ser um mltiplo de 7 terminado em 4. Isto
permite concluir que o algarismo das centenas 2 , pois 2 x 7 + 2 = 16.
Portanto, o nmero 234.
07) Um livro tem 1235 pginas. Determinar o nmero de vezes que o algarismo 1 aparece na
numerao da pginas deste livro.
Soluo: De 1 a 100, o algarismo 1 aparece 10 vezes nas unidades (1, 11, 21,... 91) e 10 vezes nas
dezenas (10, 11, 12, ...19). Portanto a cada centena o algarismo 1 aparece 20 vezes. Em 1235 temos
12 centenas. Portanto o algarismo 1 aparecer 20 x 12 = 240 vezes na posio das unidades e
dezenas. De 100 a 200, o algarismo 1 aparece 100 vezes na posio das centenas. Isto se repete de
1100 a 1200. Portanto, 200 vezes na posio das centenas.
De 1200 a 1236, o algarismo 1 aparece 4 vezes nas unidades e 10 vezes nas dezenas. Totalizando 14
vezes. De 1000 a 1235, o algarismo 1 aparece 236 vezes na posio dos milhares.
Portanto: 240 + 200 + 14 + 236 = 690 vezes.
08) Achar o inteiro que deve ser somado a cada um dos inteiros 2, 6 e 14 para que, nesta ordem,
formem uma proporo contnua.
Soluo: Uma proporo contnua aquela que tem os meios ou os extremos iguais. Pela definio
podemos ter (a) (2 + x)/(6 + x) = (6 + x)/(14 + x) ou (2 + x)/(6 + x) = (14 + x)/(2 + x). Na situao
(a), (6 + x) 6 + x) = (2 + x) (14 + x) => 36 + 12x + 2x = 28 + 16x + 2x 4x = 8 x = 2. Na
situao (b) (2 + x) (2 + x) = (6 + x) (14 + x) 4 + 4x + 2x = 84 + 20x + 2x 16x = - 80
x = - 5.
R: 2 ou 5.
09) Achar o menor inteiro cujo produto por 21 um inteiro formado apenas por 4 algarismo.
Soluo: O nmero o menor mltiplo de 21 maior que 1000.
Portanto: 1000 = 47 x 21 + 13. Portanto, o nmero 48 x 21 = 1008.
10) Escreve-se a seqncia natural dos inteiros positivos, sem separar os algarismos:

123456789101112131415...
Determinar:
(a) O 435 algarismo.
Soluo: De 1 a 9 so escritos 9 algarismos. De 10 a 99, so dois algarismos em cada nmero 2
x 90 = 180 algarismos. Portanto, at 100 so escritos: 9 + 180 + 3 = 192.
Para chegar ao algarismo que ocupa o 435 lugar sero necessrios mais 435 192 = 243
algarismos. Como a partir de 100 so usados 3 algarismos teramos 243 3 = 81 nmeros aps o
100. Portanto, o nmero 181 e o algarismo que ocupa a posio o 1.
(b) O 1756 algarismo.
Soluo: Da mesma forma 1756 192 = 1564 1564 3 = 521 e sobra 1 algarismo. Portanto
teramos at a 100 + 521 = 621. Como sobra 1 algarismo, o prximo o 6 do nmero 622.
(c) O 12387 algarismo.
Soluo: At 1000 seriam 9 + 90 x 2 + 900 x 3 + 4 = 2889.
12387 2889 = 9498 9498 4 = 2374 e sobram dois algarismo. Portanto, o ltimo nmero
inteiro 1000 + 2374 = 3374. A sobra de dois algarismos implica que o ltimo algarismo ser 3, o
segundo algarismo de 3375.
11) Mostrar que o produto de dois fatores entre 10 e 20 o dcuplo da soma do primeiro com as
unidades do segundo mais o produto das unidades dos dois.
Soluo: Sejam os nmeros 10 + b e 10 + c, com 0 < b < 10 e 0 < c < 10. Nestas condies 10 + b e
10 + c estaro compreendidos entre 10 e 20 e b e c sero os algarismos das unidades.
Efetuando o produto temos: (10 + b) (10 + b) = 100 + 10b + 10c + bc = 10[(10 + b) + c] + bc.
(10 + b) + c a soma do primeiro com as unidades do segundo, bc o produto dos dois e 10[(10 +
b) + c] o dcuplo da soma do primeiro com as unidades do segundo.

Questes Propostas
01) Calcule o inteiro positivo n, sabendo-se que: 3n + 3n+1 + 3n+2 + 3n+3 = 1080.
R: n = 3.
02) Decompor o inteiro 575 numa soma de cinco inteiros mpares consecutivos.
R: 109, 113, 115, 117, 119.
03) Achar todas as solues inteiras e positivas da equao (x + 1) (y + 2) = 2xy.
R: os valores so (x = 2, y = 6), (x = 3, y =4) e (x = 5, y = 3).
04) Verificar se o quadrado de um inteiro pode terminar em 2, 3, 7 ou 8.
R: Portanto, no pode terminar em 2, 3, 7 ou 8.
05) A soma dos quadrados de dois inteiros 3332 e um deles o qudruplo do outro. Achar os dois
inteiros.
R: 14 e 56.
06) A mdia aritmtica de dois inteiros positivos 5 e a mdia geomtrica 4. Achar os dois
inteiros.
R: 8 e 2.
07) Calcular a soma dos trs maiores nmeros inteiros de, respectivamente, trs, quatro e cinco
algarismos.
R: 110997.
08) Determinar a diferena entre o maior nmero inteiro com seis algarismos diferentes e o menor
inteiro com cinco algarismos tambm diferentes.
R: 977420.
09) Mostrar que o produto de quatro algarismos consecutivos, aumentado de 1, um quadrado
perfeito.
10) Sejam a e b dois inteiros. Demonstrar:
(a) Max (a, b) = (a + b + |a b|)/2.
(b) Min (a, b) = (a + b |a b|)/2.
11) Achar cinco inteiros positivos consecutivos cuja soma dos quadrados igual a 2010.
R: 18, 19, 20, 21 e 22.
12) Escreve-se a seqncia natural dos inteiros positivos pares, sem separar os algarismos:
24681012141618...

Determinar o 2574 algarismo que se escreve.


R: algarismo o 6.
13) Os inteiros a e b so tais que 1 < a < 3 e 2 < b < 0. Mostrar que 1 < a b < 5.
R: 1 < a b < 5.
14) Os inteiros a e b so tais que -2 < a < 2 e - 2 < b < 2. Mostrar que 4 < a b < 4.
R: 4 < a b < 4.
15) Achar o menor inteiro positivo que multiplicado por 33 d um produto cujos algarismos so
todos 7.
R: 777777: 33 = 23569.
16) No planeta Mong o nmero de horas por dia igual a nmero de dias por semana, que igual ao
nmero de semanas por ms, que igual ao nmero de meses por ano. Sabendo que em Mong h
4096 horas por ano, quantas semanas h por ms?
R:
17) A soma de trs nmeros naturais consecutivos igual ao produto desses trs nmeros. A soma
dos quadrados desses nmeros :
R:
18) Sejam a, b e c inteiros e positivos. Entre as opes abaixo, a expresso que no pode representar
o nmero 24 :
a) ab3

b) a2b3

c) acbc

d) ab2c3

e) acbccc

19) Efetuando as operaes indicadas na expresso abaixo obtemos um nmero de quatro


22007
algarismos. Qual a soma dos algarismos desse nmero: 2006
2

22005
22004

2006 .

R:
20) Qual a soma dos algarismo do nmero:

22
2

23
22

24
22005

23
22004

22006
?
22005

R:

21) Quantos so os possveis valores inteiros de x para que

x 99
seja um nmero inteiro?
x 19

R: 20.
22) Um pai tem 33 anos e seu filho, 7 anos. Depois de quantos anos a idade do pai ser o triplo da
idade do filho?
R: 6.

UNIDADE II INDUO MATEMTICA


2.1 - Introduo:
O Princpio da Induo um eficiente instrumento para a demonstrao de fatos referentes
aos nmeros naturais. Por isso deve-se adquirir prtica em sua utilizao. Por outro lado,
importante tambm conhecer seu significado e sua posio dentro do arcabouo da Matemtica.
Entender o Princpio da Induo praticamente o mesmo que entender os nmeros naturais.
Apresentamos abaixo uma breve exposio sobre os nmeros naturais, onde o Princpio da
Induo se insere adequadamente e mostra sua fora terica antes de ser utilizado na lista de
exerccios propostos ao final.
2.2 - A Seqncia dos Nmeros Naturais:
Os nmeros naturais constituem um modelo matemtico, uma escala padro, que nos
permite a operao de contagem. A seqncia desses nmeros uma livre e antiga criao do
esprito humano. Comparar conjuntos de objetos com essa escala abstrata ideal o processo que
torna mais precisa a noo de quantidade; esse processo (a contagem) pressupe, portanto o
conhecimento da seqncia numrica. Sabemos que os nmeros naturais so 1, 2, 3, 4, 5, A
totalidade desses nmeros constitui um conjunto, que indicaremos com o smbolo N e que
chamaremos de conjunto dos naturais. Portanto N = {1, 2, 3, 4, 5,}.
Evidentemente, o que acabamos de dizer s faz sentido quando j se sabe o que um
nmero natural. Faamos de conta que esse conceito nos desconhecido e procuremos investigar o
que h de essencial na seqncia 1, 2, 3, 4, 5 .
Deve-se a Giussepe Peano (1858 - 1932) a constatao de que se pode elaborar toda a teoria
dos nmeros naturais a partir de quatro fatos bsicos, conhecidos atualmente como os axiomas de
Peano. Noutras palavras, o conjunto N dos nmeros naturais possui quatro propriedades
fundamentais, das quais resultam, como conseqncias lgicas, todas as afirmaes verdadeiras que
se podem fazer sobre esses nmeros.
2.3 - Elemento Mnimo:
Definio 1.1 - Seja A um conjunto de inteiros. Chama-se elemento mnimo de A um elemento a
A tal que a

x para todo x

A. Notao: a = min A.

min A = a se, e somente se, (a A e ( x A) a

x).

Teorema: Se a elemento mnimo de A, ento este elemento nico.


2.4 - Princpio da Boa Ordenao (PBO).
Todo conjunto no vazio A, de inteiros no negativos, possui elemento mnimo.
A

Z+, A

, existe min A.

2.5 - Induo Matemtica:


Teorema: Seja P(n) uma proposio associada a cada inteiro positivo n e que satisfaz s duas
seguintes condies:
1) P(1) verdadeira.
2) Para todo inteiro k, se P(k) verdadeira, ento P(k + 1) tambm verdadeira. Nestas condies,
a proposio P(n) verdadeira para todo inteiro positivo n.
2.6 - Princpio da Induo Finita (PIF).
Teorema: Seja S um subconjunto do conjunto N dos inteiros positivos (S

N) que satisfaz as duas

seguintes propriedades:
1) 1 pertence a S (1 S).
2) Para todo inteiro positivo k, se k

S, ento (k + 1)

S;

3) Nestas condies, S o conjunto N dos inteiros positivos: S = N.


2.7 - Outra Forma da Induo Matemtica:
Teorema: Seja r um nmero inteiro positivo fixo e seja P(n) uma proposio associada a cada
inteiro n

r e que satisfaa s duas seguintes condies:

1) P(r) verdadeira.
2) Para todo inteiro k

r, se P(k) verdadeiro, ento P(k + 1) verdadeiro;

3) Nestas condies, P(n) verdadeira para todo inteiro n

r.

Questes Resolvidas
01) Demonstrar por "induo matemtica", as questes abaixo:
n(n 1)(2n 1)
1) 12 + 22 + 32 + ... + n2 =
n N.
6
1.2.3
Soluo: P(1) verdadeira, pois 12 =
. Suponhamos que para P(k) verdadeira:
6
k(k 1)(2k 1)
12 + 22 + 32 + ... + k2 =
. Somando (k + 1)2 a ambos os membros:
6
k(k 1)(2k 1)
12 + 22 + 32 + ... + k2 + (1 + k)2 =
+ (k + 1)2
6
k (k 1)(2k 1) 6(k 1) 2
=
6
(k 1) k(2k 1) 6(k 1)
=
6
2
(k 1)(2k
k 6k 6)
=
6
(k 1)(k 2)(2k 3)
=
Logo P(k + 1) verdadeira.
6

2) 13 + 23 + 33 + ... + n3 =

n 2 (n 1) 2
4

Soluo: P(1) verdadeira, pois 13 =

N.

12 (1 1) 2
. Suponhamos que para P(k) verdadeira:
4

k 2 (k 1) 2
Somando (k + 1)3 a ambos os membros
4
k 2 (k 1) 2
3
3
3
3
3
1 + 2 + 3 + ... + k + (k + 1) =
+ (k + 1)3
4
2
k (k 1) 2 4(k 1) 3
=
4
2
2
(k 1) k
4(k 1)
=
4
2
2
(k 1) (k
4k 4)
=
4
2
(k 1) (k 2) 2
=
Logo P(k + 1) verdadeira.
4

13 + 23 + 33 + ... + k3 =

n (4n 2 1)
3) 1 + 3 + 5 + ... + (2n 1) =
n N.
3
1( 4.12 1)
Soluo: P(1) verdadeira, pois 12 =
Suponhamos que para P(k) verdadeira:
3
k (4k 2 1)
12 + 32 + 52 + ... + (2k 1)2 =
Somando (2k + 1)2 a ambos os membros
3
k (4k 2 1)
12 + 32 + 52 + ... + (2k 1)2 + (2k + 1)2 =
+ (2k + 1)2
3
k (2k 1)(2k 1) 3(2k 1) 2
=
3
(2k 1) k (2k 1) 3(2k 1)
=
3
2
(2k 1)(2k 5k 3)
=
3
(2k 1)(k 1)(2k 3)
=
3
(k 1)(2k 1)(2k 3)
=
Logo P(k + 1) verdadeira.
3
2

4) 13 + 33 + 53 + ... + (2n 1)3 = n2(2n2 1)


Soluo: P(1) verdadeira, pois 13 = 12(2.12 1). Suponhamos que para P(k) verdadeira:
13 + 33 + 53 + ... + (2k 1)3 = k2(2k2 1). Somando (2k + 1)3 a ambos os membros
13 + 33 + 53 + ... + (2k 1)3 + (2k + 1)3 = k2(2k2 1) + (2k + 1)3
= 2k4 k2 + 8k3 + 12k2 + 6k + 1
= 2k4 + 8k3 + 11k2 + 6k + 1
= (k + 1)(2k3 + 6k2 + 5k + 1)
= (k + 1)(k + 1)(2k2 + 4k + 1)
= (k + 1)2(2(k + 1)2 1) Logo P(k + 1) verdadeira.

n(n 1)(n 2)
.
3
Soluo: P(1) verdadeira, pois 1(1 + 1)= 1(1 + 1)(1 + 2)/3
5) 1.2 + 2.3 + 3.4 + ... + n(n + 1) =

Suponhamos que para P(k) verdadeira: 1.2 + 2.3 + 3.4 + ... + k(k + 1) =

k(k 1)(k 2)
3

Somando (k + 1)(k + 2) a ambos os membros:

k(k 1)(k 2)
+ (k + 1)(k + 2)
3
k (k 1)(k 2) 3(k 1)(k 2)
=
3
(k 1)(k 2)(k 3)
=
Logo P(k + 1) verdadeira.
3

1.2 + 2.3 + 3.4 + ... + k(k + 1) + (k + 1)(k + 2) =

a (q n 1 1)
6) a + aq + aq + ... + aq =
,q
q 1
2

Soluo: P(1) verdadeira, pois a + aq =

1.

a (q 1 1 1)
Suponhamos que para P(k) verdadeira.
q 1

a (q k 1 1)
Somando aqk+1 a ambos os membros
q 1
a (q k 1 1)
a + aq + aq2 + ... + aqk + aqk+1 =
+ aqk+1
q 1
a (q k 1 1) aq k 1 (q 1)
=
q 1
k 1
aq
1 q k 1 (q 1)
=
q 1
k 1
a (q
1 qk 2 qk 1)
=
q 1
k 2
a (q
1)
=
Logo P(k + 1) verdadeira.
q 1
a + aq + aq2 + ... + aqk =

02) Demonstrar por induo matemtica:


1) 2n < 2n+1 n N.
Soluo: P(1) verdadeira, pois 21 < 21+1. Suponhamos que para P(k) verdadeira:
2k < 2k+1 Multiplicando por 2 ambos os membros
2.2k < 2.2k+1 ou 2k+1 < 2k+2 logo, P(k + 1) verdadeira.
2) n ! > n2 n 4.
Soluo: P(4) verdadeira, pois 4! > 42 ou 24 > 16. Suponhamos que para P(k) verdadeira:
k! > k2 Multiplicando por k + 1 ambos os membros
k!(k + 1) > k2(k + 1) ou (k + 1)! > k3 + k2 como k3 > k2 e k2 > 2k + 1 temos
(k + 1)! > k2 + 2k + 1 ou (k + 1)! > (k + 1)2 Logo P(k + 1) verdadeira.
3) 2n > n2 n 5.
Soluo: P(5) verdadeira, pois 25 > 52. Suponhamos que para P(k) verdadeira:
2k > k2 Multiplicando por 2 ambos os membros
2.2k > 2.k2 ou 2k+1 > k2 + k2 > k2 + 2k + 1 (pois k2 > 2k + 1) ou 2k+1 > (k + 1)2.
Logo P(k + 1) verdadeira.

4) 24 | (52n 1) n N.
Soluo: P(1) verdadeira, pois 24 | (52.1 1) Suponhamos que para P(k) verdadeira,
24 | (52k 1). Logo 52k 1 = 24t ou 52k = 24t + 1
Multiplicando ambos os membros por 52.52.52k = 52 (24t + 1) = 52.24t + 25 = 52.24t + 24 + 1
52k+2 = 24(25t + 1) + 1 ou 52(k+1) 1 = 24q
onde q = 24t + 1 ou 24 | (52(k+1)1).
5) 5 | (8n 3n) n N.
Soluo: P(1) verdadeira, pois 5 | (81 31) Suponhamos que para P(k) verdadeira
5 | (8k 3k) Logo 8k 3k = 5t, onde t um nmero inteiro. Vamos mostrar que P(k + 1)
verdadeira: 8k+1 3k+1 = 8.8k 3.3k = 8.8k (8.3k 5.3k)
= 8.8k 8.3k + 5.3k
= 8(8k 3k) + 5.3k
= 8.5t + 5.3k
= 5(8t + 3k) fazendo 8t + 3k = q
= 5q Logo 5 | (8k+1 3k+1).
6) 4n > n4 n 5.
Soluo: P(5) verdadeira, pois 45 > 54 Suponhamos que para P(k) verdadeira, k > 5
4k > k4 Multiplicando por 4 ambos os membros
4.4k > 4k4 ou 4k+1 > k4 + 3k4 Vamos usar a desigualdade n4 > 4n3 + 6n2 + 4n + 1
4k+1 > k4 + 3k4 > k4 + 4k3 + 6k2 + 4k + 1 ou 4k+1 > (k + 1)4 Logo P(k + 1) verdadeira
7) Demonstrar que 10n + 1 9n 10 um mltiplo de 81 para todo inteiro positivo n.
Soluo: P(1) verdadeira, pois 81 | (101+1 9 1 10) ou 81 | 81.
Suponhamos que para P(k) verdadeira: 81 | (10k+1 9k 10) Logo 10k+1 9k 10 = 81t
Multiplicando ambos os membros por 10.
10.10k+1 10.9k 10.10 = 10.81t
10k+2 9k 81k = 10.81t + 102
10k+2 9k 9 = 10.81t + 102 + 81k 9
10k+2 9(k + 1) 10 = 10.81t + 102 + 81k 9 10
10k+2 9(k + 1) 10 = 10.81t + 81k + 81
10k+2 9(k + 1) 10 = 81(10t + k + 1)
10k+2 9(k + 1) 10 = 81q fazendo 10t + k + 1 = q Logo 81 | [10k+2 9(k + 1) 10 }].
n 3 n 5 7n
8) Demonstrar que
um inteiro positivo para todo n N.
3
5 15
n 3 n 5 7n
5n 3 3n 5 7 n ,
Soluo:
=
isto : 15 | (5n3 + 3n5 + 7n) n N.
3
5 15
15
3
P(1) verdadeira, pois 15 | (5 1 + 3 15 + 7 1) ou 15 | 15.
Suponhamos que para P(k) verdadeira: 15 | (5k3 + 3k5 + 7k) ou 5k3 + 3k5 + 7k = 15t. Vamos
mostrar que.
P(k + 1) verdadeira. Somando a expresso 15k4 + 30k3 + 45k2 + 30k + 15 a ambos os membros da
igualdade acima, temos:
5k3 + 3k5 + 7k + 15k4 + 30k3 + 45k2 + 30k + 15 = 15t + 15k4 + 30k3 + 45k2 + 30k + 15.
Arranjando os termos convenientemente, temos:
5k3 + 15k2 + 15k + 5 + 3k5 + 15k4 + 30k3 + 30k2 + 15k + 3 + 7k + 7 = 15(t + k4 + 2k3 + 3k2 + 2k +
1).
5(k3 + 3k2 + 3k + 1) + 3(k5 + 5k4 + 10k3 + 10k2 + 5k + 1) + 7(k + 1) = 15(t + k4 + 2k3 +3k2 + 2k +
1).
5(k + 1)3 + 3(k + 1)5 + 7(k + 1) = 15q fazendo t + k4 + 2k3 +3k2 + 2k + 1 = q Assim.
15 | [5(k + 1)3 + 3(k + 1)5 + 7(k + 1)] o que queramos provar.

Questes Propostas
01) Demonstrar por induo matemtica, as questes abaixo:
1 1
1
1) 1 + 3 + 5 + ... + (2n 1) = n2 n .
10) 1
...
4 9
n2
2)

1
21

1
1
n
2
2
2

3) 1 r r 2 r n

2
2
n
2
3
3

1
1
1
3
n

1
, n
n

1
, n .
3n

n(n 1)
, n .
2

. 13) 20 21 22 2n

2n 1, n
n(n 1)
2

n4
, n .
4
n , a , a

15) 13 23 33 n 3

n .

1
2

2
31

12) 1 2 3 n

(n 1)(4 + 3n)
, n
2

1
1
1

2 3 3 4
n (n+1)

9) (1 1) 1

n .

6) 6 | n(n +1)(n + 2)

1
1 2

11)

n(n 1)(2n + 1)
, n . 14) 13 23 33 n 3
6

5) 12 22 32 n 2

8)

n .

1 rn + 1
, n
1 r

4) 2 5 8 (2 3n)

7) 2 | (n2 + n)

1
2n

16) (1 a)n 1 na,


n
n 1

, n

n 1, n

17) 3 | (22n 1)
.18) 3 | (n3 + 2n)

1.

n .
n .

02) Usando o principio da "induo matemtica", demonstrar:


1) O nmero de diagonais de um polgono convexo de n lados : d n

n(n 3)
.
2

2) A soma das medidas dos ngulos internos de um polgono convexo de n lados :


Sn (n 2) 1800 .
3) Se A um conjunto finito com n elementos, ento
elementos.

(A) , conjunto das partes de A, tem 2n

4) Prove que a soma de uma PG ou razo q de n termos e primeiro termo a1 dado por
a1 (q n 1)
.
Sn
q 1
5) Prove que uma P.G. de primeiro termo a1 e razo q o produto (Pn) dos n primeiros termos dado
por Pn

a1n q

n(n - 1)
2

n 1.

6) Prove que numa PA. de primeiro termo a1 e razo r a soma (Sn) dos n primeiros termos dado
n(n 1)
por Sn na1
r.
2
7) Para r + , e n 0, mostre que r (cos + isen )

r n (cos n + isen n) , onde i2 = -1.

8) Sendo z um nmero complexo no-nulo e n 0 , mostre que ( z n ) ( z ) n .

9) Prove que numa o termo geral da P. A. dado pela formula a n


10) Prove que (a + b)n

11) Seja A

0 n
n

cos

sen

sen

cos

12) Para x e n

1 n-1 n
n

Ca Ca

n n
n

C b , para n

. Determine An, para n

1, mostre que sen nx

13) Demonstrar a lei de De Morgan (A

B) '

a1 (n 1)r .

1.

n sen x .
A'

B' , sobre n conjuntos.

UNIDADE III DIVISIBILIDADE


3.1 - Introduo:
Sabemos, desde a escola bsica, que quando um nmero inteiro e dividido por um segundo
nmero inteiro no nulo, o quociente pode ou no ser um numero inteiro. Esta observao nos leva
a seguinte definio.
3.2 - Divisibilidade:
Definio 3.2 - Sejam a e b dois inteiros, com a

0. Diz-se que a divide b se, e somente se, existe

um inteiro q tal que b = aq.


Se a divide b tambm se diz que a divisor de b, que b mltiplo de a, que a um fator de b ou que
b divisvel por a.
Notao: a | b (a

0 divide b e portanto, a notao a | b significa que a

0 no divide b).

A relao a divide b (a | b) denomina-se relao de divisibilidade em .


Observao: Se a | b, ento a | b.
Teorema 3.1: Quaisquer que sejam os inteiros a, b e c tem-se:
(1) a | 0, a 0, 1 | a e a | a, a 0.
(2) Se a | 1, ento a =

1.

(3) Se a | b e se c | d, ento ac | bd.


(4) Se a | b e se b | c, ento a | c.
(5) Se a | b e se b | a, ento a =
(6) Se a | b com b

b.

0, ento | a |

| b |.

(7) Se a | b e se a | c, ento a |(bx + cy) para todos x e y em .


3.3 - Conjunto de divisores de um inteiro:
D(a) = {x *| x | a}.
3.4 - Divisores comuns de dois inteiros:
Definio 3.3: Chama-se divisor comum de dois inteiros a e b todo inteiro d
Notao: D(a, b) = {x

* | x | a e x | b}

Propriedade; D(a, b) = D(a)


Obs.: D(a, b)

0 tal que d | a e d | b.

D(b).

; D(0, 0) = *.

3.5 - Algoritmo da Diviso:


Teorema 3.2: Se a e b so dois inteiros, com b > 0, ento existem e so nicos os inteiros q e r que
satisfazem s condies: a = bq + r e 0

r < b.

Corolrio 3.2: Se a e b so dois inteiros com b


satisfazem s condies: a = bq + r, 0

r < | b |.

0, existem e so nicos os inteiros q e r que

3.6 - Paridade de um Inteiro:


Na diviso de um inteiro qualquer a por b = 2 os possveis restos so r = 1 e r = 0. Se r = 0 ento, o
inteiro a = 2q denominado par; e se r = 1, ento o inteiro a = 2q + 1 denominado mpar.
Questes Resolvidas
01) Mostrar que, se a um inteiro qualquer, ento um dos inteiros a, a + 2, a + 4 divisvel por 3.
Soluo: Quando dividimos um inteiro a qualquer por 3 , os restos so 0, 1 ou 2.
Assim a = 3q ou a = 3q + 1 ou a = 3q + 2.
Se a = 3q, ento 3 | a
Se a = 3q + 1, a + 2 = 3q + 1 + 2 ou a + 2 = 3(q + 1) ento 3 | (a + 2).
Se a = 3q + 2, a + 4 = 3q + 2 + 4 ou a + 4 = 3(q + 2) ento 3 | (a + 4).
02) Sendo a um inteiro qualquer, mostrar:
a) 2 | a(a + 1).
Soluo: Como a e a + 1 so inteiros consecutivos, ento um dos dois par, sendo o outro mpar;
ento a(a + 1) = 2k1(2k2 + 1) onde 2k1 representa o nmero par e 2k2 + 1 representa o nmero mpar.
Assim 2 | a(a + 1).
03) 3 | a(a + 1)(a + 2).
Soluo: Dividindo a por 3, temos trs hipteses. a = 3q ou a = 3q + 1 ou a = 3q + 2
Se a = 3q ento 3 | a e por conseguinte 3 | a(a + 1)(a + 2)
Se a = 3q + 1 ento a + 2 = 3q + 1 + 2 ou a + 2 = 3(q + 1); logo 3 | (a + 2) e por conseguinte 3 | a(a
+1)(a + 2)
Se a = 3q + 2 ento a + 1 = 3q + 2 + 1 ou a + 1 = 3(q + 1); logo 3 | (a + 1) e por conseguinte 3 | a(a
+ 1)(a + 2)
04) Mostrar que todo inteiro mpar da forma 4k + 1 ou 4k + 3.
Soluo: Quando dividimos um inteiro por 4 os restos possveis so: 0, 1, 2 ou 3. Se o nmero for
mpar, os restos s podero ser 1 ou 3 e, assim, n = 4k + 1 ou n = 4k + 3 onde n o nmero mpar.
05) Mostrar que o quadrado de um inteiro qualquer da forma 3k ou 3k + 1.
Soluo: Dividindo um inteiro por trs, obtemos os restos 0, 1 ou 2. Sendo assim podemos
escrever: a = 3k1, a = 3k1 + 1 ou a = 3k1 + 2, onde a um inteiro qualquer
Se a = 3k1, a2 = 9 k12 ou a2 = 3(3k12). Fazendo 3k12 = k, temos a = 3k
Se a= 3k1 + 1, a2 = 9k12 + 6k1 + 1 ou a2 = 3(3k12 +2k1) + 1. Fazendo 3k12 +2k1 = k temos a2 = 3k + 1
Se a = 3k1 + 2, a2 = 9k12 + 12k1 + 4 ou a2 = 9k12 + 12k1 + 3 + 1; assim
a2 = 3(3k12 +4k1 + 1) + 1. Fazendo 3k12 +4k1 + 1 = k temos a2 = 3k + 1.

06) Mostrar que

n( n 1)(2 n 1)
um inteiro qualquer que seja o inteiro positivo n.
6

Soluo: Devemos provar que n(n + 1)(2n + 1) mltiplo de 6.


Como n e n + 1 so inteiros consecutivos, ento um dos dois par, logo mltiplo de 2.
Vamos mostrar que um dos nmeros n, n + 1, 2n + 1 mltiplo de 3.
Se n no for mltiplo de 3, ento n = 3q + 1 ou n = 3q + 2.
Se n = 3q + 1, 2n + 1 = 6q + 2 + 1 ou 2n + 1 = 3(2q + 1). Logo 2n + 1 mltiplo de 3.
Se n = 3q + 2, n + 1 = 3q + 2 + 1 ou n + 1 = 3(q + 1). Logo n + 1 mltiplo de 3.
Ento podemos concluir que um dos trs nmeros n, n + 1, 2n + 1 mltiplo de 3.
Podemos, ento, escrever: n(n + 1)(2n + 1) = 2t13t2t3, substituindo o fator mltiplo de 2 por 2t1 e o
fator mltiplo de 3 por 3t2, sendo t3 outro inteiro qualquer. Conclumos que n(n + 1)(2n + 1) = 6t1t2t3
mltiplo de 6, como queramos demonstrar.
Demonstrar:
07) Se a um inteiro mpar, ento 24 | a(a2 1).
Soluo: a(a2 1) = a(a 1)(a + 1). Como a mpar, a = 2t + 1, a 1 = 2t e a + 1 = 2t + 2.
Ento a(a2 1) = (2t + 1)2t(2t + 2) ou a(a2 1) = (2t + 1) 2t2(t + 1) ou a(a2 1) = 4t(t + 1)(2t + 1).t(t
+ 1)(2t + 1) mltiplo de 6.
Portanto a(a2 1) = 4.6 k ou a(a2 1) = 24 k; conclumos que 24 | a(a2 1).
08) Se a e b so inteiros mpares, ento 8 | (a2 b2).
Soluo: a2 b2 = (a - b)(a + b). Como a e b so mpares a = 2q + 1 e b = 2k + 1. Onde q e k so
inteiros quaisquer. Ento: a2 b2 = 2q + 1 (2k + 1) (2q + 1 + 2k + 1)
a2 b2 = (2q 2k)(2q + 2k + 2) ou a2 b2 = 2(q k)2(q + k + 1) ou
a2 b2 = 4(q k)(q + k + 1). Quaisquer que sejam as paridades de q e k, q k e q + k tem a mesma
paridade: deste modo q - k e q + k + 1 tm paridades diferentes, isto , um par e o outro mpar.
Assim (q k)(q + k + 1) = 2t(2t + 1). Substituindo na expresso acima, obtemos: a2 b2 = 4.2t(2t +
1) ou a2 b2 = 8t(2t + 1) e logo 8 | (a2 b2 ).
09) Mostrar que a diferena entre os cubos de dois inteiros consecutivos nunca divisvel por 2.
Soluo: Sejam os inteiros consecutivos n e n + 1. Achemos a diferena entre os cubos destes
nmeros:
(n + 1)3 n3 = n3 + 3n2 + 3n + 1 n3
= 3n2 + 3n + 1
= 3n(n + 1) + 1. Como n e n + 1 so consecutivos, um dos dois par e, portanto, o
produto 3n(n + 1) tambm par e 3n(n + 1) + 1 mpar. Logo no divisvel por 2.

10) Na diviso do inteiro a = 427 por um inteiro positivo b o quociente 12 e o resto r. Achar o
divisor b e o resto r.
Soluo:: De acordo com o algoritmo da diviso 427 = 12b + r, 0

b ; onde 427 o dividendo,

b o divisor , 12 o quociente e r o resto. Desta igualdade tiramos:


427 12b = r e da 0
12b

427 12b < b. Somando 12b aos trs membros da desigualdade, obtemos:

427 < 13b. Da desigualdade 12b

427, tiramos b

Da desigualdade 427 < 13b tiramos b > 32,8 ou b

35, 5 ou b

35.

33. Assim os valores para b so os inteiros no

intervalo 33,35 ou 33, 34 e 35.


Para b = 33, r = 427 12 x 33 = 31. Para b = 34, r = 427 12 x 34 = 19. Para b = 35, r = 427 12
x 35 = 7.
11) Achar um inteiro de quatro algarismos, quadrado perfeito, divisvel por 27 e terminado em 6.
Soluo: Se a, b, c ... so fatores primos, os expoentes desses fatores devem ser par.
Como 27 = 33, deve-se ter pelo menos mais um 3 como fator. Portanto, o nmero deve ser mltiplo
de 27 x 3 ou de 81. Para que o nmero termine em 6, devemos multiplicar 81 por um quadrado
(pois 81 j quadrado), terminado em 6, pois 81 termina em 1. Assim, temos as possibilidades 81 x
16 = 1296 e 81 x 36 = 2916.
Se o nmero tivesse 6 fatores iguais a 3, ele deveria ser mltiplo de 729. Para que terminasse
em 6, deveriamos ter 729 x a, com a terminado em 4. Como os menores quadrados terminados em
quatro so 4 e 64, teramo: 729 x 4 = 2916 e 729 x 64 = 46656 que tem 5 algarismos. Para 8 fatores
iguais a 3, o nmero deveria ser mltiplo de 6561 = 38. Para que o nmero terminasse em 6,
deveriamos ter 6561 x a, com a terminado em 4. Como os menores quadrados terminados em quatro
so 4 e 64, teramos 6561 x 4 = 26244 que contm cinco algarismos. Para 10 fatores iguais a 3,
teramos 310 > 10000, que ter mais de 4 algarismos.
Portanto, os nicos nmeros so 1296 e 2916.
12) Demonstrar que se m e n so inteiros mpares, ento 8 | (m4 + n4 2).
Soluo: se m e n so mpares, podemos escrever: m = 2k + 1 e n = 2k + 1.
Temos ento: m4 + n4 - 2 = (2k + 1)4 + (2k + 1)4 2 = [(2k)4 + 4(2k)3 + 6(2k)2 + 4(2k) + 1] +
[(2k)4 + 4(2k')3 + 6(2k)2 + 4(2k)+1] 2 = 16(k4 + k4) + 32(k3 k3) + 24(k2 + k2) + 8(k + k) +
2 2 = 8[2(k4 + k4) + 4(k3 k3) + 3(k2 + k2) + (k + k)]. Como 2(k4 + k4) + 4(k3 k3) + 3(k2 +
k2) + (k + k)]. um inteiro (multiplicao e adio de inteiros), podemos escrever: m4 + n4 - 2 =
8q, q inteiro 8 | (m4 + n4 2).

Questes Propostas
01) Mostrar que, se a | b, ento (-a) | b e a | (-b) e (-a) | (-b).
02) Sejam a, b e c inteiros. Mostrar:
a) se a | b, ento a | bc.
b) Se a | b e se a | c, ento a2 | bc.
c) a | b se, e somente se, ac | bc, (c

0).

03) Mostrar que um inteiro qualquer da forma 6k + 5 tambm da forma 3k + 2.


04) Mostrar que o cubo de um inteiro qualquer de uma das formas: 9k, 9k + 1 e 9k + 8.
05) Mostrar que, se a | (2x 3y) e se a | (4x 5y), ento a | y.
06) Determinar os inteiros positivos que divididos por 17 deixam um resto igual ao quadrado do
quociente. R: 18, 38, 60 e 84.
07) Na diviso do inteiro 525 por um inteiro positivo o resto 27. Achar os inteiros que podem ser
o divisor e o quociente. R: b = 498 e q = 1; b = 249 e q = 2; b = 166 e q = 3 e b = 83 e q = 6.
08) Na diviso de dois inteiros positivos o quociente 16 e o resto o maior possvel. Achar os dois
inteiros, sabendo que a sua soma 341. R: a = 322, b = 19.
09) Achar os inteiros positivos menores que 150 e que divididos por 39 deixam um resto igual ao
quociente. R: q = 1, 2, 3 e a = 40, 80, 120.
10) Seja d um divisor de n (d | n). Mostrar que cd | n se, e somente se, c |

n
.
d

11) Mostrar que para todo inteiro n, existem inteiros k e r tais que n = 3k + r e r = 1, 0, 1.
12) Mostrar que todo inteiro mpar, quadrado perfeito, da forma 4n + 1.
13) Na diviso de 392 por 45, determinar:
a) o maior inteiro que se pode somar ao dividendo sem alterar o quociente. R: 12.
b) o maior inteiro que se pode subtrair do dividendo sem alterar o quociente. R: 32.
14) Numa diviso de dois inteiros, o quociente 16 e o resto 167. Determinar o maior inteiro que se
pode somar ao dividendo e ao divisor sem alterar o quociente. R: 11.
15) Achar o maior inteiro de quatro algarismos divisvel por 13 e o menor inteiro de cinco
algarismos divisvel por 15.
R: maior 9997 e o menor 10.005.

UNIDADE IV MXIMO DIVISOR COMUM - M. D. C


4.1 - Introduo:
Fazem parte do ensino fundamental, entre outras, as noes de Mximo Divisor Comum
(MDC), Mnimo Mltiplo Comum (MMC), Nmeros primos e Fatorao, que compem uma
parcela significativa da Teoria Elementar dos Nmeros. No caso especfico M.D.C, pretendemos
mostrar a relevncia destes atravs da abordagem de exerccios para o aprofundamento terico e
aps o estudo do M.M.C veremos aplicaes de forma contextualizada.
4.2 - Mximo Divisor Comum:
Definio 4.1 - Dados dois ou mais nmeros inteiros no nulos denominamos mximo divisor
comum destes nmeros ao maior nmero inteiro que divisor simultaneamente de todos os
nmeros dados. O mdc o maior elemento da interseco dos conjuntos dos divisores positivos dos
nmeros dados.
Definio 4.2 - Sejam a e b dois inteiros no conjuntamente nulos (a

0 ou b

0). Chama-se

mximo divisor comum de a e b o inteiro positivo d (d > 0) que satisfaz s condies:


1) d | a e d | b.
2) Se c | a e c | b, ento c

d.

Notao: d = mdc(a, b).


4.3 - Existncia e Unicidade do MDC:
Teorema 4.1 - Se a e b so inteiros no conjuntamente nulos (a

0 ou b

0), ento existe e nico

o mdc(a,b); alm disso, existem inteiros x e y tais que mdc(a, b) = ax + by, isto , o mdc(a, b) uma
combinao linear de a e b.
Teorema 4.2 - Se a e b so dois inteiros no conjuntamente nulos (a
de todos os mltiplos do mdc(a, b) = d T = {ax + by | x,y

0 ou b

0), ento o conjunto

0 ou b

0). Diz-se que a e b

Z}.

4.4 - Inteiros Primos entre si:


Definio 4.3 - Sejam a e b dois inteiros no conjuntamente nulos (a
so primos entre si se, e somente se, o mdc(a, b) = 1.
Teorema 4.3 - Dois inteiros a e b, no conjuntamente nulos (a

0 ou b

e somente se, existe inteiros x e y tais que ax + by = 1.


Corolrio 4.1 - Se o mdc(a, b) = d, ento mdc

a b
,
d d

= 1.

Corolrio 4.2 - Se a | b e se mdc(b, c) = 1, ento o mdc(a, c) = 1.


Corolrio 4.3 - Se a | c, se b | c e se mdc(a, b) = 1, ento ab | c.
Corolrio 4.4 - Se mdc(a, b) = 1 = mdc(a, c), ento o mdc(a, bc) = 1.

0), so primos entre si se,

Corolrio 4.5 - Se mdc(a, bc) = 1, ento mdc(a, b) = 1 = mdc(a, c).


Teorema 4.4 - (de Euclides) Se a | bc e se o mdc(a, b) = 1 ento a | c.
4.5 - Caracterizao do MDC de dois Inteiros:
Teorema 4.5 - Sejam a e b dois inteiros no conjuntamente nulos (a

0 ou b

0). Um inteiro

positivo d (d > 0) o mdc(a, b) se, e somente se, satisfaz s condies:


1) d | a e d | b.
2) Se c | a e c | b, ento c | d.
4.6 - Caracterizao do MDC de dois Inteiros:
O conceito de mximo divisor comum, definido para dois inteiros a e b, estende-se de maneira
natural a mais de dois inteiros. No caso de trs inteiros a, b e c, no todos nulos, o mdc (a, b, c) o
inteiro positivo d (d > 0) que, satisfaz s condies:
1) d | a, d | b e d | c.
2) Se e | a, se e | b e se e | b, ento e

d.

4.7 - MDC de Vrios Inteiros:


Teorema 4.6 - O mdc(a,b,c) = mdc(mdc(a,b), c).
Questes Resolvidas
01) Determinar:
(a) mdc(11, 99).
Soluo: 99: 11 = 9, resto zero mdc(11,99) = 11
R: 11.
(b) mdc(-21, 14).
Soluo: mdc(-21, 14) = mdc(21, 14)
21: 14 = 1 resto 7.
14:7 = 2, resto zero mdc(-21, 14) = 7
R: 7.
(c) mdc(17, 18).
Soluo: 18: 17 = 1, resto 1
17: 1 = 17 resto 0 mdc(17, 18) = 1.
R: 1.
02) Achar o menor inteiro positivo c da forma c = 22x + 55y onde x e y so dois inteiros.
Soluo: Como o menor inteiro da forma 22x + 55y o mdc(22, 55) ento c = 11.
03) Calcular mdc(n, n + 2), sendo n um inteiro par.

Soluo: Seja d = mdc(n, n+2). Ento d | n e d | (n+2). Como d | n e d | (n+2), ento, d | 2 e portanto,
d = 1 ou d = 2 e como n par a resposta ser d = 2 (o maior dos divisores).
04) Calcular mdc(n, n + 2), sendo n um inteiro mpar.
Soluo: Seja d = mdc(n, n+2). Ento, usando o mesmo raciocnio anterior, d = 1 ou d = 2. Mas
como n mpar, conclumos que d = 1.
05) Sendo n um inteiro qualquer, calcular o mdc(n, n + 1)
Soluo: Seja d = mdc(n, n+1). Ento d | n e d | (n+1). Como d | n e d | (n+1), ento d | 1. Logo d =
1.
Sejam a, b e c inteiros. Demonstrar:
06) Existem inteiros x e y tais que c = ax + by se, e somente se, mdc(a, b) | c.
Soluo: Demonstraremos primeiramente, a ida:
Seja d = mdc(a, b). Ento d | a e d | b e logo d | (ax + by) quaisquer que sejam os inteiros x e y.
Portanto d | c, desde que c = ax + by por hiptese.
Soluo: Demonstraremos agora a volta:
Seja d = mdc(a, b). Como d | c, temos c = dq. Sendo d o mdc(a, b) existem inteiros x e y tais que d
= ax + by. Substituindo este valor na igualdade c = dq obtemos: c = (ax + by )q. Da tiramos valor
c = a(xq) + b(yq). Fazendo xq = x e yq = y temos c = ax + by.
07) Se existem inteiros x e y tais que ax + by = mdc(a, b), ento o mdc(x, y) = 1
Soluo: Seja d = mdc(a, b). Ento existem inteiros x e y tais que d = ax + by, onde d | a e d |b.
Dividindo ambos os membros por d ( d > 0), obtemos:

a
x
d

b
y = 1 Como d | a e d | b as
d

expresses entre parntesis so inteiras, que representaremos por k1 e k2 obtendo assim: xk1 + yk2 =
1, donde conclumos que mdc(x, y) = 1.
Determinar inteiros positivos a e b sabendo:
08) a + b = 63 e o mdc(a, b) = 9.
Soluo: Se 9 = mdc(a, b), temos que 9 | a e 9 | b ou a = 9q1 e b = 9q2. Substituindo estes valores na
igualdade a + b = 63 temos: 9q1 + 9q2 = 63; da, dividindo por 9 obtemos:
q1 + q2 = 7. Como q1 e q2 so primos entre si, devemos procurar inteiros positivos primos entre si
que somem 7. Temos os seguintes valores:
q1 = 1 e q2 = 6; q1 = 2 e q2 = 5 e q1 = 3 e q2 = 4. Assim obtemos para a e b os seguintes valores:
a = 9 e b = 54; a = 18 e b = 45 e a = 27 e b = 36.
09) ab = 756 e o mdc(a, b) = 6.

Soluo: Se 6 = mdc(a, b) temos que 6 | a e 6 | b ou a = 6q1 e b = 6q2. Substituindo na igualdade a b


= 756, temos 6q16q2 = 756. Da obtemos que q1q2 = 21. Do mesmo modo, como q1 e q2 so primos
entre si, devemos encontrar inteiros positivos cujo produto d 21. Os valores so: q1 = 1 e q2 = 21 e
q1 = 3 e q2 = 7. Destes valores tiramos os valores de a e b: a = 6 e b = 126 e a = 18 e b = 42.
10) Demonstrar que, se n = abc + 1, ento o mdc (n, a) = mdc(n, b) = mdc(n, c) = 1.
Soluo: Provemos que mdc(n, a) = 1. O mesmo pode ser feito para b e c.
Seja d = mdc(n, a). Ento d | n e d | a. Podemos dizer, portanto, que d | abc, mltiplo de a. Como d |
n e d | abc, ento d | 1, pois n abc = 1. Logo d = 1.
11) O mdc(a, 4) = 2 = mdc(b, 4). Demonstrar que o mdc(a + b, 4) = 4.
Soluo: Temos mdc(a, 4) = 2 e mdc(b, 4) = 2. Conclumos que a e b so nmeros pares e no so
mltiplos de 4 , pois se o fossem, 2 no seria o mdc entre eles. Logo o resto da diviso de a e b por 4
2. Assim a = 4q1 + 2 e b = 4q2 + 2. Somando membro a membro estas desigualdades, temos a + b
= 4q1 + 2 + 4q2 + 2 ou a + b = 4 (q1 + q2 + 1). Logo, 4 | (a + b) e por conseguinte mdc(a + b, 4) = 4.
12) Sendo a e b dois inteiros no conjuntamente nulos (a

0 ou b

0), mostrar:

mdc(a, b) = mdc(-a, b) = mdc(a, -b) = mdc(-a, -b).


Soluo: Se c | a ento a = qc. Temos que - a = (-q)c c | (-a) todo divisor de a divisor de (-a)
maior divisor de a tambm o maior divisor de a. O mesmo ocorre com b e b. Portanto,
podemos concluir que o maior divisor comum de (a e b), tambm de (a e b), de (a e b) e o de (a, -b). Assim, mdc(a, b) = mdc(-a, b) = mdc(a, -b) = mdc(-a, -b).
13) Demonstrar que mdc(mdc(a, b), b) = mdc(a, b).
Soluo: A definio do mdc de trs nmeros mdc(a, b, c) = mdc(mdc(a, b), c), quaisquer que
sejam a, b e c. Fazendo c = b, temos mdc(mdc(a, b), b) = mdc(a, b, b) = mdc(a, mdc(b, b)) = mdc(a,
b) pois mdc(b, b) = b.
14) Demonstrar que o mdc(n + k, k) = 1 se e somente se o mdc(n, k) = 1.
Soluo: Se mdc(n + k, k) = 1, ento existem os inteiros x e y, tais que (n + k)x+ ky = 1 nx +
k(a + b) = 1 (n, k) = 1.
Por outro lado, se mdc(n, k) = 1, ento existem a e b tais que na + kb = 1. Fazendo a = x e b = x + y,
teremos nx + k(x + y) = 1 (n + k)x + ky = 1 mdc(n + k), k) = 1.
15) Calcular o mdc(a + b, a b) sabendo que a e b so inteiros primos entre si.
Soluo:
Se a e b so primos entre si, no podem ser ambos pares, pois o mdc seria 2 ou mltiplo de 2.
Portanto, a e b so ambos mpares ou so de paridades diferentes.

(1 caso) a e b com paridades diferentes (a = 2k + 1 b = 2k)


Temos ento:
a + b = 2k + 1 + 2k = 2(k + k) + 1 = 2n + 1 a + b mpar.
a b = 2k + 1 2k = 2(k k) + 1 = 2m + 1 a b mpar.
Portanto, o mdc(a + b, a b) um nmero mpar.
Seja ento mdc(a + b, a b) = 2k + 1 existem x e y tais que (a + b)x + (a b)y = 2k + 1
[(a + b)/(2k+1)]x + [(a b)/(2k + 1)]y = 1 (a + b)/(2k + 1) e (a b)/(k + 1) so primos entre si.
Fazendo r = (a + b)/(2k + 1) e s = (a b)/(2k + 1), resulta: a + b = r(2k + 1) (i) e a b = s(2k + 1)
(ii).Como (a + b), (a b) e (2k + 1) so mpares, r e s tambm so mpares.
Alm disso r e s mpares, r + s e r s so pares.
Somando membro a membro as igualdades (i) e (ii), resulta:
(1) 2a = (2k + 1)(r + s) a = (2k + 1)[(r + s)/2] pois s + r par (inteiro), portanto 2 | (r + s).
Assim, existe o inteiro (r + s)/2, tal que a = (2k + 1)[(r + s)/2) 2k + 1 | a.
Subtraindo membro a membro as igualdades (i) e (ii),
(2) 2b = (2k + 1)(r s) b = (2k + 1)[(r s)/2]. (r s) par. Portanto, (r s)/2 inteiro.
Assim, existe o inteiro (r s)/2, tal que b = (2k + 1)[(r s)/2] 2k + 1 | b.
Ora, a e b so primos entre si. Portanto, o nico divisor comum 2k + 1. Disto permite-se escrever
2k + 1 = 1 1 = mdc(a + b, a b).
(2 caso) a e b so mpares a = 2k + 1 e b = 2k + 1.
Temos, ento:
(a + b) = 2k + 1 + 2k + 1 (a + b) = 2(k + k + 1)
(a b) = 2k + 1 2k 1 = 2(k k)
Das igualdades acima, conclumos que (a + b) e (a b) so pares. Portanto, o mdc da forma 2k.
Assim, existem x e y, tais que: (a + b)x + (a b)y = 2k r = (a + b)/2k e s = (a b)/2k so primos
entre si. (a + b) = 2kr (i) e (a b) = 2ks (ii).
Somando membro a membro, 2a = 2k(r + s) a = k(r + s) k | a.
Subtraindo membro a membro, 2b = 2k(r s) b = k(r s) k | b.
Como a e b so primos entre si, o nico divisor comum de a e b 1. Portanto, k = 1 e Mdc(a + b, a
b) = 2k mdc(a + b, a b) =2.1 = 2.
Portanto, se a e b so primos ento mdc(a + b, a b) 1 ou 2.
16) O mdc de dois inteiros positivos 10 e o maior deles 120. Determinar o outro inteiro.
Soluo: Seja a o outro nmero. a deve ser um mltiplo de 10 que no divide 120. Como o maior
dos nmeros e 120, a deve ser menor que 120. Os mltiplos de 10 que no dividem 120 so: 50, 70,
90 e 110.

17) O mdc(a, b) = p, sendo p um primo. Achar os possveis valores do:


( a ) mdc(a2, b).
Soluo: Sejam a = p.a1.a2.a3 ...an, onde p, a1, a2, a3, ... an so os fatores primos de a e b =
p.b1.b2.b3...bn, onde p, b1, b2, b3, ...bn so os fatores primos de b. Assim, a2 = p.p.a1.a2.a2.a3a3...an.an
que a2 e b so divisveis ao mesmo tempo apenas por p. mdc(a2, b) = p.
( b ) mdc(a3, b) = p, mesma concluso acima.
( c ) mdc(a2, b3) = p2. Pois aparecem 2 fatores iguais a p em a2 e 3 fatores iguais a p em b3.
18) Sejam a e k inteiros no conjuntamente nulos. Demonstrar que mdc(a, a + k) | k.
Soluo: Seja m o mdc(a, a + k). Assim, existem os inteiros x e y tais que: a = mx e a + k = my.
Subtraindo primeira igualdade da segunda resulta: (a + k) a = my mx k = m(y x). Como x e
y so inteiros, y - x inteiro. Portanto, existe o inteiro (y x) tal que k = m(y x) m | k ou
mdc(a, a + k) | k.
19) Seja o quadrado abaixo em que cada lado mede 3 cm. Quantos quadradinhos de 1cm cabem no
quadrado?

R: 9 quadradinhos.
Com o mesmo quadrado acima, obter o valor de 3. R: 3 = 9.
20) De quantos cubinhos de 1cm de lado, isto , um centmetro cbico, precisaremos para construir
um cubo com 3cm de comprimento, 3cm de largura e 3cm de altura?

R: 27 cubinhos.

Questes Propostas
01)Achar os elementos do conjunto A = {1, 2, 3, 4, 5} que so primos com 8. R: 1, 3 e 5.
02) Sabendo que o mdc(a, 0) = 13, achar todos os valores do inteiro a. R: 13
03) Sendo n um inteiro qualquer, calcular o mdc(n, n + 1). R: 1.
04) Sendo n um inteiro qualquer, achar os possveis valores do mximo divisor comum dos inteiros
n e n + 10. R: 1, 2, 5, 10.
05) Sendo n um inteiro qualquer, calcular o mdc(n 1, n2 + n + 1). R: 1 ou 3.
06) Sejam a, b e c inteiros. Demonstrar:
( a ) se o mdc(a, b) = 1 ento o mdc(ac, b) = mdc(b, c) Portanto, todo divisor de d divisor de ac.
( b ) Se o mdc(a, b) = 1 e se c | (a + b), ento o mdc(a, c) = 1 e o mdc(b, c) = 1.
( c ) se b | c, ento o mdc(a, b) = mdc(a + c, b).
( d ) Se mdc(a, b) = 1, ento mdc(am, bn) = 1, onde m e n so inteiros positivos.
07) Achar o maior inteiro positivo pelo qual se devem dividir os inteiros 160, 198 e 370 para que os
restos sejam respectivamente 7, 11 e 13. R: 17.
08) Os restos das divises dos inteiros 4933 e 4435 por um inteiro positivo n so respectivamente
37 e 19. Achar o inteiro n. R: n = 96 ou n = 48.
09) Demonstrar que, se a | bc e se mdc(a, b) = d, ento a | cd.
10) Demonstrar que, se a | c, se b | c e se o mdc(a, b) = d ento ab | cd.
11) Demonstrar que se mdc(a, b) = 1 e se mdc(a,c) = d,ento mdc(a, bc) = d.
12) O inteiro mpar d um divisor de a + b e de a b. Demontrar que d tambm um divisor do
mdc(a, b).
13) Os inteiros positivos m e n so tais que o mdc(m, n) = d. Mostrar que o mdc(2m 1, 2n 1) = 2d
1.
14) Demonstrar que mdc(a, b) = mdc(a, b, a + b).
15) Demonstrar que mdc(a, b) = mdc(a, b, ax + by), quaisquer que seja os inteiros x e y.
16) Demonstrar que se o mdc(a, b) = d ento o mdc(a2, b2) = d2.
17) Demonstrar que mdc(a, b) = mdc(a, c) implica mdc(a2, b2) = mdc(a2, c2).
18) Demonstrar que mdc(a, b) = mdc(a, c) implica mdc(a, b) = mdc(a, b, c).

19) Demonstrar que mdc(a, b, c) = mdc(mdc(a, b), mdc(a, c).


20) Sejam a e b inteiros positivos tais que ab um quadrado perfeito e o mdc(a, b) = 1. Demonstrar
que a e b so quadrados perfeitos.
21) Demonstrar que mdc( a + b, a b) > mdc(a, b).
22) Sejam a, b, c, d (b

d) inteiros tais que mdc(a, b) = mdc(c, d) = 1. Mostrar que a soma a/b + c/d

no um inteiro.
23) Determinar os inteiros positivos a e b, sabendo que a2 b2 = 7344 e mdc(a, b) = 12.
R: a = 312 e b = 300, ou a = 120 e b = 84.
24) Dividindo-se dois inteiros positivos pelo seu mdc, a soma dos quocientes 8. Determinar os
dois inteiros, sabendo-se que sua soma 384. R: 48 e 336 ou 144 e 240.
25) Trs rolos de arame farpado tm, respectivamente, 168 m, 264m e 312 m. Deseja-se cort-los
em partes de comprimentos iguais, de maneira que cada parte seja a maior possvel. Qual o
comprimento e o nmero de partes? R: 24 m e 31 partes.
26) Um terreno retangular de 221 m por 117 m ser cercado. Em toda a volta deste cercado sero
plantadas rvores igualmente espaadas. Qual o maior espao possvel entre as rvores? R: 13 m.
27) Em uma excurso ao parque do Caraa, em Minas Gerais, viajaram dois nibus um com 42
pessoas e outro com 30. Os guias queriam organizar grupos com o mesmo nmero de pessoas, mas
sem misturar as pessoas que vieram nos dias nibus. Eles queriam tambm que esse nmero de
pessoas por grupo fosse o maior possvel. Quantos grupos e de pessoas, foram formadas em cada
nibus?
R: Foram formados 12 grupos de 6 pessoas sendo 7 grupos no 1 nibus e 5 grupos no 2 nibus.
28) Uma tecelagem fabrica peas de tecidos em trs metragens diferentes: 45m, 60m e 105m.
Desejando cortar as peas em partes de mesmo comprimento e que esteja e que este seja o maior
possvel, qual dever ser o comprimento de cada parte? Em quantas partes cada pea ser cortada?
R: Cada parte dever ter 15m de comprimento. A pea 45m ser cortada em 3 partes, a pea de 60m
ser cortada em 4 partes e a pea de 105m em 7 partes.
29) De um aeroporto, partem todos os dias, 3 avies que fazem rotas internacionais. O primeiro
avio faz a rota de ida e volta em 4 dias, o segundo em 5 dias e o terceiro em 10 dias. Se num certo
dia os trs avies partem simultaneamente, depois de quantos dias esses avies partiro novamente
no mesmo dia?
R: 20 dias.

UNIDADE V ALGORITMO DE EUCLIDES: MNIMO MLTIMPLO COMUM M.M.C


5.1 - Introduo:
Fazem parte do ensino fundamental, entre outras, as noes de Mximo Divisor Comum
(MDC), Mnimo Mltiplo Comum (MMC), Nmeros primos e Fatorao, que compem uma
parcela significativa da Teoria Elementar dos Nmeros. No caso especfico M.M.C, pretende-se
mostrar a relevncia destes atravs da abordagem de temas atuais onde aparecem e sua conexo com
outras reas do conhecimento. Com isso, visamos a contextualizao e a interdisciplinaridade,
ambas importantes para que o aluno veja a matemtica como uma aliada na vida prtica e sua
relao com outras disciplinas. Neste sentido, busca-se que o aluno perceba que os nmeros e a
lgebra formam um sistema de cdigos ligados especialmente a diversas aplicaes.
Definio 5.1 - Dados dois ou mais nmeros inteiros no nulos denominamos mnimo mltiplo
comum destes nmeros dados ao menor nmero inteiro positivo que mltiplo simultaneamente te
todos os nmeros dados. O mmc o menor elemento da interseco dos conjuntos dos mltiplos
positivos dos nmeros dados.
Lema: 5.2 - Se a = bq + r, ento mdc(a, b) = mdc(b, r):
5.3 - Algoritmo de Euclides: Sejam a e b dois inteiros no conjuntamente nulos (a

0 ou b

0)

cujo mximo divisor comum se deseja determinar.


1) Se a

0, ento mdc(a,0) = |a|.

2) Se a

0, ento mdc(a,a) = |a|.

3) Se b | a, ento o mdc(a,b) = |b|.


4) Se a no divide b e b no divide a, ento a = bq + r e mdc(a, b) = mdc(b, r).
Dispositivo prtico para o clculo do Mximo divisor comum de dois inteiros positivos a e b
denominado Algoritmo de Euclides.
a
r1

q1
b
r2

q2
r1
r2

q3
r2
r4

qn
rn - 1
0

qn - 1
rn

Teorema 5.1 - Se k > 0, ento o mdc(ka, kb) = k mdc(a, b).


Corolrio 5.1 - Para todo k

0, o mdc(ka, kb) = |k| mdc(a, b).

5.4 - Mltiplos Comuns de dois Inteiros:


M(a) = {x

Z | a | x} = {aq | q

Z}.

M(1) = M(1) = Z.
Definio 5.2 - Sejam a e b dois inteiros diferentes de zero (a

0eb

comum de a e b todo inteiro x tal que a | x e b | x. M(a, b) = M(a)

M(b).

0). Chama-se mltiplo

5.5 - Mnimo Mltiplo Comum:


Definio 5.3 - Sejam a e b dois inteiros diferentes de zero (a

0eb

0). Chama-se mnimo

mltiplo comum de a e b o inteiro positivo m (m > 0) que satisfaz s condies:


1. a | m e b | m.
2. se a | c e b | c, com c > 0, ento m

c.

Notao: m = mmc(a, b).


Observao:

a) mmc(a, b)

ab.

b) Se a | b, ento mmc(a, b) = |b|.

5.6 - MMC de Vrios inteiros:


O conceito de mnimo mltiplo comum, definido para dois inteiros a e b, estende-se de
maneira natural a mais de dois inteiros. No caso de trs inteiros a, b e c, diferentes de zero, o
m.m.c(a, b, c) o inteiro positivo m(m > 0) que satisfaz s condies:
1) a | m, b | m e c | m.
2) Se a | e, b | e, e se c | e, com e > 0, ento m

e.

5.7 - Relao Entre o MDC e o MMC:


Teorema 5.2 - Para todo par de inteiros positivos a e b subsiste a relao mdc(a, b) mmc(a, b) = ab.
Corolrio 5.2 - Para todo par de inteiros positivos a e b, o mmc(a, b) = ab se, e somente se mdc(a,
b) = 1.
5.7 - Regra Prtica para obteno do MDC e MMC de Vrios inteiros:
Podemos determinar o mdc e o mmc de dois ou mais nmeros inteiros positivos procedendo
do seguinte modo:
1) Fatoramos os nmeros, decompondo-se em fatores primos positivos;
2) Calculamos o mdc multiplicando os fatores comuns aos nmeros, tomando cada fator uma s
vez e com o menor expoente que ele apresenta nas decomposies dos nmeros dados;
3) Calculamos o mmc multiplicando os fatores comuns e os no comuns aos nmeros, tomando
cada fator uma s vez e com o maior expoente que ele apresenta nas decomposies dos nmeros
dados;
Exemplo: Dados as seguintes decomposies de inteiros a = 32.19.712, b = 2.35.19.61 e c =
24.192.71, determine:
a) MDC(a, b) = 32.19
b) MMC(a, b) = 24. 35.192.61.712
c) MMC(a, c) = 24. 32.192.712
d) MDC(a, b, c) = 19
e) MDC(b, c) = 2.19

Questes Resolvidas
01) Usando o algoritmo de Euclides, achar os inteiros x e y que verifiquem cada uma das seguintes
igualdades:
a) mdc(56, 72) = 56x + 72y
mdc(56, 72) = 8 8 = 56x + 72y
72 = 56.1 + 16
56 = 16.3 + 8
16 = 8.2 + 0
b) mdc(24, 138) = 24x + 138y
138 = 24.5 + 18
24 = 18.1 + 6
18 = 6.3 + 0 (mdc = 6)

Tomando a penltima igualdade;


8 = 56 16.3. Tirando o valor de 16 na primeira igualdade e
substituindo na penltima:
8 = 56 (72 56.1).3 8 = 56 + 56.3 72.3
8 = 56.4 + 72(-3). Portanto, x = 4 e y = -3.
6 = 24 18.1
6 = 24 (138 24.5).1
6 = 24 + 24.5 138.1
6 = 24.6 + 138(-1) x = 6 e y = -1

02) Achar inteiros x, y e z que verifiquem as seguintes igualdades:


1) 11x + 19y + 3z = 1
2) 56x + 6y + 32z = 2
3) 6x + 3y + 15z = 9
4) 14x + 7y + 21z = 4
Soluo:
01) Achando o mdc(11, 19)
1
1
2
19
11
8
3
8
3
2
1
Usando o algoritmo da diviso, podemos escrever:

1
2
0

2
1

19 = 11 x 1 + 8
11 = 8 x 1 + 3
8=3x2+2
3=2x1+1
2=1x2
Desprezando a ltima igualdade, eliminemos os restos, a partir da penltima igualdade:
1=32
1 = 3 (8 3 x 2)
1=3x38
1 = (11 8) x 3 8
1 = 11 x 3 8 x 4
1 = 11 x 3 (19 11) x 4
1 = 11 x 7 19 x 4

Achemos agora o mdc(3, 1). Como mdc(3, 1) = 1, vamos escrever este mdc em funo de 3 e 1:
1 = 3 x 1 + 1 x (-2). Agora substituamos o valor de 1, dado na igualdade acima, nesta ltima
igualdade:
1 = 3 x 1 + (11 x 7 19 x 4) (2)
1 = 3 x 1 + 11 (14) + 19 x 8 ou 1 = 11 (14) + 19(8) + 3(1). Logo x = 14, y = 8 e z = 1.
02) 56x + 6y + 32z = 2. Achando o mdc(56, 32)
56
24

1
32
8

1
24
0

3
8

Usando o algoritmo da diviso, podemos escrever:


56 = 32.1 + 24
32 = 24.1 + 8
24 = 8.3 Desprezando a ltima igualdade e eliminando os restos a partir da penltima:
8 = 32 24
8 = 32 (56 32)
8 = 32.2 + 56(1)
(1) Achemos o mdc(8, 6)
1
8
6
2
0
Usando o algoritmo da diviso, podemos escrever:

3
2

8=6+2
2 = 8 + 6(1) Agora, substituamos o valor de 8 na expresso (1)
2 = 32(2) + 56(1) + 6(1)
2 = 56(1) + 6(1) + 32(2) .Logo, x = 1, y = 1 e z = 2.
03) 6x + 3y + 15z = 9
Achando o mdc(15, 6)

15
3

2
6
0

2
3

Usando o algoritmo da diviso, podemos escrever:


15 = 6.2 + 3
3 = 15(1) + 6(2)
(1) Achemos o mdc(3, 3).Como o mdc(3, 3) = 3,vamos escrever 3 como combinao de 3 e 3: 3 =
3(2) + 3(1) Substituamos o valor de 3 encontrado na igualdade (1) nesta ltima igualdade:
3 = 3(2) + [15(1) + 6(2)](1)
3 = 3(2) + 15(1) + 6(2). Como escrever 9 como combinao linear de x, y e z, devemos multiplicar
por 3 esta ltima igualdade, obtendo:

9 = 3(6) + 15(3) + 6(6)


9 = 6(6) + 3(6) + 15(3). Logo, x = 6, y = 6 e z = 3
04) 14x + 7y + 21z = 4
Como o mdc(14, 7, 21) = 7 e 7 no divide 4, ento a equao no tem soluo inteira.
Calcular:
5) mmc (45, 21).
6) mmc (83, 68).
7) mmc (120, 110).
Soluo:
5) Como o mdc(45, 21) = 3, ento, 3.mmc(45, 21) = 45.21. Logo, mmc(45, 21) = 315.
6) Como mdc(83, 68) = 1, ento, 1.MMC(83, 68) = 8.8. Logo, mmc(83, 68) = 5644.
7) Como mdc(120, 110) = 10 , ento, 10.mmc(210, 110) 210.110. Logo, mmc(210, 110) = 1320
08) O mdc de dois inteiros positivos a e b 8 e na sua determinao pelo algoritmo de Euclides os
quocientes sucessivamente obtidos foram 2, 1, 1 e 4. Calcular a e b.
Resoluo: Temos o seguinte esquema:
2

4
8
Sabemos que se o mdc 8, o ltimo resto zero e o penltimo 8. Assim, temos:
2
1
1
4
8
8
0
Como 8 o divisor, 4 o quociente e zero o resto, achamos o dividendo desta diviso:
4 x 8 + 0 = 32. Logo o nmero anterior a oito 32. Deste modo 32 ser o outro resto
Temos o seguinte esquema:
2

1
4
32
8
32
8
0
Tendo 32 para divisor, 1 para quociente e 8 para resto, o prximo dividendo ser:
32 x 1 + 8 = 40. De modo semelhante, encontramos os outros nmeros:
184
40

2
72
32

Logo a = 184 e b = 72.


09) Usando o algoritmo de Euclides, determinar:
a) mdc(624, 504, 90).
Soluo:

1
40
8

1
32
0

4
8

Pelo processo anterior acha-se o mdc(624, 504) que 24. A seguir acha-se o mdc(24, 90) que 6.
R: 6.
Determinar os inteiros positivos a e b sabendo:
10) ab = 4032 e o mmc(a, b) = 336.
11) mdc(a, b) = 8 e o mmc(a, b) = 560.
Solues:
10) Como mmc(a, b) = 336, temos 336 = a k1 e 336 = b k2 .Multiplicando membro a membro estas
duas igualdades, temos: 336 x 336 = a b k1 k2. Substituindo o valor de a b = 4032 nesta ltima
igualdade, temos: 112896 = 4032 k1 k2 ou k1 k2 = 28. Assim, como k1 e k2 so primos entre si,
devemos procurar dois inteiros primos entre si, cujo produto 28. Encontramos k1 = 1 e k2 = 28, k1
= 4 e k2 = 7. Com estes valores temos a = 336 e b = 12 e a = 84 e b = 48.
11) Temos: mdc(a, b) mmc(a, b) = a b. Ento a b = 8 560. Temos, portanto um problema j
resolvido sobre mdc. A resposta ser: a = 8, b = 560; a = 16, b = 280; a = 40, b = 112; a = 56, b =
80.
12) Se a soma de dois nmeros 320 e o mnimo mltiplo comum entre eles 600, quais so esses
nmeros? Qual o mximo divisor comum entre eles?
Soluo: Se X e Y so os nmeros procurados, eles devem ser divisores de 600, logo devem
pertencer ao conjunto D(600):
R: {1, 2, 3, 4, 5, 6, 8, 10, 12, 15, 20, 24, 25, 30, 75, 100, 120, 150, 200, 300, 600}.
Pares de nmeros deste conjunto que somam 320, so: 300 e 20 ou 200 e 120. O primeiro par no
serve, pois MMC (300, 20) = 300. Os nmeros que servem so X = 200 e Y = 120, pois MMC (200,
120) = 600 e MDC (200, 120) = 40.
13) Se a diferena entre dois nmeros naturais 126 e o mximo divisor comum entre eles 18,
quais so esses nmeros?
Soluo: Se X e Y so os nmeros procurados, eles devem ser mltiplos de 18 e podem ser escritos
na forma X = 18a e Y = 18b onde a e b devem ser determinados. Assim: 18a - 18b = 126, de onde
segue que 18(a - b) = 187, o que equivalente a: a - b = 7. Tomando a = 8 e b = 1 teremos X = 144
e Y = 18.
14) Se a soma de dois nmeros naturais 420 e o mximo divisor comum entre eles 60, quais so
esses nmeros?
Soluo: Sejam X e Y os nmeros procurados. Se MDC(X, Y)=60, os nmeros X e Y devem ser
mltiplos de 60, logo podem ser escritos na forma X = 60a e Y = 60b onde a e b so nmeros

inteiros positivos. Assim: 60a + 60b = 420, o que garante que a + b = 7. Devemos escolher nmeros
naturais tal que a + b = 7, e assim, temos vrias opes.
Se a = 6 e b = 1 ento X =360 e Y = 60

Se a = 5 e b = 2 ento X = 300 e Y = 120

Se a = 4 e b = 3 ento X = 240 e Y = 180

Se a = 3 e b = 4 ento X = 180 e Y = 240

Se a = 2 e b = 5 ento X = 120 e Y = 300

Se a = 1 e b = 6 ento X = 60 e Y = 360

Questes Propostas
01) Usando o algoritmo de Euclides, determinar o mdc (306, 657).
02) Usando o algoritmo de Euclides, determinar:
a) mdc(285, 675, 405). R: 5.
b) mdc(209, 299, 102). R:- 1.
c) mdc(69, 398, 253). R: 23.
03) Usando o algoritmo de Euclides, achar inteiros x e y que verifiquem a seguinte igualdade:
mdc(56, 72) = 56x + 72y.
04) Achar inteiros x e y que verifiquem a seguinte igualdade:
a) 78x + 32y = 2

e) 238x + 51y = 3

b) 104x + 91y = 13

f) 52x + 13y = 1

c) 31x + 19y = 7

g) 145x + 58y = 87

d) 42x + 26y = 16

h) 17x + 5y = -2

05) Achar inteiros x, y e z que verifiquem a igualdade 198x + 288y + 512z = mdc(198, 288, 512).
R: x = -5, y = -217, z = 124.
06) Calcular as solues de todos os itens abaixo podendo ser obtidas a partir da propriedade
mdc(a,b).mmc(a, b) = a.b.
a) mmc(83, 68)

R: 5644

b) mmc( 120, 110)

R: 1320

c) mmc(86, 71)

R: 6106

d) mmc(224, 192)

R: 1344

e) mmc(1287, 507)

R: 16731

f) mmc(143, 227)

R: 32461

g) mmc(306, 657)

R: 22338

07) Determinar a e b se, a + b = 589 e

mmc(a , b)
mdc(a , b)

84 .

R: a = 57 e b = 532; a = 217 e b = 372.


08) Demonstrar que se a e b so inteiros positivos tais que o mdc(a, b) = mmc(a, b) ento a = b.
09) Sendo a e b inteiros positivos, demonstrar quo o mdc(a, b) sempre divide o mmc(a, b).

10) Quais os dois menores nmeros pelos quais devemos dividir 252 e 234 para que os quocientes
obtidos sejam iguais?
R: 7 e 9.
11) Quais os nmeros compreendidos entre 100 e 300 divisiveis ao mesmo tempo por 6, 9 e 15?
R: 180 e 270.
12) Quais os dois nmeros de trs algarismo divisiveis ao mesmo tempo por 8, 9 e 10?
R: 360 e 720.
13) Quais os dois menores nmeros pelos quais devemos multiplicar 30 e 54 para que os produtos
obtidos seja iguais?
R: 9 e 5.
14) Calcular o nmero que, dividido por 12, 40 e 60 deixa sempre o mesmo resto 5?
R: 125.
15) A editora do livro Matemtica recebeu pedidos de trs livrarias sendo que um pedido de 1300
livros, o segundo pedido de 1950 livros e o terceiro pedido de 3900 livros. A editora deseja remeter
em n pacotes iguais de tal forma que n seja o menor possvel. Calcule o valor de n.
R: 650 livros em cada pacote, num total de 11 pacotes.
16) Trs peas de tecido medem respectivamente, 180m, 252m e 324m. Pretende-se dividir em
retalhos de igual comprimento. Qual dever ser esse comprimento de modo que o nmero de
retalhos seja o menor possvel? Em quantos pedaos as peas sero dividas?
R: O comprimento de 36 m e o nmero de peas sero de 5, 7 e 9 pedaos.
17) Duas rodas dentadas se engrenam uma a outra, a primeira tem 48 dentes e demora 4 segundos
em cada volta, a segunda tem 104 dentes. Colocam-se em movimento e se pergunta ao cabo de
quanto tempo, se encontram na mesma posio inicial? R: 52 segundos.
18) Dois ciclistas correm sobre uma pista circular, partindo ao mesmo tempo de uma mesma linha.
O primeiro realiza uma volta completa, em 30 minutos e o segundo em 36 minutos. Quantas voltas
devero dar cada um, para que tornem a encontrar-se, sobre a linha de partida? R: 6 e 5.
19) Um remdio deve ser tomado diariamente em intervalos regulares. O fabricante quer que a
durao desses intervalos seja um nmero inteiro de horas (como 3 horas, por exemplo, e nunca trs
horas e meia). Alm disso, o fabricante quer que os horrios em que se deve tomar o remdio no
mudem de um dia para outro. Existem vrias possibilidades para a durao dos intervalos que
satisfazem essas exigncias do fabricante. Quais so elas?
R: D(24)

1, 2, 3, 4, 6, 8, 12, 24 .

20) Os planetas Jpiter, Saturno e Urano tm perodo de translao em torno do Sol de


aproximadamente 12, 30 e 84 anos, respectivamente. Quanto tempo decorrer, depois de uma
observao, para que eles voltem a ocupar simultaneamente as mesmas posies em que se
encontram no momento de observao? R: 420 anos.
21) Duas pessoas fazendo seus exerccios dirios partem de um mesmo ponto e contornam,
andando, uma pista oval que circula um jardim. Uma dessas pessoas andando de forma mais
acelerada d uma volta completa na pista em 12 min, enquanto a outra, andando mais devagar, leva
20 min para completar a volta. Depois de quantos minutos essas duas pessoas voltaro a se
encontrar no ponto de partida? R: 60 minutos ou 1 hora.
22) Em um certo pais as eleies para presidente ocorrem de 6 em 6 anos e para senador de 4 em 4
anos. Em 1992 essas eleies coincidiram. D os anos das quatro prximas vezes em que elas
voltaram a coincidir. R: 2004, 2016, 2028, 2040.
23) Jos daquelas pessoas que gostam de complicar as coisas. Quando lhe perguntam a sua idade,
ele responde Tenho mais de 40 anos, menos de 50 e minha idade um mltiplo de 3 e de 8. Qual
a idade do Jos?
R: 48 anos.
24) De uma rodoviria, parte um nibus da empresa X a cada 20 minutos e um da empresa Y a cada
45 minutos. Supondo que esses dois nibus partem juntos s 8 horas da manh, depois de quanto
tempo os nibus das duas empresas partiram juntos novamente?
R: 180 minutos ou 3 horas.
25) Numa estao rodoviria, os nibus para a cidade A partem de 6 em 6 horas, e para a cidade B,
de 8 em 8 horas. Numa ocasio, um nibus para a cidade A partiu junto com outro para cidade B.
Quanto tempo depois isso acontecer de novo?
R: 24 horas.
26) Da Praa da Repblica partem, s 6 horas da manh, dois bondes das linhas X e Y, iniciando o
servio do transporte de passageiros. Sabendo-se que o bonde X volta ao ponto de partida ao cabo
de 50 minutos, e o Y, ao cabo de 45 minutos, pergunta-se a que horas os dois bondes partiro
novamente juntos da praa da Repblica?
27) Tenho trs rguas divididas em partes iguais. Cada parte da primeira tem 3 mm, da segunda, 5
mm, e da terceira, 12 mm. Coloco as trs rguas uma do lado da outra, de modo que as suas
extremidades coincidam. Quais so os traos de diviso das trs rguas que coincidem?

Questes Propostas Envolvendo M.D.C e M.M.C


01) Determine s e t inteiros tais que MDC (a, b) = sa + tb para os seguintes pares de inteiros:
a) a = 145; b = 72

R: s = 1 e t = -2.

b) a = 896; b = 143

R: s = 64 e t = -401.

c) a = -123; b = 32

R: s = 13 e t = 50.

d) a = -75; b = -15

R: s = 0 e t = 1.

e) a = 102; b = 49

R: s = -12 e t = 25.

f) a = 138; b = 24

R: s = -1 e t = 6.

02) Classifique cada afirmao abaixo em Verdadeira ou Falsa, justificando:


a) MDC de dois nmeros naturais expressos por n e 2 + 1 sempre 1, para qualquer natural n. (V).
b) Considere a e b nmeros naturais. Ento MDC(a, ab + 1) = 2. (F).
c) MDC de dois nmeros naturais sempre um divisor do MMC destes mesmos nmeros. Se a e b
so relativamente primos, MMC(a, b) = |a.b|. (V).
03) Seja a

N:

Determine o mdc (a, a 1) .

R: d = 1.

Quais as possibilidades para o mdc (a, a 2) ?

R: d = (1, 2).

Quais as possibilidades para o mdc (a, a 6) ?

R: d = (1, 2, 3, 6).

Quais as possibilidades para o mdc (a, 3a 5) ?

R: d = (1, 5).

04) Determine todos os nmeros de trs algarismos divisveis por 8, 11 e 12, simultaneamente.
R: 264, 528 e 792.
05) Encontre todos os possveis pares de nmeros naturais cujo produto 3600 e cujo mmc 1200.
R: a = 3 e b = 1200 ou a = 48 e b = 75.
06) Determine dois nmeros cuja soma 120 e o mmc 144.
R: a = 12 e b = 108 ou a = 24 e b = 96.
07) Achar o menor nmero natural que satisfaa simultaneamente as condies:
a) quando dividido por 2 tem resto 1.

R: 3.

b) quando dividido por 3 tem resto 2.

R: 5.

c) quando dividido por 4 tem resto 3.

R: 7.

d) quando dividido por 5 tem resto 4.

R: 9.

e) quando dividido por 6 tem resto 5.

R: 11.

f) quando dividido por 7 tem resto 6.

R: 13.

g) quando dividido por 8 tem resto 7.

R: 15.

h) quando dividido por 9 tem resto 8.

R: 17.

08) Determine todos os possveis nmeros naturais n tais que:


a) mmc(n, 54) = 54

R: D(54)

1, 2, 3, 6, 9, 18, 27, 54

b) mmc(n, 26) = 26

R: D(26)

1, 2, 13, 26

09) O mmc dois nmeros naturais a e b igual a 1260 e quando dividimos este mmc pelos nmeros
a e b o produto dos quocientes obtidos igual a 90. Determine todos os nmeros naturais a e b
satisfazendo esta condio.
R: a = 1260 e b = 14, a = 630 e b = 28 e a = 252 e b = 70.
10) O mmc dois nmeros naturais 300. Dividimos este mmc por a e b, os quocientes obtidos so
tais que o seu produto vale 50. Determinem todos os pares de nmeros a e b que satisfazem estas
condies.
R: a = 300 e b = 6 e a = 150 e b = 12.
11) Prove que o produto de trs nmeros consecutivos divisvel por 6.
12) Se o resto da diviso de um nmero primo por 3 1, mostre que na diviso deste nmero por 6
o resto tambm 1.
13) Prove: se o resto da diviso de um nmero inteiro n por 6 5, ento o resto da diviso de n por 3
2.
14) Considere a e b nmeros naturais no primos entre si, cujo produto 420. Determine mdc(a, b).
R: 2.
15) Sejam m = 26.33.52, n = 2r.3s.5t e p = 25.54.73. Escreva as condies que devem satisfazer r, s e t
para que n seja divisor comum de m e p.
R: r = 5, s = 0 e t = 2.
16) Seja x

2 5 3 17 4 41 , y

3 4 17 6 312 e z

2 3 5 41 6 47 2 . Determine:

a) mdc (x,y) .

R: 17 4

b) mdc (x,z) .

R: 2.5

c) mdc (x,y,z) .

R: 2.41

d) mmc (x,y,z) .

R: 23.34.53.176.312.41.472

e) mmc (x,y) .

R: 2.34.53.176.312.41

f) mmc (x,z) .

R: 23.53.174.416.472

17) Encontre mdc (a,b) e mmc (a,b) , atravs da decomposio em fatores primos:
1) a = 20.600, b = 3.300.

R: mdc (a, b) = 22.52 e mmc (a, b) = 23.3.52.11.103.

2) a = 147.875, b = 166.725.

R: mdc (a, b) = 52.13 e mmc (a, b) = 53.7.132.19.

18) Encontre os valores de x para os quais mdc(20 + x, x) = 4.


R: os valores de x devero ser divisvel por 4.
19) Um professor d aulas numa 7 srie, de 30 alunos, e numa 8 srie, de 18 alunos. Em cada sala,
ele formou grupos, e de todos os grupos (tanto na 7 como na 8) tinham o mesmo nmero de
alunos. Qual o maior nmero de alunos que cada grupo pode ter?
R: Cada grupo pode ter no mximo 6 alunos.
20) Na minha escola, h 180 alunos na 5 srie, 168 na 6 srie, 144 na 7 srie e 120 na 8. Para
uma feira de cincias, todas esses alunos sero organizados em grupos com o mesmo nmero de
elementos, sem misturar alunos de srie diferentes.
a) Qual o nmero mximo de alunos que pode haver em cada grupo?
R: Pode ter no mximo 12 pessoas.
b) Quantos grupos sero formadas em cada uma das sries?
R: 15 grupos na 5 srie; 14 grupos na 6 srie; 12 grupos na 7 srie e 10 grupos na 8 srie.
21) Um pas tem eleies para presidente de 5 em 5 anos, e para governador de 4 em 4 anos. Em
1998, essas duas eleies coincidiram. D os anos das trs prximas vezes em que elas voltaro a
coincidir.
R: 2018, 2038 e 2058.
22) Um pas tem eleies para presidente de 4 em 4 anos, e para senador de 6 em 6 anos. Em 1997,
houve eleies para presidente, e em 1988 para senador. As eleies podero cair alguma vez no
mesmo ano? Explique sua resposta. R: no.
23) Muitos cometas nos visitam de tempos em tempos. Um certo cometa passa pela terra de 12 em
12 anos. Outro passa de 32 em 32 anos. Em 1913, os dois passaram por aqui. Qual a prxima
ocasio em que os dois passaro pela Terra no mesmo ano?
R: 2009.
24) Um cometa A passa pela terra de 26 em 26 anos. O cometa B passa de 32 em 32 anos. Ambos
visitaram a Terra em 1930. Pergunta-se:
a) Qual ser a prxima ocasio em que os dois visitaro a Terra no mesmo ano? R: 2346.
b) Depois de 1930, quantas sero as passagens do cometa a at que os dois visitem a Terra ao
mesmo ano? R: 16 passagens o cometa A e 13 passagens o cometa B.

25) Alguns cometam visitam a Terra periodicamente. Um cometa A visita terra de 12 em 12 anos. O
cometa B passa de 32 em 32 anos. Ambos visitaram a Terra em 1910. Qual a prxima ocasio em
que os dois passaro pela Terra no mesmo ano? R: 2006.
26) Uma rvore de Natal tem trs tipos de luzes. As vermelhas acendem a cada 8 segundos, as
verdes a cada 10 segundos e as amarelas a cada 12 segundos. Se elas acenderem todas juntas num
determinado momento, depois de quantos segundos ascendero juntas novamente? R: 120
segundos.
27) Numa competio, partiram juntos dois ciclistas. O primeiro leva 20 segundos para dar uma
volta completa na pista e o segundo leva 18 segundos. Eles estaro juntos novamente depois de
quantos segundos? R: 180 segundos.
28) Uma certa Irm recebe periodicamente a visita de seus trs filhos, Srgio a visita a cada 15 dias,
Marta a cada 20 dias e Rodrigo a cada 24 dias. Como hoje dia de seu aniversrio, os trs filhos
foram v-la. Daqui a quantos dias coincidira a visita dos trs filhos? R: 120 dias.
29) No terminal de nibus ABCD, chegam nibus da Vila Romana a cada 30 minutos e da Vila
Inglesa a cada 40 minutos. De quanto em quanto tempo os horrios dos nibus coincidem?
R: 120 minutos.
30) Uma avenida mede 4500 metros. A partir do inicio da avenida, a cada 250 metros h uma
parada de nibus, e a cada 225 metros uma para de bonde. Pergunta-se:
a) A que distncia do inicio da avenida ocorre a primeira coincidncia das paradas de nibus e de
bonde? R: 2250m.
b) Quantos so os pontos comuns de paradas de nibus e de bonde? R: 2 paradas comuns.
31) Durante um evento, o organizador pretende distribuir, como brindes, a alguns dos participantes,
caixas (kits), com o mesmo contedo, formado de camisetas e chaveiros. Sabe-se que ele possui
exatamente 200 camisetas e 120 chaveiros.
a) Decomponha os nmeros 200 e 120 em fatores primos? R: 200 = 23.52 e 120 = 2.3.5
b) Determine os nmeros mximos de caixas, com o mesmo contedo, que o organizador
conseguir formar utilizando todos os chaveiros e camisetas disponveis? R: 40.
32) (UnB) Quatro pessoas saem de uma praa a caminhar numa mesma hora. Elas repetiro vrias
vezes o mesmo percurso, e seus percursos duram respectivamente, 5 min, 9 min, 10 min e 15 min.
Aps quantos minutos elas estaro juntas na praa pela primeira vez? R: 90.
33) (UFRJ) Uma escola deseja distribuir cadernos entre os seus 480 alunos, de forma que cada um
deles receba o mesmo nmero de cadernos e no haja sobras. Os cadernos so adquiridos pela

escola em pacotes de uma dzia e meia cada. Determine o nmero de pacotes que a escola deve
adquirir para que cada aluno receba a menor quantidade possvel de cadernos. R: 80.
34) (UNICAMP) Trs lquidos diferentes, A, B e C, devem ser distribudos em barris iguais. H 108
litros do lquido A, 96 litros do B e 72 litros do C. Para que o nmero de barris seja o menor
possvel, qual deve ser a capacidade de cada barril? Quantos barris sero necessrios para conter
cada um dos lquidos? R: 12 litros, nmeros de barris: 9, 8 ou 6.
35) (PUC) Dois livros, um dos quais tem 256 pginas e o outro com 160 pginas, so formados por
fascculos com o mesmo nmero de pginas (superior a 10 e inferior a 50). Cada fascculo pode ter?
R: Pode ter 32 pginas.
36) (PUC) Um lojista dispe de trs peas de um mesmo tecido, cujos comprimentos so 48 m, 60
m e 80 m. Nas trs peas, o tecido tem a mesma largura. Ele deseja vender o tecido em retalhos
iguais, cada um tendo a largura das peas e o maior comprimento possvel, de modo a utilizar todo a
tecido das peas. Quantos retalhos ele dever obter? R: 47.
37) (UNESP) Trs cidades brasileiras, A, B e C, realizam grandes festas; de 5 em 5 meses em A, de
8 em 8 em B e 12 em 12 meses em C. Essas festas coincidiram em setembro de 1982. Coincidiro
novamente em? R: Setembro 1992.
38) (UFES) Trs vergalhes, medindo 400 cm, 480 cm e 720 cm, devem ser cortados em pedaos
iguais, de maior tamanho possvel, de modo que cada um deles tenha, por medida, um nmero
inteiro de centmetros. Desse modo, sero obtidos. Quantos pedaos? R: 20 pedaos.
39) (UFMG) As medidas tomadas sobre as divisas de um campo de formato triangular so 595 m,
459 m e 340 m. O proprietrio deseja plantar cajueiros nessas divisas, de tal modo que as distancia
entre eles sejam iguais e as maiores possveis. Se h um cajueiro em cada canto do campo, a
quantidade de cajueiros necessria ao plantio ? R: 82.
40) (UFRN) Para as festas natalinas, uma fbrica de doces lanar uma caixa de chocolates. O
nmero de chocolates poder ser dividido igualmente (sem fracion-las) entre 2, 3, 4, 5 e 6 pessoas,
no havendo sobra. O menor nmero de chocolates que essa caixa dever conter ser: R: 60.
41) (UFCE) Dois relgios tocam uma msica periodicamente, um deles a cada 60 segundos e o
outro a cada 62 segundos. Se ambos tocassem (simultaneamente) s 10 horas, que horas estaro
marcando os relgios quando voltaro a tocar juntos (simultaneamente) pela primeira vez aps as 10
horas? R: 10 horas e 31 minutos.

42) (UFMG) Numa repblica hipottica, o presidente deve permanecer 4 anos em seu cargo; os
senadores, 6 anos e os deputados, 3 anos. Nessa repblica, houve eleio para os trs cargos em
1989. A prxima eleio simultnea para esses trs cargos ocorrer, novamente, em: R: 2001.
43) (VUNESP) Uma concessionria vendeu no ms de outubro n carros do tipo A e m carros do
tipo B, totalizando 216 carros. Sabendo-se que o nmero de carros vendidos de cada tipo foi maior
do que 20, que foram vendidos menos carros do tipo A do que do tipo B, isto , n < m, e que
MDC(n, m) = 18, os valores de n e m so, respectivamente: R: 90, 126.
44) (UFMG) De uma praa partem, s 6 horas da manha, dois nibus A e B. Sabe-se que o nibus A
volta a ponto de partida a cada 50 minutos, e o nibus B, a cada 45 minutos. O primeiro horrio,
aps as 6 horas, em que os nibus partiro juntos : R: 13 horas e 30 minutos.
45) (U. E. Londrina - PR) Existem para doao a escolas, 2000 ingressos de um espetculo e 1575
de outro. Cada escola deve receber ingressos para somente um dos espetculos e todas as escolas
devem receber a mesma quantidade de ingressos. Distribuindo-se todos os ingressos, o nmero
mnimo de escolas que podem ser contemplados nessa doao : R: 143.
46) (U. E. Londrina - PR) Para levar os alunos de certa escola a um museu, pretende-se formar
grupos que tenham iguais quantidades de alunos e de modo que em cada grupo todos sejam do
mesmo sexo. Se nessa estudam 1350 rapazes e 1224 garotas e cada grupo dever ser acompanhado
de um nico professor, o nmero mnimo de professores necessrios para acompanhar todos os
grupos nessa visita : R: 143.
47) (UFMG) Entre algumas famlias de um bairro, foi distribudos um total de 144 cadernos, 192
lpis e 216 borrachas. Essa distribuio foi feita de modo que o maior nmero possvel de famlias
fosse contemplado e todas recebessem o mesmo nmero de cadernos, o mesmo nmero de lpis e o
mesmo nmero de borrachas, sem haver sobra de qualquer material. Nesse caso, o nmero de
cadernos que cada famlia ganhou foi de: R: 6.
48) (UE-RJ) Dois sinais luminosos fecham juntos num determinado instante. Um deles permanece
10 segundos fechado e 40 segundos aberto, enquanto o outro permanece 10 segundos fechado e 30
segundos abertos. O nmero mnimo de segundos necessrios, a partir daquele instante, para que os
dois sinais voltem a fechar juntos outra vez de: R: 200.
49) (UE-RJ) o nmero de fitas de vdeo que Marcela possui est compreendido entre 100 e 150.
Grupando-as de 12 em 12, de 15 em 15 ou de 20 em 20, sempre resta uma fita. A soma dos trs
algarismo do nmero total de fitas que ela possui igual a: R: 4.

50) (UFMG) Trs atletas correm numa pista circular e gastam, respectivamente, 2,4 min, 2,0 min e
1,6 min, para completar uma volta na pista. Eles partem do mesmo local e no mesmo instante. Aps
algum tempo, os trs atletas se encontram, pela primeira vez, no local da largada. Nesse momento, o
atleta mais veloz estar completando: R: 15 voltas.
51) (Cesgranrio-RJ) Certo botnico desenvolveu em laboratrio 3 variedades de uma mesma planta
V1, V2 e V3, que se desenvolvem cada uma a seu tempo, de acordo com a tabela abaixo. Plantandose as 3 variedades no mesmo dia, confiando-se na exatido, no ocorrendo nenhum fato que
modifique os critrios da experincia tabulada e levando-se em conta que, a cada dia de colheita,
outra semente da mesma variedade ser plantada, o nmero mnimo de sementes necessrio para
que a colheita das trs variedades ocorra simultaneamente ser:
Variedades

V1
V2
V3

Tempo de germinao
semanas aps o plantio)

(em

4
2
1

Tempo de florao (em semanas


aps a germinao)

Tempo para nica colheita (em


semanas aps a florao)

3
3
2

1
1
1

R: 24
52) Numa escola pretende-se distribuir, em partes iguais, 36 puzzles e 90 livros pelas bibliotecas
das varias turmas. Qual o nmero Maximo de turmas que a escola pode ter, para que essa
distribuio possa ser feita? Para esse numero Maximo, quantos puzzles e quantos livros recebero
cada biblioteca de turma? R:
53) Trs amigas, a Ana, a Patrcia e a Lena tiveram folga dos respectivos empregos no sbado
passado. Sabendo que a Ana tem folga a um sbado de 6 em 6 semanas, a Patrcia de 3 em 3
semanas e a Lena de 4 em 4 semanas, quantas semanas vo passar at que as trs amigas estejam de
folga, em simultneo, a um sbado? R:
54) Duas rodas gigantes comeam girar, num mesmo instante, com uma pessoa na posio mais
baixa em cada uma. A primeira d uma volta em 30 segundos e a segunda d uma volta em 35
segundos. As duas pessoas estaro ambas novamente na posio mais baixa aps: R:
55) Trs cidades A, B e C, realizam grandes festas: de 5 em 5 meses em A, de 8 em 8 meses em B e
de 12 em 12 meses em C. Essas festas coincidiram em setembro de 1982. Coincidiro novamente
em: R:

UNIDADE VI NMEROS PRIMOS


6.1 - Introduo:
No ano de 2002 trs matemticos indianos descobriram um algoritmo de primariedade, que
informa se um dado nmero primo ou no. Essa descoberta divulgada pela imprensa causou uma
preocupao mundial devido os cdigos criptogrficos que utilizam os nmeros primos.
J tinha lido sobre criptografia e sabia da sua importncia para a proteo das informaes, o
que despertou o meu interesse sobre o alcance desta descoberta, e na medida que a pesquisa se
desenvolvia outros movimentos em torno dos nmeros primos apareciam, envolvendo desde
matemticos e tcnicos de computao profissionais at usurios de computadores domsticos.
O nmero a entidade mais importante da Matemtica estando na origem de diversos ramos
desta cincia. Entre os seres vivos, o homem um dos poucos que possui senso numrico. Por isso,
desde os primrdios da raa humana os nmeros j estavam presentes, tendo surgido para auxiliar o
homem a controlar quantidades a partir do contraste entre pouco e muito, resultando na criao dos
primeiros sistemas de contagem. Juntamente com a linguagem, a escrita e outras habilidades, o
nmero est no conjunto das criaes humanas em que se baseou o desenvolvimento das nossas
sociedades. Nesta unidade, falaremos sobre nmeros, mas de um tipo especial os nmeros primos.
um assunto em que muitos especialistas em segurana eletrnica de dados tem conhecimento, e a
grande maioria das pessoas no sabem que a inviolabilidade dos seus dados pessoais depende em
parte destes nmeros. Os nmeros primos, um conhecimento sem aplicao desde as civilizaes
mais antigas, so a base dos cdigos de segurana de informao para computadores. Como estamos
vivendo, segundo alguns historiadores e socilogos na "Era da Informao" pode-se perceber sua
importncia para a nossa vida diria, embora no apaream de forma explcita. A propsito,
podemos citar a frase do matemtico Nicolai Lobachevsky (1793-1856) "No h ramo da
Matemtica, por abstrato que seja, que no possa um dia vir a ser aplicado nos fenmenos do
mundo real".
Os primos so apresentados pela primeira vez aos alunos na 5a srie e depois so quase
esquecidos. No nvel mdio, apesar do aluno estar mais amadurecido para a Matemtica, eles no
reaparecem, embora pudessem ajudar na fixao do contedo especfico, assim como devido ao
fascnio que exercem por conta das curiosidades e mistrios que os envolvem, despertar no aluno o
gosto por problemas da Teoria dos Nmeros. Cabe ressaltar, que os nmeros primos tem ganhado
importncia por causa das aplicaes na criptografia, deixando de ser uma mera curiosidade.
Desta forma, um papel de destaque est reservado para o conhecimento matemtico, j que
ele a "porta de entrada" para o mundo tecnolgico. Segundo Ubiratan D' Ambrosio (1996)" a

educao para a cidadania, que um dos grandes objetivos da educao de hoje, exige uma
apreciao do conhecimento moderno, impregnado de cincia e tecnologia".
Os nmeros primos so um exemplo para os alunos, de como podemos a partir de uma
definio antiga e relativamente simples, construir uma teoria que foi sendo enriquecida ao longo do
tempo de outros conhecimentos, culminando no seu aproveitamento em aplicaes tecnolgicas de
ltima gerao.
6.2 - Nmeros Primos:
Definio 6.1 - Dizemos que um nmero inteiro positivo p maior que 1 primo, se, e somente se, p
possui exatamente dois divisores positivos distintos, ou seja, 1, p .
Exemplo: O nmero 2 primo, pois os divisores positivos de 2 so

1, 2 . E mais, 2 o nico

nmero primo par, pois se existe primo par maior que 2, seria da forma N = 2q (q 1). Portanto, 1, 2
e q so divisores de N, o que torna absurdo, pois N primo. Um inteiro maior que 1 e que no
primo diz-se composto.
Teorema 6.1: Se um nmero primo p no divide um inteiro a, ento a e p so primos entre si.
Corolrio 6.1: Se p um primo tal que p | ab, ento p | a ou p | b.
Corolrio 6.2: Se p um primo tal que p | a1a2a3 ... an, ento existe um ndice k, com 1

n tal

que p | ak..
Corolrio 6.3: Se os inteiros p, q1,q2 ,..., qn so todos primos e se p | q1q2 ... qn, ento existe um
ndice k, com 1

n tal que p = qk..

Teorema 6.2: - Todo inteiro composto possui um divisor primo.


6.3 - Teorema Fundamental da Aritmtica:
Teorema 6.3 Todo inteiro positivo n > 1 igual a um produto de fatores primos.
Corolrio 6.4: A decomposio de um inteiro positivo n > 1 como produto de fatores primos
nica, a menos da ordem dos fatores.
Corolrio 6.5: Todo inteiro positivo b > 1 admite uma nica decomposio da forma n =

p1k1 p k2 2 ... p kr r onde, para i =1,2,..., r cada ki um inteiro positivo e cada pi um primo, com p1 < p2 <
... < pr, denominada decomposio cannica do inteiro positivo n > 1.
Exemplo 6.1: Definir mdc e mmc dos nmeros 588 e 936 pela decomposio cannica.
Teorema 6.4 - (de Euclides) - H um nmero infinito de primos.
Teorema 6.5 - Se um inteiro positivo a > 1 composto, ento a possui um divisor primo p
6.4 - Crivo de Eraststenes:

a.

A construo de uma tabela de primos que no um dado inteiro n faz-se usando o processo
conhecido pelo nome de crivo de Eraststenes, e que consiste no seguinte: escrevem-se na ordem
natural todos os inteiros desde 2 at n e, em seguida, eliminam-se todos os inteiros composto que
so mltiplos dos primos p tais que p

n , isto , 2p, 3p, 4p .

6.5 - Primo Gmeos:


Definio 6.2 Chama-se primos gmeos de dois inteiros positivos mpares e consecutivos que so
ambos primos.
Exemplos: 3 e 5; 5 e 7; 11 e 13; 17 e 19; 29 e 31.
No se sabe at hoje se h um nmero infinito de pares de primos gmeos muito grandes,
tais como:
140.737.488.353.507

140.737.488.353.509

140.737.488.353.699

140.737.488.353.701

Um fato interessante a existncia de apenas um terno de inteiros positivos mpares e


consecutivos que so todos primos: 3, 5 e 7.
6.4 - Seqncia de Inteiros Consecutivos Compostos:
Teorema 6.5 - Existem seqncias de n inteiros positivos consecutivos e compostos, qualquer que
seja o inteiro positivo n. A seqncia a seguinte: (n + 1)! + 2, (n + 1)! + 3, ... , (n + 1)! + (n + 1).
Os seus n termos so inteiros positivos consecutivos, e cada um deles composto, porque (n + 1)! +
J. divisvel por j se 2

j n 1.

Exemplo:
Suponha n = 4, obtemos a seqncia.
5! + 2,

5! + 3,

5! + 4,

5! + 5

Cujos termos so 4 inteiros positivos consecutivos, cada um dos quais composto, pois, temos:
5! + 2 = 122 = 2.61

5! + 3 = 123 = 3.41

5! + 4 = 124 = 4.31

5! + 5 = 125 = 5.25

Outra seqncias de 4 inteiros positivos consecutivos e composto existem, tais como:


24, 25, 26, 27

32, 33, 34, 35

54, 55, 56, 57

74, 75, 76, 77

Nota: Chama-se: Nmeros primos: primeiros ou indecomponveis e Nmeros no primos:


secundrios ou compostos.

Questes Resolvidos
01) Achar todos os primos que so iguais a um quadrado perfeito menos 1.
Soluo: Seja p este primo. Ento, p = n2 1 ou p = (n 1)(n + 1). Como p primo seus fatores s
podem ser 1 e p. Assim temos: n 1 = 1 e n + 1 = p. Da, conclumos que n = 2 e p = 3 que o
primo pedido.
02) Achar todos os primos que so iguais a um cubo perfeito menos 1.
Soluo: Seja p este nmero primo. Ento p = n3 1 ou p = (n 1)(n2 + n + 1). Como p primo
seus nicos fatores so 1 e p. Temos n 1 = 1 e n2 + n + 1 = p. Da tiramos n = 2 e p = 7.
03) Determinar todos os inteiros positivos tais que n, n + 2 e n + 4 so todos primos.
Soluo: que se n um inteiro qualquer, um dos inteiros n, n + 2 e n + 4 divisvel por 3. Sendo
assim s existe a possibilidade da seqncia ser 3, 5 e 7.
04) Determinar todos os primos p tais que 3p + 1 um quadrado perfeito.
Soluo: Temos 3p + 1 = n2 ou 3p = n2 1 ou 3p = (n 1)(n + 1). Como p primo a decomposio
do primeiro membro deve ser igual a do segundo. Assim 3 = n 1 e p = n + 1. Da, n = 4 e p = 5.
05) Achar uma seqncia de 100 inteiros positivos consecutivos e compostos.
Soluo: (100 + 1)! + 2, (100 + 1)! + 3, ... , (100 + 1)! + 101.
06) Mostrar que todo primo, exceto 2 e 3, da forma 6k 1 ou 6k + 1, onde k um inteiro positivo.
Soluo: Como o nmero primo no divisvel por 6, e esto excludos os primos 2 e 3, os restos
da diviso deste primo por 6, s podero ser 1 ou 5. Assim p = 6 q + 1 ou p = 6q + 5. A primeira
igualdade j est na forma exigida, fazendo q = k; e se p = 6q + 5, basta somarmos e subtrairmos um
ao segundo membro, obtendo: p = 6q + 5 + 1 1 ou p = 6q + 6 1 ou p = 6(q + 1) 1 o que d p =
6k 1, fazendo q + 1 = k.
07) Achar o menor inteiro positivo pelo qual se deve dividir 3720, para se obter um quadrado
perfeito.
Soluo: Achando a decomposio cannica de 3720 temos: 3720 = 23 x 3 x 5 x 31.
Para que um nmero seja um quadrado perfeito os expoentes de seus fatores primos, tm de ser par.
Ento o menor nmero que deve dividir o 3720 ser 2 x 3 x 5 x 31 = 930.
08) Achar todos os primos que so divisores de 50!
Soluo: So todos os primos menores que 50, pois 50! = 1 x 2 x 3 x ... x 49 x 50
Logo 2, 3, 5, 7, ... , 47.
09) Mostrar que todo inteiro da forma n4 + 4, com n > 1 composto.

Soluo: Vamos decompor o nmero em um produto de fatores maiores que um


n4 + 4 = n4 + 4 + 4n2 4n2 (somando e subtraindo 4n2 para formar um quadrado perfeito)
n4 + 4 = (n2 + 2)2 4n2 (fatoremos a diferena entre dois quadrados)
n4 + 4 = (n2 + 2 + 2n)(n2 + 2 2n). Sendo n > 1 os dois fatores so maiores que 1.
10) Mostrar que, se n > 4, composto, ento n divide (n 1)!.
Soluo: Como n composto, ele pode ser decomposto como produto de dois inteiros a e b: n = a b
, com 1 < a < n e 1 < b < n. Suponhamos que a b e consideremos a < b. Temos: 1 < a < b < n, ou 1
<a<b

n 1. Logo (n 1)! = 1.2.a.b. (n1) sendo a e b fatores de (n1)!. Deste modo ab = n

divide (n1)!. Se a = b, n = a2 e como n > 4, temos a2 > 4 ou a > 2 e a2 > 2 a ou 2 a < n = a2. Assim 2
a

n 1 e como a < 2 a, a e 2 a so fatores de (n1)! Logo: (n1)! = 1.2.3...a ...2 a ... (n1).

Portanto a2 um fator de (n1)!. E assim, a2 = n divide (n1)!


11) Demonstrar que o inteiro positivo a > 1 um quadrado perfeito se e somente se todos os
expoentes dos fatores primos da sua decomposio cannica so inteiros pares.
Soluo: Seja a um quadrado perfeito ento a tem a forma n2. Se n = n1k1.n2k2.n3k3. ... nnkn, onde
n1k1.n2k2.n3k3. ... nnkn a decomposio cannica em fatores primos de n, teremos n2 = (n1k1.n2k2.n3k3.
... nnkn)2 = n12k1.n22k2.n32k3. ... nn2kn. Como todos os expoentes so da forma 2k, conclui-se que todos
os expoentes so pares. Seja ento ni um fator primo de n cujo expoente no seja par. Neste caso, o
fator teria expoente da forma 2k + 1. Ora, ni2k + 1 = ni2k . ni. ni2k tem expoente para, portanto est de
acordo com o que foi dito anteriormente. Para que fosse quadrado, ni deveria ter dois fatores primos
iguais. Como ni primo isto no possvel. Portanto, todo nmero quadrado perfeito, se e
somente se, todos os expoentes dos fatores primos na decomposio cannica for par.
12) Demonstrar que, se o inteiro n composto, ento 2n 1 tambm composto.
Soluo: Evidentemente se trata de n positivo, pois se n < 0, n 1 negativo e 2n 1 no ser um
inteiro. Assim, para n > 0 e n composto, n > 3. Teremos ento n 1 > 2 2n 1 = 2k, k = n 1 > 2
inteiro: k 2 > 0 (k 1) > 0. Ora, 2n 1 = 2k = 2(2k 1) que mltiplo de 2 (no se esquea que k
1 > 0) 2n 1 composto.
13) Mostrar que so primos gmeos:
( a ) 1949 e 1951
Soluo: Primos gmeos so dois primos que so mpares consecutivos. 1949 e 1951 so dois
mpares consecutivos. Devemos verificar se ambos so primos. 1949 e 1951. Como ambos no so
divisveis por 2, 3, 5, 7, 11, 13, 17, 19, 23, 29, 31, 37, 41 e 43, os dois so primos. Portanto so
primos gmeos.

Questes Propostas
01) Achar os cinco menores primos da forma n2 2.
R: 2, 7, 23, 47 e 79.
02) Achar trs primos mpares cuja soma seja:
( a ) 81.
R: (3, 5, 73)
( b ) 125.
R: (5, 13, 107)
03) Achar todos os pares de primos p e q, tais que p q = 3.
R: p = 5 e q = 2.
04) Determinar se so primos os nmeros.
( a ) 169 R: composto.
( b ) 197 R: primo
( c ) 239 R: primo
( d ) 473 R: composto
05) Achar a decomposio cannica do inteiro 5040.
R: 24 . 32. 5 . 7
06) Achar o mdc(a, b) e mmc(a, b) sabendo a = 230 . 521 . 19 . 233 e b = 26 . 3 . 74 . 112 . 195. 237.
R: Mdc(a, b) = 26 . 19. 233 e (a, b) = 230.3.521.74.112.195.237.
07) Mostrar que so primos gmeos:
( a ) 1997 e 1999.
R: so primos gmeos.
08) Achar todos os primos gmeos entre 400 e 500.
R:- 491 e 421, 461 e 463.
09) Achar uma sequncia de quatro inteiros positivos consecutivos e compostos.
R: 5! + 2, 5! + 3, 5! + 4, 5! + 5
10) Verificar a conjectura de Goldbach para os seguintes inteiros pares:
( a ) 32
( b ) 100
( c ) 456
( d ) 1024

11) Verificar que todo par entre 4 e 100 a soma de dois primos.
12) Achar o menor inteiro positivo n tal que 2n2 + 29 um inteiro composto.
R: 1
13) Mostar que a soma de inteiros positivos mpares e consecutivos sempre um inteiro composto.
14) Usando a decomposio cannica dos inteiros 507 e 1287, achar o mdc(507, 1287) e o
mmc(507, 1287).
R: mdc = 39 e mmc = 16731.
15) Mostrar que o nico primo da forma n3 1 7.
R: o nico n3 1 primo 7.
16) Mostrar que todo inteiro da forma 8n + 1, com n > 1, composto.
17) Mostrar que se n2 + 2 primo ento 3 | n.
18) Mostrar, mediante um exemplo, que a seguinte conjectura falsa:
Todo inteiro positivo pode-se escrever sob a forma a2 + p, onde o inteiro a > 0 e p um inteiro
primo ou 1.
19) Demonstrar as seguintes propriedades:
( a ) Todo primo da forma 3n + 1 tambm da forma 6m + 1.
( b ) Todo inteiro da forma 3n + 2 tem um fator primo desta forma.
( c ) Se p > 5 um primo, ento p2 + 2 composto.
( d ) Se p um primo e se p | an, ento pn | an.
( e ) Todo inteiro n > 11 pode ser expresso como a soma de dois inteiros compostos.
( f ) Se p > q > 5 e se p e q so ambos primos, ento 24 | p2 q2.
( g ) Se p

5 um primo mpar, ento p2 1 ou p2 + 1 divisvel por 10.

20) Verificar que todo inteiro pode escrever-se sob a forma 2km, onde o inteiro k > 0 e m um
inteiro mpar.
21) Demonstrar que, se o inteiro n > 2, ento existe um primo p tal que n < p < n!.
22) Demonstrar que todo primo mpar da forma 4k + 1 ou 4k 1, onde k um inteiro positivo.

UNIDADE VII EQUAES DIOFANTINAS


7.1 - Introduo:
Diofante foi um grande matemtico que dedicou - se resoluo de problemas. Sua mais
importante obra foi a Aritmtica, uma coleo de 13 livros nos quais o autor reuniu cerca de 150
problemas resolvidos atravs de operaes numricas, nas quais demonstra seu alto grau de
habilidade e engenho. Tambm chamado de pai da lgebra, devido a sua contribuio na
introduo de notaes algbricas, Diofante utilizou abreviaes para a subtrao, a igualdade e a
incgnita.
Epitfio de Diofante:
Bastante curioso o epitfio de Diofante, matemtico grego da Antiguidade, que viveu 200
anos a.C. Encontramos na Antologia Grega um problema que apresentado sob a forma de epitfio:
Eis o tmulo que encerra Diofante, maravilha de contemplar. Com um artifcio aritmtico a
pedra ensina a sua idade. Deus concedeu-lhe passar a sexta parte de sua vida na juventude; um
duodcimo na adolescncia; um stimo em seguida, foi passado num casamento estril. Decorreram
mais cinco anos, depois do que lhe nasceu um filho. Mas este filho desgraado e, no entanto, bem
amado! apenas tinha atingido a metade da idade que viveu seu pai, morreu. Quatro anos ainda,
mitigando sua prpria dor com o estudo da cincia dos nmeros, passou-os Diofante, antes de
chegar ao termo de sua existncia. Em linguagem algbrica o epigrama da Antologia seria traduzido
pela equao:

x
x
x
x
+
+
+5+
+ 4 x , na qual x representa o nmero de anos que viveu
6 12
7
2

Diofante.
Resolvendo essa equao, achamos x = 84. Trata-se, afinal, de uma equao muito simples
do 1 grau com uma incgnita vamos verificar alguns exemplos:
Considere o seguinte problema (1): Se um trabalhador recebe 510 reais em tquetes de
alimentao, com valores de 20 reais ou 50 reais cada tquetes, de quantas formais pode ser formado
o carn de tquetes desse trabalhador.
Soluo:
Se x denota a quantidade de tquetes de 20 reais e y denota a quantidade de tquete de 50
reais ento a equao 20x 50y 510 , deve ser satisfeita e o problema resolvido determinando-se
todas as solues todas as solues inteiras no negativas desta equao.
Considere o seguinte problema (2): Se o custo de postagem de uma encomenda e de 85
centavos e devemos usar selos de 6 centavos e de 15 centavos, como devemos combinar os selos na
postagem.
Soluo:

Se x denota a quantidade de selos de 6 centavos e se y denota a quantidade de 15 centavos


ento a equao 6x 15y 85 , deve ser satisfeita e o problema resolvido determinando-se todas
as solues todas as solues inteiras no negativas desta equao. Desta forma passamos estudar
nesta unidade a resoluo das equaes diofantinas e suas aplicaes.
7.2 - Equaes Diofantinas:
Definio 7.1: Equao diofantina linear, uma equao da forma ax + by = c em que a, b, c, so
nmeros inteiros. Uma soluo de uma equao diofantina linear um par de inteiros x 0, y0 que
satisfaz a equao.
Exemplos 7.1: 3x + 6y = 18 solues 4 e 1, 6 e 6, 10 e 2 etc...
2x + 4y = 7 no tem soluo, pois o primeiro membro ser sempre par e o segundo membro mpar.
7.3 - Condio de Existncia de Soluo:
Teorema 7.1 - A equao diofantina linear ax + by = c tem soluo se, e somente se, d = mdc (a, b)
divide c.
7.4 - Solues da Equao ax + by = c:
Teorema 7.2 - Se d = mdc(a, b) divide c (d | c), e se o par de inteiros x0, y0 uma soluo particular
da equao diofantina linear ax + by = c, ento todas as outras solues desta equao so dadas
pela frmula: x = x0 +

b
d

t e y = y0

a
d

t onde t um inteiro arbitrrio.

Corolrio 7.1 - Se o mdc(a, b) = 1 e se (x0, y0) uma soluo particular da equao diofantina ax +
by = c, ento todas as outras solues desta equao so dadas pelas frmulas: x = x0 + bt e y = y0
at onde t um inteiro arbitrrio.

Questes Resolvidas
Determinar todas as solues inteiras das seguintes equaes diofantinas lineares:
01) 56x + 72y = 40
Soluo: Determinemos o mdc (72, 56).

Como o mdc (56, 72) = 8 divide 40, a equao possui soluo. Usando o algoritmo da diviso,
temos:
72 = 56 x 1 + 16
56 = 16 x 3 + 8
16 = 8 x 2
Vamos escrever o mdc 8 como combinao linear de 56 e 72.
8 = 56 16 x 3 = 56 (72 56) x 3 = 56 x 4 72 x 3 = 56(4) + 72 (3)
Como queremos resolver a equao 56x + 72y = 40 multipliquemos a ltima igualdade acima por 5
40 = 56(20) + 72(15).Logo xo = 20 e yo = 15, dando a soluo geral:
x = 20 + (72/8)t e y = 15 (56/8)t ou x = 20 + 9t e y = 15 7t.
02) 24x + 138y = 18
Soluo: Determinemos o mdc (138, 24).

Usando o algoritmo da diviso, temos:


138 = 24 x 5 + 18
24 = 18 x 1 + 6
18 = 6 x 3
Vamos escrever o mdc 6, como combinao linear de 24 e 138
6 = 24 18 = 24 (138 24 x 5) = 24 x 6 138 = 24(6) + 138(1)
Como queremos resolver a equao 24x + 138y = 18, multipliquemos a igualdade acima por 3
18 = 24(18) + 138(3): Logo xo = 18 e yo = 3, dando a soluo geral: x = 18 + (138/6)t e y = 3
(24/6)t ou x = 18 + 23t e y = 3 4t.
3) 221x + 91y = 117

Soluo: Determinemos o mdc (221, 91).

Como o mdc (221, 91) = 13 divide 117, a equao possui soluo.


Usando o algoritmo da diviso, temos:
221 = 91 x 2 + 39
91 = 39 x 2 + 13
39 = 13 x 3
Vamos escrever o mdc 13 como combinao linear de 221 e 91
13 = 91 39 x 2 = 91 (221 91 x 2) x 2 = 91 x 5 221 x 2 = 221(2) + 91(5)
Como queremos resolver a equao 221x + 91y = 117, multipliquemos a igualdade acima por 9.
117 = 221(18) + 91(45). Logo xo = 18 e yo = 45 dando a soluo geral:
x = 18 + (91/13)t e y = 45 (221/13)t ou x = 18 + 7t e y = 45 17t.
04) 84x 438y = 156
Soluo: Determinemos o mdc (438, 84).

Como o mdc (438, 84) = 6 divide 156, a equao possui soluo.


Usando o algoritmo da diviso, temos:
438 = 84 x 5 + 18
84 = 18 x 4 + 12
18 = 12 x 1 + 6
12 = 6 x 2
Vamos escrever o mdc 6 como combinao linear de 84 e 438
6 = 18 12 = 18 (84 18 x 4) = 18 x 5 84 = (438 84 x 5) x 5 84 = 438 x 5 84 x 26 ou 6 =
84(26) 438(5)
Como queremos resolver a equao 84x 438y = 156 multipliquemos a igualdade acima por 26.
156 = 84(676) - 438(130). Logo xo = 676 e yo = 130, dando a soluo geral:
x = 676 (438/6)t e y = 130 (84/6)t ou x = 676 73t e y = 130 14t.
05) 57x 99y = 77
Soluo: Determinemos o mdc (57, 99).

Como o mdc (57, 99) = 3 no divide 77, a equao no possui soluo.


06) 11x + 30y = 31
Soluo: Determinemos o mdc (11, 30).

Como o mdc (11, 30) = 1 divide 31, a equao possui soluo.


Usando o algoritmo da diviso, temos:
30 = 11 x 2 + 8
11 = 8 x 1 + 3
8=3x2+2
3=2x1+1
2=1x2
Vamos escrever o mdc 1 como combinao linear de 11 e 30.
1 = 3 2 = 3 (8 3 x 2) = 3 x 3 8 = (11 8) x 3 8 =11 x 3 8 x 4 =
= 11 x 3 (30 11 x 2) x 4
1 = 11 x 11 30 x 4 = 11(11) + 30(4)
Como queremos resolver a equao 11x + 30y = 31 multipliquemos a igualdade acima por 31
31 = 11(341) + 30(124). Logo xo = 341 e yo = 124, dando a soluo geral:
x = 341 + 30t e y = 124 11t.
07) 27x 18y = 54
Soluo: Determinemos o mdc (27, 18).

Como o mdc.(27, 18) = 9 divide 54, a equao possui soluo. Usando o algoritmo da diviso,
temos:
27 = 18 x 1 + 9
18 = 9 x 2
Vamos escrever o mdc 9 como combinao linear de 27 e 18.

9 = 27 18 = 27(1) 18(1)
Como queremos resolver a equao 27x 18y = 54 multipliquemos a igualdade acima por 6
54 = 27(6) 18(6). Logo xo = 6 e yo = 6 dando a soluo geral:
x = 6 (18/9)t e y = 6 (27/9)t ou x = 6 2t e y = 6 3t.
08) 13x 7y = 21
Soluo: Determinemos o mdc (13, 7).

Como o mdc (13, 7) = 1 divide 21, a equao possui soluo. Usando o algoritmo da diviso, temos:
13 = 7 x 1 + 6
7=6x1+1
6=1x6
Vamos escrever o mdc 6 como combinao linear de 13 e -7.
1= 7 6 = 7 (13 7) = 7 x 2 13 = 13(1) 7(2)
Como queremos resolver a equao 13x 7y = 21 multipliquemos a igualdade acima por 21
21 = 13(21) 7(42). Logo xo = 21 e yo = 42, dando a soluo geral:
x = 21 7t e y = 42 13t
09) 44x + 66y = 11
Soluo: Determinemos o mdc (66, 44).

Como o mdc (44, 66) = 22 no divide 11, a equao no possui soluo.


10) 21x 12y = 72
Soluo: Determinemos o mdc (21, 12).

Como o mdc (21, 12) = 3 divide 72, a equao possui soluo. Usando o algoritmo da diviso,
temos:
21 = 12 x 1 + 9
12 = 9 x 1 + 3

9=3x3
Vamos escrever o mdc 3 como combinao linear de 21 e -12.
3 = 12 9 = 12 (21 12) = 12 x 2 21 = 21 (1) 12(2)
Como queremos resolver a equao 21x 12y = 72 multipliquemos a ltima igualdade acima por 24
72 = 21 (24) 12(48). Logo xo = 24 e yo = 48, dando a soluo geral:
x = 24 (12/3)t e y = 48 (21/3)t ou x = 24 4t e y = 48 7t.
02) Determinar todas as solues inteiras e positivas das seguintes equaes diofantinas lineares
01) 5x 11y = 29
Soluo: A equao geral desta equao : x = 58 11t e y = 29 5t
Como queremos as solues positivas, devemos ter: 58 11t > 0 e 29 5t > 0
Da 1a. desigualdade, tiramos: t < 5,27 ou t
ser t

6 e da 2a, t < 5,8 ou t

6. Sendo assim, a soluo

6, dando uma infinidade de solues positivas.

02) 32x + 55y = 771


Soluo: A soluo geral desta equao : x = 9252 + 55t e y = 5397 32t
Como queremos as solues positivas, devemos ter: 9252 + 55t > 0 e 5397 32t > 0
Da primeira desigualdade, tiramos: t > 168,21 ou t

169

Da segunda desigualdade, tiramos: t < 168,5 ou t

168. Ento, no h solues positivas.

03) 58x 87y = 290


Soluo: A soluo geral desta equao : x = 10 3t e y = 10 2t
Como queremos as solues positivas, devemos ter: 10 3t > 0 e 10 2t > 0
Da primeira desigualdade, tiramos: t < 3,33 ou t
Da segunda desigualdade, tiramos: t < 5 ou t

4
6. Logo todos os valores de t no intervalo t

6, satisfazem o problema.
04) 62x + 11y = 788
Soluo: A soluo geral desta equao : x = 2364 + 11t e y = 13396 62t
Como queremos as solues positivas, devemos ter: 2364 + 11t > 0 e 13396 62t > 0
Da primeira desigualdade tiramos: t > 214,9 ou t
Da segunda desigualdade tiramos: t < 216,06 ou t

215
216. Logo os valores de t que satisfazem as

duas condies so: t = 215 e t = 216.


Quando t = 215, x = 1 e y = 66 e quando t = 216, x = 12 e y = 4.
05) 30x + 17y = 300
Soluo: A soluo geral desta equao : x = 1200 + 17t e y = 2100 30t
Como queremos as solues positivas, devemos ter: 1200 + 17t > 0 e 2100 30t > 0

Da primeira desigualdade tiramos: t > 70,58 ou t


Da segunda desigualdade tiramos: t < 70 ou t

70

71. Portanto no h solues positivas.

06) 54x + 21y = 906


Soluo: A soluo geral desta equao : x = 604 + 7t e y = 1510 18t
Como queremos as solues positivas, devemos ter: 604 + 7t > 0 e1510 18t > 0
Da primeira desigualdade tiramos: t > 86,28 ou t

86

Da segunda desigualdade tiramos: t < 83,88 ou t

84, o que nos d o intervalo 86,84 ou

t = 86, t = 85 e t = 84 dando os valores:


Para t = 86, x = 2 e y = 38
Para t = 85, x = 9 e y = 20
Para t = 84, x = 16 e y = 2
07) 123x + 360y = 99
Soluo: A soluo geral desta equao : x = 1353 + 120t e y = 462 41t
Como queremos as solues positivas, devemos ter: 1353 + 120t > 0 e 462 41t > 0
Da primeira desigualdade tiramos: t > 11,27 ou t

11

Da segunda desigualdade tiramos: t < 11, 26 ou t

-12. Portanto a equao no possui solues

positivas.
08) 158x 57y = 7
Soluo: A soluo geral desta equao : x = 154 57t e y = 427 158t
Como queremos as solues positivas, devemos ter: 154 57t > 0 e 427 158t > 0
Da primeira desigualdade tiramos: t < 2,7 ou t

Da segunda desigualdade tiramos: t < - 2,7 ou t

- 3. Ento todos os valores de t

- 3 satisfazem o

problema, dando infinitas solues positivas.


Questes Propostas
01) Determinar todas as solues inteiras e positivas das seguintes equaes diofantinas lineares:
01) 90x - 28y = 22

R: x = 13, y = 41.

02) 40x - 65y = 135

R: x = 5, y = 1.

03) 18x - 20y = -8

R: x = 4, y = 4.

04) 8x - 13y = 23

R: No existem solues positivas.

05) 3x + 4y = 20

R: x = 4, y = 2.

06) 50x - 56y = 74

R: x = 25, y = 21.

07) 8x - 13y = 23

R: x = 11, y = 5.

08) 5x - 2y = 2

R: x = 2, y = 4.

09) 24x + 138y = 18

R: No existem solues positivas.

10) 93x + 81y = 3

R: No existem solues positivas.

11) 43x + 128y = 1

R: No existem solues positivas.

12) 16x + 7y = 601

R: x = 3, y = 79 e x = 31, y = 15.

13) 47x + 29y = 1288

R: x = 20, y = 12.

14) 30x + 17y = 201

R: x = 5, y = 3.

15) 17x + 13y = 100

R: No existem solues positivas.

16) 12x + 18y = 50

R: No admite soluo.

17) 60x + 18y = 67

R: No admite soluo.

18) 1402x + 1969y = 1

R: No existem solues positivas.

19) 102x + 1001y = 533

R: No existem solues positivas.

20) 33x + 25y = 0

R: x = 0, y = 0.

21) 56x + 634y = 168

R: x = 0, y = 2.

22) 5x + 7y = 14

R:

25) 172x + 20y = 1000

R:

26) 18x + 5y = 48

R:

27) 39x + 26y = 105

R:

28) 14x + 22y = 50

R:

02) Encontre as solues das seguintes equaes diofantinas:


01) 2x - 10y + 35z = 0

R: x = 0, y = 0 e z = 0.

02) 2x + 3y + 5z = 7

R: x = 7, y = -14 e z = 7.

03) 1521x + 1955y + 455z = 221

R: x = 7956, y = -84.442 e z = 336.141.

04) 101x - 102y + 103z = 1

R: x = 1, y = -100 e z = -100.

05) 12x + 21y + 9z + 15w = 9

R: x = 2, y = -1, z = -1 e w = 1.

03) Determinar o menor inteiro positivo que dividido por 8 e por 15 deixa os restos 6 e 13,
respectivamente. R: 188.
04) Exprimir 100 como soma de dois inteiros positivos de modo que o primeiro seja divisvel por 7
e o segundo seja divisvel por 11. R: 56 e 44.
05) Determinar as duas menores fraes positivas que tenham 13 e 17 para denominadores e cuja
soma seja igual a 305|221.

R: 8|13 e 13|17.

06) Determine duas fraes cujos denominadores sejam 12 e 16 e cuja soma seja 10|48.
R: 1|12 e 2|16.

07) Demonstrar que se a e b so inteiros positivos primos entre si, ento a equao diofantina ax
by = c, tm um nmero infinito de solues inteiras e positivas.
08) Um grupo de pessoas gastou 1000 dlares num hotel. Sabendo-se que apenas alguns dos
homens estavam acompanhados pelas esposas e que cada homem gastou 19 dlares e cada mulher
gastou 13 dlares, pede-se determinar quantas mulheres e quantos homens estavam no hotel?
R: 41 homens e 17 mulheres.
09) Ache o inteiro estritamente positivo com a seguinte propriedade da resto 6 quando dividido por
11 e resto 3 quando dividido por 7?. R: 17.
10) Determine todos os mltiplos de 11 e de 9 cuja a soma igual a:
a) 79. R: no existe.

b) 80. R: 36 e 44.

c) 270. R: 99 e 171; 72 e 198.

11) Determine o menor inteiro positivo que tem restos 11 e 35 quando dividido respectivamente por
37 e 48. R: 899.
12) O valor da entrada de um cinema e R$ 8,00 e da meia entrada R$ 5,00. Qual o menor nmero
de pessoas que pode assistir a uma sesso de maneira que a arrecadao da bilheteria seja R$
500,00.
R: Este problema no tem uma nica soluo. As solues possveis so: x = 5 e y = 92 ou x = 60 e
y = 4.
13) Ao entrar num bosque alguns viajantes avistaram 37 montes de maa. Aps serem retiradas 17
frutas, o restante foi dividido igualmente entre 79 pessoas. Quantas frutas coube a cada pessoa?
R: 4 para cada pessoa.
14) Dispondo de 100 reais, quais so as quantias que se podem gastar comprando selos de R$ 5,00 e
de R$ 7,00. R: x = 6 e y = 10.
15) Numa criao de coelhos e galinhas contaram-se 400 ps. Quantas so as galinhas e quantos so
os coelhos, sabendo que a diferena entre esses dois nmeros seja a menor possvel?
R: 99 coelhos e 2 galinhas.
16) Um grupo de pessoas gastou 690 dlares num hotel. Sabendo-se que apenas alguns dos homens
estavam acompanhados pelas esposas e que cada homem gastou 18 dlares e cada mulher gastou 15
dlares, pede-se determinar quantas mulheres e quantos homens estavam no hotel.
R: Este problema no tem uma nica soluo. As solues possveis so: 25 homens e 16 mulheres
ou 30 homens e 10 mulheres ou 35 homens e 4 mulheres.

17) Um teatro vende ingressos e cobra R$ 18,00 por adulto e R$ 7,50 por criana. Numa noite,
arrecada R$ 900,00. Quantos adultos e crianas assistiram ao espetculo, sabendo que eram mais
adultos do que criana?
R: Este problema no tem uma nica soluo. As solues possveis so: x = 0 e y = 50 ou x = 12 e
y = 45 ou x = 24 e y = 40.
18) Uma agncia de correios possui apenas selos de 14 centavos e de 21 centavos. Determine as
combinaes desses selos que podem ser feitas para postar cartas dos seguintes valores postais:
(a) R$ 3,50
R:

x
1
y
16

x
4
y
14

x
7
y
12

x
10
y
10

x
13
y
8

x
16
y
6

x
19
y
4

x
22
y
2

x
25
y
0

(b) R$ 4,00
R: No admite soluo
(c) R$ 7,77
R: Varias solues por ter o valor de entre 37 t 55 .
19) Com R$ 5,49 podemos comprar maas, a 18 centavos cada, e perras, a 33 centavos cada. Qual e
o numero mnimo de frutas que podem ser compradas?
R: 18 frutas.
20) Um estudante, viajando da Europa aos Estados Unidos, troca seus francos suos e francos
franceses por dlares. Ele recebe US$ 17,06 tendo recebido US$ 0,19 (19 cents') por cada francos
francs e US$ 0,59 por cada francos suos. Quanto de cada moeda ele possua?
R: Francos Francs 7,03 e Francos Suos 17,03.
21) De que modos e possvel combinar 50 moedas, misturando moedas de 1, de 10 e de 25
centavos, de modo a totalizar 3 reais?
R: x = 1, y = -3 e z = 7.
22) Temos duas balanas: uma que marca pesos mltiplos de 10 e outra que marca pesos mltiplos
de 13. Como que com essas balanas podemos pesar 107 gramas?
R: x = 12, y = 9.
23) Apenas com a utilizao de dois relgios que s do intervalos de tempo de 5 e de 11 minutos
como podemos cozer um ovo durante 3 minutos?
R: No existem solues positivas.

24) Numa papelaria vendem-se dois tipos de canetas por 110 e 70 reais respectivamente. Ao fim de
um dia a importncia total recebida pela venda dessas canetas foi 6570 reais. Qual o menor
numero possvel de canetas vendidas? E qual o maior?
R: Qual o menor numero possvel de canetas 63. E qual o maior 91.
25) Subindo uma escada de dois em dois degraus, sobra um degrau. Subindo a mesma escada de trs
em trs degraus, sobram dois degraus. Determine quantos degraus possui a escada, sabendo que o
seu nmero e mltiplo de 7 e esta compreendido entre 30 e 100.
R: 35
26) Se um trabalhador recebe 510 reais em tquetes de alimentao, com valores de 20 reais ou 50
reais cada tquetes, de quantas formais pode ser formado o carn de tquetes desse trabalhador.
R: carn com 3 tquetes de R$ 20,00 reais e 9 tquetes de R$ 50,00, com 8 tquetes de R$ 20,00 reais
e 7 tquetes de R$ 50,00, com 13 tquetes de R$ 20,00 reais e 5 tquetes de R$ 50,00, com 18
tquetes de R$ 20,00 reais e 3 tquetes de R$ 50,00, com 23 tquetes de R$ 20,00 reais e 1 tquetes
de R$ 50,00.

UNIDADE VIII CONGRUNCIAS


8.1 - Introduo:
O conceito de congruncia, bem como a notao atravs da qual essa noo tornou um dos
instrumentos mais poderosos da teoria dos nmeros, foi introduzido por Karl Friedrich Gauss (1777
1855), em sua obra Disquisitions arithmeticae em 1801.
Para dar uma idia da noo de congruncia consideremos a seguinte questo, talvez ingnua
mas ilustrativa: Se hoje sexta-feira, que dia da semana ser daqui a 1520 dias?
Para organizar o raciocnio, indiquemos por o dia de hoje (sexta - feira), e por 1 o dia de
amanha (sbado), e assim por diante. A partir dessa escolha, podemos construir a seguinte tabela:
Sexta
0
7

Sbado
1
8

Domingo
2
9

Segunda
3
10

Tera
4
11

Quarta
5
12

Quinta
6
13

Nossa questo agora se resume em saber que coluna se encontra o nmero 1520. Para isso
basta observar que dois nmeros da seqncia 0, 1, 2, , esto na mesma coluna se, e somente
se, sua diferena divisvel por 7. Suponhamos que o nmero 1520 se encontre na coluna
encabeada pelo nmero a (0 a

6) , logo 1520 - a = 7q.

Para algum inteiro positivo q. donde obtemos 1520 = 7q + a, com (0 a

6) , Ora, pela

unicidade do resto na diviso euclidiana, segue dessa igualdade que o resto da diviso de 1520 por
7. portanto 1520 = 217 7 + 1, desta forma conclui-se que o resto 1 e que portanto 1520 esta na
segunda coluna. Logo, daqui a 1520 dias ser um sbado.
8.2 - Congruncias:
Definio 8.1 - Sejam a e b inteiros quaisquer e seja m um inteiro positivo fixo. Diz-se que a
congruente a b mdulo m se, e somente se, m divide a diferena a b. Em outros termos a
congruente a b mdulo m se, e somente se, existe um inteiro k tal que a b = km.
Notao: a

b (mod m) Simbolicamente: a

Exemplos 8.1 - 3

24 (mod 7); 31

b (mod m) se, e somente se, m | ( a b ).

11 (mod 6); 15

63 (mod 8).

Definio 8.2 - Se m no divide a diferena a b, ento, diz-se que a, incongruente a b mdulo m.


Notao: a

b (mod m).

Observaes:
1) Dois inteiros quaisquer so congruentes mdulo 1.
2) Dois inteiros so congruentes mdulo 2, se ambos so pares ou ambos mpares.
3) a

0 (mod m) se, e somente se, m | a.

Exemplos 8.2:

1) Mostrar que n 7 (mod 12) ento, n


2) Mostrar que n2

0 (mod 4) ou n2

3 (mod 4) n
1 (mod 4) n

Z.
Z.

8.3 - Caracterizao de Inteiros Congruentes:


Teorema 8.1 - Dois inteiros a e b so congruentes mdulo m se, e somente se, a e b deixam o
mesmo resto quando divididos por m.
8.4 - Propriedades das Congruncias:
Teorema 8.2- Seja m um inteiro positivo fixo (m > 0) e sejam a, b e c inteiros quaisquer. Subsistem
as propriedades:
1) a

a (mod m).

2) Se a

b (mod m), ento b

3) Se a

b (mod m) e se b

a (mod m).
c(mod m), ento a

c (mod m)

Teorema 8.3 - Seja m um inteiro positivo fixo (m > 0) e sejam a, b dois inteiros quaisquer.
Subsistem as seguintes propriedades:
1) Se a

b (mod m) e se n | m, com n > 0, ento a

b (mod n).

2) Se a

b (mod m) e se c > 0, ento ac

3) Se a

b (mod m) e se a, b, m so todos divisveis pelo inteiro d > 0, ento

bc (mod mc).

a
d

m
b
mod
.
d
d

Teorema 8.4 - Seja m um inteiro positivo fixo (m > 0) e sejam a, b, c, d inteiros quaisquer.
Subsistem as seguintes propriedades:
1) Se a

b (mod m) e se c

d (mod m), ento a + c

2) Se a

b (mod m), ento a + c

3) Se a

b (mod m), ento an

b + c (mod m) e ac

b + d (mod m) e ac

bd (mod m).

bc (mod m).

bn (mod m) para todo inteiro positivo n.

Exemplos 8.3 - Mostrar que:


a) Se a

b (mod m), ento a

b) Se a + b

c (mod m) ento a

Teorema 8.5 - Se ac

b (mod m).
c b (mod m).

bc (mod m) e se o mdc(c, m) = d, ento a

b mod

m
.
d

Corolrio 8.1 - Se ac

bc (mod m) e se o mdc (c, m) = 1, ento a

b (mod m).

Corolrio 8.2 - Se ac

bc (mod p), com p primo, e se p no divide c, ento a

b (mod p).

8.4 - Sistemas Completos de Restos:


Definio 8.3 - Chama-se sistema completo de restos mdulo m todo conjunto S = {r1, r2,..., rm} de
m inteiros tal que um inteiro qualquer a congruente mdulo m a um nico elemento de S.

Exemplos 8.3 - {1, 2, 3}, {0, 1, 2}, { 1, 0, 1}, {1, 5, 9} so sistemas completo de restos mdulo 3.
Teorema 8.6 - O conjunto S = {0, 1, 2, ..., n1 } um sistema completo de restos mdulo m.
Corolrio 8.3 - Se S = {r1, r2,..., rm} um sistema completo de restos mdulo m, ento os elementos
de S so congruentes mdulo m aos inteiros 0, 1, 2, ... , m 1, tomados numa certa ordem.
Questes Resolvidas
01) Achar o menor inteiro positivo que represente a soma:
( a ) 5 + 3 + 2 + 1 + 8 (mod. 7).
Soluo: 5 + 2

0 (mod.7), 3 + 1

Portanto 5 + 3 + 2 + 1 + 8

4 (mod. 7) 8

0+4+1

1 (mod. 7).

5 (mod. 7)

R: 5
( b ) 2 + 3 1 + 7 2 (mod.4).
Soluo: 2 + 3

1 (mod. 4) e -1 + 7 2 = 4

Portanto: 2 + 3 1 + 7 2

1+0

0 (mod. 4).

1 (mod. 4)

R. 1
02) Sabendo que 1066

1776 (mod m), achar todos os possveis valores do mdulo m.

Soluo: Como m | (1066 1776) devemos achar todos os divisores positivos de 710.
Logo os divisores positivos de 710 so: 1, 2, 5, 10, 71, 142, 355 e 710.
03) Exprimir que n mpar de trs outras maneiras.
Soluo:
01) n

1 (mod. 2)

02) n

- 1 (mod. 2)

03) n = 2k + 1
04) n = 2k 1
04) Mostrar que todo primo (exceto 2) congruente mdulo 4 a 1 ou 3.
Soluo: Se p primo, diferente de 2, ento ele mpar. Dividindo p por 4 obtemos os restos 1 ou
3. Assim p = 4q + 1 ou p = 4q + 3. Na primeira igualdade p 1 = 4q, ento p
segunda igualdade p 3 = 4q , dando p
05) Mostrar que 1110
Soluo: 112
1110 = 112. 118

3 (mod 4).

1 (mod 100).

21 (mod 100); 114


21 . 81

212

41 (mod 100); 118

412

81 (mod 100)

1 (mod 100).

06) Mostrar que todo primo (exceto 2 e 3) congruente mdulo 6 a 1 ou 5.

1 (mod 4) e na

Soluo: Se p um primo diferente de 2 e 3, ento ao dividirmos p por 6, obtemos os restos 1 ou 5.


Assim p

1 (mod 6) ou p

5 (mod 6).

07) Mostrar que 41 divide 220 1.


Soluo: Devemos mostrar que 220
26
220

23 (mod 41); 24
402

1 (mod 41).

16 (mod 41); 210= 26 . 24

23 . 16

40 (mod 41)

1 (mod 41).

08) Mostrar que 89 | (244 - 1).


Soluo: Devemos mostrar que 244
27

39 (mod 89); 24

244 = (211)4

1 (mod 89)

16 (mod 89); 211 = 27 . 24

39 . 16

1 (mod 89)

31 . 32

22 (mod 97)

1 (mod 89).

09) Mostrar que 97 | (248 1).


Soluo: Devemos mostrar que 248
27

31 (mod 97); 25

248 = (212)4
Logo 248

1 (mod 97).

32 (mod 97); 212 = 27 . 25

224 (mod 97); 222

-1 (mod 97); 224 = (222)2

96

(-1)2 (mod 97)

1 (mod 97).

10) Determinar quais dos seguintes conjuntos so sistemas completos de restos mdulo 4:
a) {2, 1, 0, 1}.
Soluo: 2

2; 1

3; 0

0; 1

0; 8

0; 12

1 (mod 4). Portanto um sistema completo de restos mdulo 4.

b) {0, 4, 8, 12}.
Soluo: 0

0; 4

0 (mod 4). Portanto no um sistema completo de restos

mdulo 4.
c) {13, 4, 17, 18}.
Soluo: 13

3; 4

0; 17

1; 18

2 (mod 4). Portanto um sistema completo de restos mdulo

4.
( d ) -5, 0, 6, 22 .
Soluo:
-5 = 4(-2) + 3

3, 0

0; 6 = 4.1 + 2 2; 22 = 4.5 + 2

O conjunto equivalente a {0, 2, 3}. Portanto, no um conjunto completo.


R: So conjuntos completos os das letras a e c.
Questes Propostas
01) Achar todos os inteiros x tais que 0

x 15 e 3x

6 (mod 15).

R: x = 2; x = 7 e x = 12.
02) Achar todos os inteiros x tais que 1 x 100 e x

7 (mod 17).

R:7, 24, 41, 58, 75 e 92.


03) Sabendo que k

1 (mod 4), mostrar que 6k + 5

3 (mod 4).

04) Achar os restos das divises de 250 e 4165 por 7.


R: 4 e 6.
Demonstrar as seguintes proposies:
05) Se a um inteiro mpar, ento a2

1 (mod 8).

06) Se a um inteiro qualquer, ento a3

0, 1 ou 8 (mod 9).

07) Se a um inteiro qualquer, ento a3

a (mod 6).

08) Mostrar, mediante um exemplo, que a2


09) Demonstrar que, se a

b2 (mod.m) no implica a

b (mod. m) ento mdc(a, m) = mdc(b, m).

10) Mostrar, mediante um exemplo, que ak

bk (mod. m) e k

11) Achar os restos das seguintes divises:


1) 245 por 7.

R: r = 1.

2) 11100 por 100.

R: r = 1.

3) 310.425+68 por 5.

R: r = 1.

4) 52.4841+285 por 3.

R: r = 0.

5) 710 por 51.

R: r = 19.

6) 2

100

por 11.

R: r = 1.

7) 734 por 51.

R: r = 49.

8) 14256 por 17.

R: r = 1.

9) 521 por 127.

R: r = 126.

10) 1212 por 5.

R: r = 3.

11) (116 + 1717)21 por 8.

R: r = 5.

12) 1316-225.515 por 3.

R: r = 0.

13) 23728 por 13.

R: r = 13.

14) 563 por 29.

R: r = 28.

12) Mostrar que 47 | (223 1) :


13) Para todo n

b (mod.m).

N, Mostrar que:

1) 1016n 1 divisvel por 70.

R: r = 1.

j no implica aj

bj.

2) 198n 1 divisvel por 17.

R: r = 1.

14) Achar os restos da diviso por 7 do nmero:


2

100

1) 1010 + 1010 + 1010 +........+ 1010 .

R: r = 4.

2) 16 + 26+.......+1006.

R: r = 2.

3) 17 + 27+.......+1007.

R: r = 3.

4) 22225555 + 55552222.

R: r = 0.

15) Achar os restos da diviso por 4 do nmero:


1) 1 + 2 + 22+........+219.

R: r = 3.

2) 15 + 25+.......+1005.

R: r = 0.

16) Determinar quais dos seguintes conjuntos so sistemas completos de restos mdulo 6.
a) {1, 2, 3, 4, 5}.
R: No , porque possui somente 5 elementos.
b) {0, 5, 10, 15, 20, 25}.
R: Portanto um sistema completo de restos mdulo 6.
c) {4, 3, 2, 1, 0, 1}.
R: Portanto um sistema completo de restos mdulo 6.
d) {17, 4, 6, 7, 10, 3}.
R: Portanto um sistema completo de restos mdulo 6.
17) Achar um sistema completo de restos {p1, p2, ... p7} mdulo 7, tal que todo pi primo.
R: r = {2, 3, 5, 7, 11, 13, 28}.
18) Achar um sistema completo de restos mdulo 7 formado s de mltiplos no negativos de 4.
R: r = { 36, 40, 44, 48, 52, 56, 60}.
19) Encontrar um sistema completo de restos mdulo 11 formado somente por mltiplo de 6.
R: r: 0, 1, 2, 3, 4, 5, 6, 7, 8, 9, 10 .
20) Encontrar um sistema completo de restos mdulo 7 onde todos os elementos so primos:
R: r: 0, 1, 2, 3, 4, 5, 6 .
21) Dado um primo p sempre possvel encontrar um sistema completo de restos mdulo p
formado s por primos? Justificar. Sua resposta. R:

UNIDADE XI CONGRUNCIAS LINEARES


9.1 - Introduo:
Usamos congruncias rotineiramente em nossa vida. Por exemplo, na contagem das horas,
fazemos uso de congruncia modulo 12 ou modulo 24. Na contagem de minutos e segundos,
usamos congruncia modulo 60. Nos calendrios, na contagem dos dias da semana fazemos uso, de
certa forma, da congruncia modulo, e na contagem dos meses, empregamos congruncia modulo
12. Os hodmetros dos carros, usados para marcar quilmetros rodados, geralmente trabalham
modulo 100.000. Veremos nesta unidade como resolve congruncias lineares e aplicando na
resoluo de equaes diofantinas.
9.2 - Congruncias Lineares:
Definio 9.1: Chama-se congruncia linear toda equao da forma ax

b (mod m), onde a e b so

inteiros quaisquer e m um inteiro positivo. Todo inteiro x0 tal que ax0

b (mod m) diz-se uma

soluo da congruncia linear.


Observao 9.1 - Se x0 uma soluo da congruncia linear ax

b (mod m), ento todos os

inteiros x0 + km, onde k um inteiro arbitrrio, tambm so solues da congruncia linear.


Observao 9.2 - Duas solues x0 e x1 da congruncia ax
, x0

b (mod m) congruente mdulo m isto

x1 (mod m) no so consideradas solues distintas. O nmero de solues da congruncia

dado pelo nmero de solues mutuamente incongruente mdulo m.


9.3 - Condio de Existncia de Solues:
Teorema 9.1 - A congruncia linear ax

b (mod m) tem soluo se, e somente se, d divide b, sendo

d = mdc(a, m). logo ax 0 b my0 ou ax 0 my0 = b .


9.4 - Solues da Congruncia ax

b (mod m):

Teorema 9.2 - Se d divide b, sendo d = mdc(a, m), ento a congruncia linear ax

b (mod m) tem

precisamente d solues mutuamente incongruentes mdulo m, dada pela frmula:


x = xo +

m
t, 0
d

d 1.

Corolrio 9.1 - Se o mdc(a, m) = 1, ento a congruncia linear ax

b (mod m) tem uma nica

soluo mdulo m.
9.5 - Resoluo de Equaes Diofantinas Lineares por Congruncias:
Uma equao diofantina linear, uma equao da forma ax + by = c em que a, b, c, so nmeros
inteiros. e de acordo com o Teorema 7.1 A equao diofantina linear ax + by = c tem soluo se, e

somente se, d = mdc (a, b) divide c. Uma soluo de uma equao diofantina linear um par de
inteiros x0, y0 que satisfaz a equao, ento:
ax0 + by0 = c
o que implica: ax0

ax0 - c = - by

b (mod m), Assim sendo, para obter uma soluo particular da equao

diofantina linear, basta determinar uma soluo qualquer x = x0 da congruncia linear ax

c (mod

b), e substituir este valor x0 de x na equao ax + by = c afim de encontrar o valor correspondente y0


de y, isto , tal que ax0 + by0 = c. Obviamente, tambm se pode obter uma soluo particular da
equao diofantina linear, determinando uma soluo qualquer y = y0 da congruncia linear. bx

(mod a).
Questes Resolvidas
01) Resolver as seguintes congruncias lineares:
a) 2x

1 (mod 17).

Soluo: O mdc(2, 17) = 1 e 1 | 1, logo a congruncia possui uma soluo.


2x 1 (mod 17); 1 2.9 (mod 17); 2x 2.9 (mod 17); x 9 (mod 17)
b) 3x 1 (mod 17).
Soluo: O mdc(3, 17) = 1 e 1 | 1, logo a congruncia possui uma soluo.
3x 1 (mod 17); 1 3.6 (mod 17); 3x 3.6 (mod 17); x 6 (mod 17)
c) 3x 6 (mod 18).
Soluo: O mdc(3, 18) = 3, e 3 | 6, logo a congruncia possui trs solues. Dividindo por 3 a
congruncia dada, obtemos: x

2 (mod 6) Assim a soluo geral x = 2 + 6t, t = 0, 1 e 2 dando x =

2, x = 8 e x = 14
d) 25x 15 (mod 29).
Soluo: O mdc(25, 29) = 1 e 1 | 15, logo a congruncia possui uma soluo
25x 15 (mod 29) dividindo por 5, temos: 5x 3 (mod 29); 3 5.18 (mod 29)
5x 5.18 (mod 29); x 18 (mod 29)
e) 5x 2 (mod 26).
Soluo: O mdc(5, 26) = 1, logo a congruncia possui uma soluo
5x 2 (mod 26); 2 5.16 (mod 26); 5x 5.16 (mod 26); x 16 (mod 26)
f) 6x 15 (mod 21).
Soluo: O mdc(6, 21) = 3 e 3 | 15, logo a congruncia possui trs solues
6x 15 (mod 21). Dividindo por 3, temos : 2x 5 (mod 7); 5 2.6 (mod 7)
2x 2.6 (mod 7); x 6 (mod 7). A soluo geral : x = 6 + 7t, t = 0, 1 e 2, dando:

x = 6, x= 13 e x = 20
g) 36x 8 (mod 102).
Soluo: O mdc(36, 102) = 6 e como 6 no divide 8, a congruncia no possui soluo.
02) Resolver por congruncia as seguintes equaes diofantinas lineares.
a) 12x + 25y = 331.
Soluo: O mdc(12, 25) = 1 e 1 | 331, logo a equao diofantina possui soluo
12x 331 = 25(-y) da, 12x 331 (mod 25); 331 12.13 (mod 25)
12x 12.13 (mod 25); x 13 (mod 25). Assim x0 = 13 uma soluo particular da equao
diofantina linear. Substituindo este valor na equao diofantina, obtemos:
12.13 + 25y = 331; 25y = 331 - 156; 25y = 175; y0 = 7
A soluo geral : x = 13 + 25t, y = 7 12t.
b) 5x 53y = 17.
Soluo: O mdc(5, 53) = 1 e 1 | 17, logo a equao diofantina possui soluo
5x 17 = 53y da, 5x 17 (mod 53); 17 5.14 (mod 53); 5x 5.14 (mod 53)
x 14 (mod 53). Assim x0 = 14 uma soluo particular da equao diofantina linear. Substituindo
este valor na equao diofantina, obtemos:
5.14 53y = 17; 53y = 17 - 70; 53y = 53; y0 = 1. A soluo geral : x = 14 53t, y = 1 5t.
c) 7x + 6y = 9.
Soluo: O mdc(7, 6) = 1 e 1 | 9, logo a equao diofantina possui soluo
7x - 9 = 6(y) da, 7x

9 (mod 6); 9 7.3 (mod 6); 7x 7.3 (mod 6)

x 3 (mod 6). Assim x0 = 3 uma soluo particular da equao diofantina linear. Substituindo este
valor na equao diofantina, obtemos: 7.3 + 6y = 9.
6y = 9 21; 6y = 12; y0 = 2. A soluo geral : x = 3 + 6t, y = 2 7t.
03) Determinar o nmero de solues de cada uma das seguintes congruncias lineares:
(a ) 3x

6 (mod. 15)

Mdc(3, 15) = 3.
Como 6 mltiplo de 3, a equao tem duas solues.
( b ) 4x

8 (mod. 15)

Mdc(4, 15) = 1. 8 mltiplo de 1. Portanto, a equao tem 1 soluo.


( c ) 5x

10 (mod. 15)

mdc(5, 15) = 5. Como 10 mltiplo de 5, a equao tem 2 solues.


( d ) 6x = 11 (mod. 15)

mdc(6, 15) = 3. Como 11 no mltiplo de 3, a equao no tem soluo.


Questes Propostas
01) Resolver as seguintes congruncias lineares:
1) 34x
2) 8x
3) 14x

60(mod 98)
16(mod 12)
36(mod 48)

R: x

45 e 94 (mod 98).

R: x

2, 5, 8, e 11 (mod. 12).

R: x

6 ou 30 (mod. 48).

4) 5x

3(mod 24)

R: x

4 (mod. 24).

5) 3x

5(mod 7)

R: x

4 (mod. 7).

R: x

16 (mod. 19).

R: x

13 (mod. 18).

8) 25x 15(mod120)

R: x

15, 39, 63, 87 e 111 (mod. 120).

9) 14x

36(mod 48)

R: x

30 e 61 (mod. 48).

10) 5x 15(mod12)

R: x

3 (mod. 12).

11) 3x

R: x

3, 11, e 19 (mod. 24).

6) 23x
7) 7x

7(mod19)

5(mod18)

9(mod 24)

02) Resolver por congruncia as seguintes equaes diofantinas lineares.


( a ) 4x + 51y = 9

R: x = 15 + 51t e y = -1 4t.

( b ) 7x + 6y = 9

R: x = 3 + 6t e y = - 2 7t

( c ) 11x + 27y = 4

R: x = 20 + 27t e y = -8 11t

( d ) 79x 131y = 6

R: x = 42 - 131t e y = 24 - 75t

( e ) 39x + 26y = 104

R: x = 2t e y = 37 + 61t

( f ) 61x 11y = 81

R: x = 8 - 11t e y = 37 - 61t

( g ) 65x + 77y = 200

R: x = 9 + 77t e y = -5 65t

( h ) 51x + 85y = 1037

R: x = 2 + 5t e y = 11 3t

03) Determinar o nmero de solues que pode ter uma congruncia linear cujo mdulo 20.
R: 1, 2, 4, 5, 10 ou 20 solues pois estes so os divisores de 20.
04) Demonstrar que se d = mdc (a, m) e se d | b, ento as congruncias lineares ax
(a|d)x

b (mod. m) e

(b|d) (mod. m|d) tm precisamente as mesmas solues.

05) Encontrar todas as solues de cada uma das seguintes congruncias lineares:
1) 5x

3(mod 7)

R: x = 15 + 51t e y = -1 4t

2) 13x 14(mod 29)

R:

3) 15x

R:

9(mod 25)

4) 37x 16(mod 19)

R:

5) 5x

R:

20(mod 15)

6) 3x 1(mod 25)

R:

7) 9x 1(mod 65)

R:

8) 6x 10 (mod 22)

R:

9) 14x 1(mod 77)

R:

10) 15x

9 (mod 12)

R:

11) 6x 10 (mod 22)

R:

12) 9x 12 (mod 21)

R:

UNIDADE X SISTEMAS CONGRUNCIAS LINEARES


10.1 - Introduo:
No sculo um, o matemtico chins chamado Sun-Tsu se perguntou? Que nmero ser a esse de
forma que quando dividido por 3, o resto 2; quando dividido por 5, o resto 3, e quando dividido
por 7, o resto 2?. A pergunta Qual a soluo para o seguinte sistema de congruncias?
x

2 (mod 3)

3 (mod 5)

2 (mod 7)

Definio 10.1 - Um sistema de congruncias lineares um sistema da forma abaixo:


A1x

B1 (mod m1 )

A2x

B2 (mod m 2 )

Ar x

Arx

Br (mod m r )

Br (mod mr) onde Ai, i = 1,2, ..., r so inteiros supostamente no nulos. Uma soluo

do sistema um inteiro x0 que soluo de cada uma das congruncias que dele fazem partes.
Exemplo 10.1: 3x

1 (mod 5) 2x

Teorema 10.1 - Um sistema x

3 (mod 9).

a1 (mod m1); x

a2 (mod m2) admite soluo se, e somente se, a1

a2 divisvel por d = mdc(m1, m2). Neste caso, se x0 uma soluo particular do sistema e se m =
mmc(m1, m2) ento x = x0 (mod m) sua soluo geral.
Corolrio 10.1 - Um sistema de congruncias lineares x

a1 (mod m1); x

a2 (mod m2); ... ; x

ar

(mod mr) admite soluo se, e somente se, ai aj divisvel por dij = mdc(mi, mj). Nesse caso se x0
uma soluo particular, ento a soluo geral do sistema dada por x

x0 (mod m) onde m =

mmc(m1, m2, ... , mr).


Exemplo 10.2: x

1 (mod 5); x

3 (mod 4); x

9 (mod 6).

Teorema 10.2 - (do Resto Chins): Sejam m1, m2,..., mr nmeros inteiros maiores que zero e tais
que mdc(mi, mj) = 1, i

j. Faamos m = m1m2...mr e sejam b1, b2, ... , br, respectivamente, solues

das congruncias lineares


;x

m
y
mj

1 (mod mj). Ento o sistema x

ar (mod mr) admite solues para quaisquer a1, a2, ... , ar

a1b1

M
m1

a 2 b2

M
m2

... a r br

a1 (mod m1); x

a2 (mod m2); ...

e sua soluo geral dada por:


M
(mod m).
mr

Este algoritmo, utilizado para resolver sistemas de congruncias lineares, muito antigo e foi
inventado, independentemente, pelos chineses e pelos gregos, para resolver problemas de

astronomia. O algoritmo chins do resto tem este nome porque um dos primeiros lugares em que
aparece foi no livro Manual de aritmtica do mestre Sun-Tsu, escrito entre 287 d.C. e 473 d.C.
Exemplo: x

1 (mod 2); x

2 (mod 3); x

3 (mod 5).

Questes Resolvidas
01) Resolver os seguintes sistemas de congruncias lineares:
a) x 1 (mod 2), x 1 (mod 3).
Soluo: Como o mdc(2, 3) = 1 o sistema possui soluo.
A soluo geral da 1a. x = 1 + 2a, substituindo este valor na 2a, obtemos
1 + 2 a 1 (mod 3); 2 a 0 (mod 3); a 0 (mod 3). Logo a = 3b; substituindo este valor em x = 1 +
2a, temos x = 1 + 2(3b), dando x = 1 + 6b que a soluo geral do sistema ou x 1(mod 6) .
b) x 5 (mod 12), x 7 (mod 19).
Soluo: Como mdc(12, 19) = 1 o sistema possui soluo. A soluo geral da 1a.
x = 5 + 12a; substituindo este valor na 2a, obtemos: 5 + 12 a 7 (mod 19); 12 a 2 (mod 19); 6
a 1 (mod 19); 1 6.16 (mod 19) 6; a 6.16 (mod 19); a 16 (mod 19). A soluo geral a = 16 +
19b; substituindo este valor em x = 5 + 12 a, temos x = 5 + 12(16 + 19b), dando x = 197 + 228b,
que a soluo do sistema ou x 197(mod 228) .
02) Resolver os seguintes sistemas de congruncias lineares:
a) x 3 (mod 5), x 5 (mod 7), x 7 (mod 11).
Soluo: Como mdc(5, 7) = mdc(5, 11) = mdc(7, 11) = 1 o sistema possui soluo.
A soluo geral da 1a. x = 3 + 5 a, substituindo este valor na 2a, obtemos
3 + 5 a 5 (mod 7); 5 a 2 (mod 7); 2 5.6 (mod 7); 5 a 5.6 (mod 7)
a 6 (mod 7) dando a soluo geral a = 6 + 7b; substituindo este valor em x = 3 + 5 a temos: x = 3 +
5(6 + 7b) = 33 + 35b. Substituindo este valor na 3a equao, temos:
33 + 35b 7 (mod 11); 35b 26 (mod 11); 35b 2b (mod 11); 2b 26 (mod 11).
b 13 (mod 11) dando a soluo b = 13 + 11c. Substituindo este valor na expresso
x = 33 + 35 b, temos x = 33 + 35(13 + 11c) = 422 + 385c que a soluo geral do sistema
x

348(mod 385) .

b) x 1 (mod 5), x 5 (mod 7), x 7 (mod 11).


Soluo: Como o mdc(5, 7) = mdc(5, 11) = mdc(7, 11) = 1 o sistema possui soluo.
A soluo geral da 1a congruncia : x = 1 + 5 a.; substituindo este valor na 2a., obtemos
1 + 5 a 5 (mod 7); 5 a 4 (mod 7); 4 5.5 (mod 7); 5 a 5.5 (mod 7)
a 5 (mod 7), dando a soluo geral a = 5 + 7b; substituindo este valor em x = 1 + 5 a, obtemos x =
1 + 5(5 + 7b) = 26 + 35b; substituindo este valor na 3a congruncia, temos:

26 + 35b 7 (mod 11); 35b 19 (mod 11); 19 35.7 (mod 11).


35b 35.7 (mod 11); b 7 (mod 11), dando a soluo geral b = 7 + 11c; substituindo este valor em
x = 26 + 35b, x = 26 + 35(7 + 11c) = 271 + 385c que a soluo geral do sistema ou
x

271(mod 385) .

c) x 5 (mod 6), x 4 (mod 11), x 3 (mod 17).


Soluo: Como o mdc(6, 11) = mdc(6, 17) = mdc(11, 17) = 1 o sistema possui soluo.
A soluo geral da 1a congruncia : x = 5 + 6 a, substituindo este valor na 2a congruncia,
obtemos: 5 + 6 a 4 (mod 11); 6 a 1 (mod 11); 1 6.9 (mod 11).
6 a 6.9 (mod 11); a 9 (mod 11), dando a soluo geral a = 9 + 11b; substituindo este valor em x
= 5 + 6 a, obtemos: x = 5 + 6(9 + 11b) = 59 + 66b; substituindo este valor na 3a congruncia, temos:
59 + 66b 3 (mod 17); 66b 56 (mod 17); dividindo por 2, temos: 33b 28 (mod 17);
28 33.11 (mod 17); 33b 33.11 (mod 17); b 11 (mod 17), dando a soluo geral b = 11 + 17c,
substituindo este valor em x = 59 + 66b, temos: x = 59 + 66(11 + 17c) = 785 + 1122c que a
soluo geral do sistema ou x

785(mod 1122) .

d) x 5 (mod 11), x 14 (mod 29), x 15 (mod 31).


Soluo: Como o mdc(11, 29) = mdc(11, 31) = mdc(29, 31) = 1 o sistema possui soluo.
A soluo geral da 1a congruncia : x = 5 + 11 a, substituindo este valor na 2a congruncia,
obtemos 5 + 11 a 14 (mod 29); 11 a 9 (mod 29); 9 11.14 (mod 29); 11 a 11.14 (mod 29);
a 14 (mod 29), dando a soluo geral a = 14 + 29b substituindo este valor em x = 5 + 11 a, temos:
x = 5 + 11(14 + 29b); x = 159 + 319b, substituindo este valor na 3a congruncia, temos: 159 +
319b 15 (mod 31); 319b 144 (mod 31); 144 319.15 (mod 31); 319b 319.15 (mod 31);
b 15 (mod 31), dando a soluo geral b = 15 + 31c; substituindo este valor em x = 159 + 319b,
temos: x = 159 + 319(15 + 31c); x = 4944 + 9889c que a soluo geral do sistema ou
x

4944(mod 9889) .

e) x 7 (mod 9), x 10 (mod 4), x 1 (mod 7).


Soluo: Como o mdc(9, 4) = mdc(9, 7) = mdc(4, 7) = 1 o sistema possui soluo.
A soluo geral da 1a congruncia : x = 7 + 9a, substituindo este valor na 2a congruncia, obtemos
7 + 9a 10 (mod 4); 9a 3 (mod 4); 3a 1 (mod 4); 1 3.3 (mod 4); 3 a

3.3 (mod 4); a 3 (mod

4), dando a soluo geral a = 3 + 4b; substituindo este valor em x =7 + 9 a, temos x = 7 + 9(3 + 4b)
= 34 + 36b; substituindo este valor na 3a congruncia, temos: 34 + 36b 1 (mod 7); 36b 33 (mod
7); 33 36.2 (mod 7); 36b 36.2 (mod 7); b 2 (mod 7), dando a soluo geral b = 2 + 7c,
substituindo este valor em x = 34 + 36b, temos x = 34 + 36(2 + 7c); x = 106 + 252c, que a soluo
geral do sistema ou x 106(mod 252) .

f) x 28 (mod 29), x 30 (mod 31), x 10 (mod 11).


Soluo: Como o mdc( 29, 31) = mdc(29, 11) = mdc(31, 11) = 1 o sistema possui soluo.
A soluo geral da 1a congruncia : x = 28 + 29 a, substituindo este valor na 2a congruncia,
obtemos: 28 + 29 a 30 (mod 31); 29 a 2 (mod 31); 2 29 (mod 31); 29 a 29 (mod 31); a 1
(mod 31), dando a soluo geral a = 1 + 31b; substituindo este valor em x = 28 + 29 a, temos: x =
28 + 29(1 + 31b); x = 1 + 899b; substituindo este valor na 3a congruncia, temos: 1 + 899b 10
(mod 11); 899b 11 (mod 11); 11 899.11 (mod 11); 899b 899.11 (mod 11); b 11 (mod 11);
dando a soluo geral b = 11 + 11c; substituindo este valor em x = 1 + 899b, temos x = 1 +
899(11 + 11c) = 9888 + 9889c, que a soluo geral do sistema ou x

-1(mod 9889) .

g) x a (mod 3), x b (mod 5), x c (mod 8).


Soluo: Como o mdc(3, 5) = mdc(3, 8) = mdc(5, 8) = 1 o sistema possui soluo.
Vamos resolver este sistema pelo mtodo do resto chins. Temos m1 = 3, m2 = 5 e m3 = 8, sendo m
= 3.5.8 = 120.
m/m1 = 5.8 = 40, m/m2 = 3.8 = 24, m/m3 = 3.5 = 15. Vamos resolver as seguintes congruncias
lineares: 40b1 1 (mod 3); 24b2 1 (mod 5); 15b3 1 (mod 8); 40b1 1 (mod 3), b1 = 1; 24b2 1
(mod 5); b2 = 1; 15b3 1 (mod 8); b3 = 1. Com a1 = a, a2 = b e a3 = c, que a soluo geral do
sistema x 40a 24b 15c (mod 120).
03) Resolver o seguinte sistema de congruncia:
1) 5x 11 (mod 17), 3x 19 (mod 32), 11x 6 (mod 37).
Soluo: Como o mdc(17, 32) = mdc(17, 37) = mdc(32, 37) = 1 o sistema possui soluo.
De 5x 11 (mod 17), x 9 (mod 17)
De 3x 19 (mod 32), x 17 (mod 32)
De 11x 6 (mod 37), x 14 (mod 37), assim temos o seguinte sistema:
x 9 (mod 17), x 17 (mod 32), x 14 (mod 37)
Usando o mtodo do resto chins: a1 = 9, a2 = 17, a3 = 14, m1 = 17, m2 = 32, m3 = 37 e m = m1m2m3
= 17.32.37 = 20128. m/m1 = 32.37 = 1184, m/m2 = 17.37 = 629, m/m3 = 17.32 = 544.
Ento 1184b1 1 (mod 17), 629b2 1 (mod 32) e 544b3 1 (mod 37). De onde conclumos: b1 = 14,
b2 = 29 e b3 = 10. A soluo geral ser: x 12.113 (mod 20.128).
2) 2x 1 (mod 5), 3x 9 (mod 6), 4x 1 (mod 7), 5x 9 (mod 11).
Soluo: Como o mdc(5, 6) = mdc(5, 7) = mdc(5, 11) = mdc(6, 7) = mdc(6, 11) = mdc(7, 11) = 1 o
sistema possui soluo.
De 2x 1 (mod 5), x 3 (mod 5).
De 3x 9 (mod 6), x 3 (mod 2).

De 4x 1 (mod 7), x 2 (mod 7).


De 5x 9 (mod 11), x 4 (mod 11).
Vamos resolver pelo mtodo do resto chins: a1 = 3, a2 = 3, a3 = 2, a4 = 4, m1= 5 , m2 = 2, m3 = 7,
m4 = 11.
m = m1 m2 m3 m4 = 5.2.7.11 = 770.
m
= 2.7.11 = 154, 154b1 1 (mod 5), b1 = - 1.
m1
m
= 5.7.11 = 385, 385b2 1 (mod 2), b2 = 1.
m2
m
= 5.2.11 = 110, 110b3 1 (mod 7), b3 = 3.
m3
m
= 5.2.7 = 70, 70b4 1 (mod 11), b4 = 3.
m4

A soluo : x 3.(1).154 + 3.1.385 + 2.3.110 + 4.3.70 (mod 770) ou x 653 (mod 770).
4) Resolver os seguintes sistemas de congruncias lineares:
1) x 8 (mod 9), x 2 (mod 3), x 5 (mod 7).
Soluo: Como o mdc(9, 3) = 3 e 3 | (8 - 2), mdc(9, 7) = mdc(3, 7) = 1 o sistema possui soluo.
A soluo geral da 1a congruncia : x = 8 + 9 a , substituindo este valor na 2a congruncia,
obtemos: 8 + 9 a 2 (mod 3); 9 a 6 (mod 3); 3 a - 2 (mod 1); a

1 (mod 7), dando a soluo

geral a = 1 + b; substituindo este valor em x = 8 + 9 a, temos: x = 8 + 9(1 + b) = 17 + 9b;


substituindo este valor na 3a congruncia, temos:
17 + 9b 5 (mod 7); 9b 12 (mod 7); 3b 4 (mod 7); 4 3 (mod 7); 3b 3 (mod 7); b 1
(mod 7); dando a soluo geral b = 1 + 7c; substituindo este valor em x = 17 + 9b, temos x = 17 +
9(1 + 7c) = 26 + 63c que a soluo geral do sistema.
2) x 4 (mod 6), x 13 (mod 15), x 8 (mod 14), x 1 (mod 7).
Soluo: Como o mdc(6, 15) = 3 e 3 | (13 - 4), mdc(6, 14) = 2 e 2 | (8 4), mdc(6, 7) = mdc(15, 14)
= mdc(15, 7) = 1 e mdc(14, 7) = 7 e 7 | (8 1) o sistema possui soluo.
A soluo geral da 1a congruncia : x = 4 + 6 a; substituindo este valor na 2a congruncia,
obtemos: 4 + 6 a 13 (mod 15); 6 a 9 (mod 15); 2 a 3 (mod 5); 3 8 (mod 5); 2 a 8 (mod 5);
a 4 (mod 5); dando a soluo geral a = 4 + 5b; substituindo este valor em x = 4 + 6 a, temos x = 4
+ 6(4 + 5b) = 28 + 30b; substituindo este valor na 3a congruncia, temos: 28 + 30b 8 (mod 14);
30b 20 (mod 14); 3b 2 (mod 7); 2 12 (mod 7); 3b 12 (mod 7); b 4 (mod 7); dando a
soluo geral b = 4 + 7c; substituindo este valor em x =28 + 30b, temos: x = 28 + 30(4 + 7c).

x = 148 + 210c; substituindo este valor na 4a congruncia, temos: 148 + 210c 1 (mod 7); 210c
147 (mod 7); 30c 21 (mod 1); 21 30 (mod 1); 30c 30 (mod 1); c 1 (mod 1); dando a
soluo geral c = 1 + d; substituindo este valor em x =148 + 210c; temos: x = 148 + 210(1 + d) =
358 + 210c que a soluo geral do sistema.
3) x 0 (mod 3), x 1 (mod 4), 17x 9 (mod 23)
Soluo: Como o mdc(3, 4) = mdc(3, 23) = mdc(4, 23) = 1 o sistema possui soluo.
A congruncia 17x 9 (mod 23) pode ser transformada em x 10 (mod 23)
A soluo geral da 1a congruncia : x = 3 a; substituindo este valor na 2a congruncia, obtemos 3
a 1 (mod 4); 1 9 (mod 4); 3 a 9 (mod 4); a 3 (mod 4); dando a soluo geral a = 3 + 4b;
substituindo este valor em x =3 a, temos; x = 3(3 + 4b); x = 9 + 12b; substituindo este valor na 3a
congruncia, temos: 9 + 12b 10 (mod 23); 12b 1 (mod 23); 1 12.2 (mod 23); 12b 12.2 (mod
23); b 2 (mod 23); dando a soluo geral b = 2 + 23c; substituindo este valor em x = 9 + 12b,
temos: x = 9 + 12(2 + 23c) = 33 + 276c que a soluo geral do sistema.

Questes Propostas
01) Resolver os seguintes sistemas de congruncias lineares:
3x 1(mod 7)
x 1(mod 10)
01) 5x 2(mod11) R: x 810(mod 1001)
11) x 4(mod 11)
4x 3(mod13)
x 6(mod 13)
x

8(mod 9)

02) x
x

2(mod 3)

R: x

26(mod 63)

5(mod 7)

2 (mod 5)

12) x
x

3 (mod 6)

x 1(mod 3)

03) x
x

2(mod 5)

2(mod11)

04) x
x

4(mod12)
5(mod13)

3(mod11)

R: x

06) x
x

52(mod 105)

13) x
x
x

R: x

772(mod 1716)

14) x
x

4128(mod 6061)

R: x

26(mod 630)

6(mod10)

2(mod 3)

09) 3x
7x

7 (mod 15)
4 (mod 32) R:
7 (mod 11)

16) x
x

3(mod 5)

8(mod 5)

R: x

5(mod 42)

5(mod 3)

17)

5(mod 7)

2(mod 7)

R:

6 (mod 9)

2(mod 3)

2x 1(mod 5)

08) 3x
5x

3 (mod 6)

x 1(mod 2)

07) x
x

8 (mod 9)

15) 27x 33(mod 48)


22x 18(mod 46)

5(mod 7)
1(mod 9)

R:

21x 15(mod 45)

05) x 5(mod19) R: x
x 10(mod 29)
x

851(mod 1430)

x 1 (mod 4)

3(mod 7)

R: x

R: x

283(mod 385)

18)

7(mod11)

5(mod13) R: x
4(mod 5)

2(mod 7)

x 11(mod 7)
x

2(mod 4)

2(mod 3)

3(mod 4)

4(mod 5)

5(mod 6)

R: x 158(mod 420)

R: x 119(mod 360)

x 1(mod 3)

7(mod11)
227(mod 715)

19)

3(mod10)

10) x 11(mod13) R: x 1103(mod 2210)


x 15(mod17)

20)

R:

3(mod 5)

5(mod 8)

7(mod11)

2(mod 4)

3(mod 5)

4(mod 7)

5(mod 9)

R:

02) Determinar o menor inteiro positivo mltiplo de 7 que tem para resto 1, quando dividido por 2,
3, 4 e 5.
R:
03) Determine os nmeros inteiros cujos restos da diviso por 3, 4 e 5 so, respectivamente, 1, 2 e 3.
R:
04) Determine um certo inteiros entre 1 e 1200 tem como restos 1, 2 e 6 quando dividindo
respectivamente por 9, 11 e 13. Determin-lo. R:
05) Um Coronel do Corpo de Bombeiros depois de assumir o comando da corporao quis saber
qual era o efetivo do Comando Geral para esse objetivo mandou o Ajudante Geral dispor o efetivo
sucessivamente em colunas de:
07 indivduos, tendo sobrado 06 indivduos;
11 indivduos, tendo sobrado 05 indivduos;
13 indivduos, tendo sobrado 03 indivduos;
Sabendo que o efetivo do Comando Geral, tem menos de 1000 de militares Determine quantos
militares constituem o efetivo.
R = 874 militares efetivo.
06) Um Coronel depois de destacado para comandar um regimento do Exrcito quis saber qual era o
efetivo desse regimento para esse objetivo mandou-os dispor sucessivamente em colunas de:
37 indivduos, tendo sobrado 01 indivduos;
32 indivduos, tendo sobrado 04 indivduos;
27 indivduos, tendo sobrado 01 indivduos;
Sabendo que um regimento, tem menos de 10.000 de militares. Determine quantos militares
constituem esse regimento.
R = 4996 militares efetivo.
07) Um bando de 17 piratas, ao tentar dividir igualmente entre si as moedas de uma arca, verificou
que haveria uma sobra de 3 moedas. Seguiu-se uma discusso, na qual um pirata foi morto. Na nova
tentativa de diviso j com um pirata a menos, verificou-se que haveria uma sobra de 10 moedas.
Nova confuso, e mais um pirata foi morto. Ento, por fim, eles conseguiram dividir igualmente as
moedas entre si. Qual o menor nmero de moedas que arca poderia conter?
R= 3930 moedas.
08) Dois satlites S1 e S2 em rbita sobre a terra passam periodicamente sobre salvador. Sabendo-se
que S1 gasta 32 horas para completar sua rbita e que S2 gasta 23 horas, e que hoje S1 foi visto s 11

horas da manha e S2 foi visto s 10 horas da manha determine quando S1 e S2 sero vistos
simultaneamente sobre salvador.
R: s 03:00 horas sero vistos simultaneamente.
09) Trs satlites passaro sobre uma cidade esta noite. O primeiro 01 h da madrugada, o segundo
s 04 hs e o terceiro s 08 hs da manh. Cada satlite tem um perodo diferente. O primeiro leva 13
hs para completar uma volta ao redor da terra; o segundo 15 h/s e o terceiro 19 h/s. Determine
quantas horas decorrero a partir da meia-noite at que os trs satlites passem ao mesmo tempo
sobre a cidade?
R: 10 horas.
10) Uma senhora estava caminhando para um mercado quando um cavalo se bateu com a sua cesta
de ovos. O cavaleiro queria pagar os danos e perguntou para a senhora quantos ovos haviam na
cesta. Ela no se lembrava exatamente da quantidade, mas sabia que se tirasse os ovos da cesta de
trs em trs, sobravam dois ovos. Se tirasse de 5 em 5, sobravam 3 ovos e de 7 em 7 sobravam 2.
Qual seria a menor quantidade de ovos que ela poderia ter?
R: Pelo menos 23 ovos.
11) Um casal resolveu ir fazer uma viagem volta do mundo. Sabendo que partiram no dia 01 de
maro de um ano bissexto num domingo que chegariam no dia 6 de maro, Segunda-feira e que
demoraram menos de 4 anos para fazer a viagem. Determine quantos dias demorou a viagem?
R:
12) Num cesto ha mais de 200 e menos de 600 ovos. Sabe-se que, se os ovos forem retirados 2 de
cada vez, no fim resta 1 ovo; se forem retirados 3 de cada vez, restam 2; se forem retirados 4 de
cada vez, restam 3; se forem retirados 5 de cada vez, restam 4; se forem retirados 6 de cada vez,
restam 5; e, se forem retirados 7 de cada vez, o cesto fica vazio. Quantos ovos esto no cesto?
R:
13) Trs marinheiros sobreviveram a um naufrgio e foram ter a uma ilha onde vivia um macaco.
Comeram alguns cocos e resolveram apanhar mais alguns para comerem nos dias seguintes. Ao
anoitecer, imediatamente antes de irem dormir, constataram que tinham apanhado menos de 100
cocos. A meio da noite, um dos marinheiros acordou e, como no confiava nos companheiros,
decidiu guardar a parte dele. Assim, tentou dividir o monte dos cocos em trs montes, todos com
igual nmero de cocos. Ao fazer isto sobrou um coco que ele deu ao macaco. Enterrou a parte dele e
foi dormir. Um segundo marinheiro acordou e, pelo mesmo motivo, decidiu tambm enterrar a parte
dele. Dividiu o monte em trs montes iguais, tendo dado ao macaco um coco que, tambm neste
caso, sobrou. Enterrou a parte dele e foi dormir. Passado algum tempo o terceiro marinheiro

acordou e fez exatamente aquilo que haviam feito os outros dois anteriormente. Na manha seguinte
os trs marinheiros dividiram novamente o monte de cocos em trs partes e voltou a sobrar um coco
que deram ao macaco. Quantos cocos havia inicialmente no monte? Com quantos cocos ficou cada
um dos marinheiros?
R:
14) Um grupo de 17 macacos guarda suas bananas em 11 cestas de igual contedo e em uma 12
cesta contendo 6 bananas. Eles podem dividir o total de suas bananas em 17 grupos iguais. Qual o
menor nmero de bananas que eles podem possuir?
R:
15) Generais Chineses contavam o nmero de soldados sobreviventes de uma batalha alinhando-os
sucessivamente em filas de determinados tamanhos, contando cada vez o nmero de soldados
restantes e calculando o total de sobreviventes partir desses dados. Um general tinha inicialmente
1200 soldados antes de uma batalha; aps a batalha, ao alinh-los em filas de 5 soldados, restaram 3
soldados; ao alinh-los em filas de 6 soldados, restaram tambm 3 soldados; ao alinh-los em filas 7
soldados, restou 1 soldado; finalmente, ao alinh-los em filas de 11 soldados, nenhum restou.
Quantos soldados sobreviveram batalha?
R:

UNIDADE XI TEOREMA DE FERMAT e WILSON


11.1 - Introduo:
Desde, pelo menos, 500 anos antes de Cristo, os chineses sabiam que, se p um nmero
primo, ento p| 2p-2. Coube a Pierre de Fermat, no sculo XVII, generalizar este resultado,
enunciando um pequeno, mas notvel teorema que se constitui no resultado central desta unidade.
Nesta unidade estudaremos algumas propriedades referentes a congruncias modulo nmeros
primos que so as aplicaes do pequeno teorema de Fermat e o teorema Wilson e sua relevncia no
estudo da teoria dos nmeros.
11.2 - Teorema de Fermat:
Teorema 11.1 - (de Fermat) - Se p um primo e se p no divide a, ento ap-1
Corolrio 11.1 - Se p primo, ento ap

1 (mod p).

a (mod p) qualquer que seja o inteiro a.

Exemplo 11.1 - Verificar o teorema de Fermat para a = 3 e p = 7.


Exemplo 11.2 - Mostrar que 538

4 (mod 11).

Exemplo 11.3 - Mostrar que 117 composto.


Teorema 11.2: Se p e q so primos distintos tais que ap
ap.q

a (mod q) e aq

a (mod p), ento:

a (mod p.q).

Exemplo 11.4 - Mostrar que 2340

1 (mod 341).

11.2 - Teorema de Wilson:


Teorema 11.3 - (de Wilson)- Se p primo, ento (p 1)!
Teorema 11.4 - Se (n 1)!

1 (mod p)

1 (mod n), ento n primo.

Obs: O teorema de Wilson e o seu recproco do um critrio para se reconhecer se um inteiro dado
primo um inteiro n > 1 primo se e somente se, (n 1)!

1 (mod n). Entretanto, para inteiros

grandes este critrio impraticvel e por isso apenas tem utilizao terica.
Exemplo 11.5 - Verificar o teorema de Wilson com p = 7.
Exemplo 11.6 - Reconhecer se o inteiro 11 primo.
Questes Resolvidas
01) Verificar o teorema de Fermat com a = 2 e p = 13.
Soluo: Temos que mostrar que 212
24

3 (mod 13); 28

1 (mod 13).

9 (mod 13); 212 = 24.28

3.9

1 (mod 13)

02) Verificar o teorema de Wilson para p = 5.


Soluo: Temos que mostrar que (5 1)!
2.3

1 (mod 5); 2.3.4

4 (mod 5); 4

1 (mod 5)

1 (mod 5); logo 4!

1 (mod 5)

03) Mostrar que 8 composto usando o teorema de Wilson:


Soluo: (8 1)! = 2.3.4.5.6.7

0 (mod 8); portanto, como (8 1)! no congruente a 1

conclumos que 8 no primo.


04) Mostrar que 19 primo usando o recproco do teorema de Wilson.
Soluo: (19 1)! = 2.3.4...17.18. Vamos ver a que nmero congruente este fatorial.
2.3.4 = 5 (mod 19); 5.6

1 (mod 19); 9.10

11 ((mod 19); 7.8

11.12 1 (mod 19); 13.14

11 (mod 19); 15.16

12 (mod 19); 7.18 2 (mod 19).

Assim 6! = 2.3.4.5.6 5.11 17 2 (mod 19); 8! = 6!.7.8


2.(5)

9 (mod 19); 12! = 10!.11.12

19). 16! = 14!.15.16

4.12

5 (mod 19)

2.(1)

2 (mod 19); 10! = 8!.9.10

9 (mod 19); 14! = 12!.13.14

9.(1)

9 (mod 19); 18! = 16!.17.18

9.2

18

9.11

4 (mod

1 (mod 19).

05) Reconhecer se 17 primo usando o teorema de Wilson:


Soluo: Temos que verificar se (171)!
4!

7 (mod 17); 5.6

7.8

13 (mod (17); 6! = 4!.5.6

5 (mod 17); 8! = 6!.7.8


4.5

10! = 8!.9.10

1 (mod 17). Temos:

6.5

7.13

6 (mod 17)

4 (mod 17); 9.10 = 5 (mod 17)

3 (mod 17); 11.12

13 (mod 17)

12! = 10!.11.12

3.13 5 (mod 17); 13.14

12 (mod 17)

14! = 12!.13.14

5.12

2 (mod 17)

16! = 14!.15.16

8.2

8 (mod 17); 15.16


16 1 (mod 17)

06) Verificar:
a) 186

1 (mod 49)

186= 182.184

Soluo: 182 30 (mod 49); 184 30.30 18 (mod 49).

30.18 1 (mod 49)

b) 186 1 (mod 343)

Soluo: 183 = 5832

1 (mod 343); 186 1 (mod 343).

7) Achar o resto da diviso de 15! por 17.


Soluo: 16!
16!

1 (mod 17) pelo teorema de Wilson; 1 16 (mod 17) ento,

16 (mod 17); da 15!.16 16 (mod 17). Podemos cancelar o fator 16, desde que mdc(16, 17) =

1 e portanto 15!

1 (mod 17). Conclumos que o resto procurado 1.

8) Mostrar que, se o mdc(a, 35) = 1, ento a12

1 (mod 35).

Soluo: 35 = 5.7 e portanto mdc(a, 5) = mdc(a, 7) = 1. Aplicando o teorema de Fermat:


a4 1 (mod 5); a12 1 (mod 5) logo 5 | (a121)
a6 1 (mod 7); a12 1 (mod 7) logo 7 | (a12 1). Como mdc(5, 7) = 1 ento,
35 = 5.7 | (a12 1) o que demonstra a12 1 (mod 35)
9) Demonstrar que, para todo inteiro a, se tem:

a ) a13 a (mod 7).


Soluo: Suponhamos que 7 | a. Ento a 0 (mod 7); a13

0 (mod 7) e portanto

a13 a (mod 7). Agora, suponhamos que 7 no divide a ento, pelo teorema de Fermat:
a6 1 (mod 7); a7 a (mod 7); a13 = a6.a7 a (mod 7)
b) a37 a (mod 13).
Soluo: Suponhamos que 13 | a. Ento a 0 (mod 13); a37 0 (mod 13) e portanto a37 a (mod
13). Agora suponhamos que 13 no divide a. Ento, pelo teorema de Fermat a12 1 (mod 13);
a13 a (mod 13); a24 = (a12)2

1 (mod 13)

a37 = a13.a24 a (mod 13).


Questes Propostas
01) Verificar o teorema de Fermat com a = 2 e p = 17.
02) Demonstrar que, para todo inteiro a, se tem:
a) a21 a (mod 15)

b) a7 a (mod 42)

03) Demonstrar que, para todo inteiro positivo n, se tem:


a) 22n 1 (mod 3)

b) 23n 1 (mod 7)

04) Mostrar que 18! + 1


05) Mostrar que 538

0 (mod 437).

4 (mod 11).

06) Mostrar:
a) 561 | (2561 2)

b) 561 | (3561 3).

07) Verificar o teorema de Wilson p = 5.


08) Mostrar que 8 composto usando o teorema de Wilson.
09) Verificar que 186

1 (mod 49).

10) Achar o resto da diviso de 15! Por 17.


11) Usando o pequeno Teorema de Fermat, encontrar o resto da diviso de 2100.000 por 17.
12) Encontrar o digito das unidades de 3100, quando expresso na base 7. R: r = 4.
13) Mostrar que se p e q so primos distintos, ento pq - 1 q p - 1 1(mod p q) .
14) Mostre que 13 | 270 + 370, usando o teorema de Fermat.
15) Usando o teorema de Wilson encontrar o menor resduo positivo de: 6 7 8 9(mod 5) .
16) Formar com os inteiros 2, 3, 4, ... , 21 todos os pares a e b tais que ab

1 (mod 23).

O LTIMO TEOREMA DE FERMAT


Desde da antiguidade o homem sentem uma fascinao por nmeros que surge quase
imediatamente da prtica. Um dos problemas mais antigos o de dividir um quadrado em soma de
dois quadrados e uma das solues {3, 4, 5}, i.e., escrever 25 = 16 + 9. Aparentemente os antigos
egpcios j usavam estes nmeros para construir ngulos retos e reconheciam que {3, 4, 5} eram
lados de um tringulo retngulo. Por volta de 500 AC os chineses tambm sabiam deste tringulo.
Provavelmente tambm os babilnios sabiam o que est por trs disto o teorema de Pitgoras Para
que um tringulo seja retngulo necessrio e suficiente que o quadrado de sua hipotenusa seja
igual a soma dos quadrados dos catetos. Olhando de outra maneira a equao a2 = b2 + c2, admita
como soluo uma terna {c, b, a} de valores inteiros, as quais chama-se ternas pitagricas. A tabela
babilnica Plimton 322 apresentam uma lista de ternas pitagricas. Por volta de 250 AC o
matemtico grego Diofanto escreveu o primeiro livro de titulo Aritmtica, dedicado ao que hoje
chamamos de Teoria de Nmeros. Ele estuda solues inteiras de equaes que hoje chamamos de
equaes diofantinas.
Para construir-se ternas pitagricas basta observarmos a identidade algbrica (a+b)2

(ab)2

= 4ab, ajeitando a = u2 e b = v2, obtendo-se a terna {u2 - v2, 2uv, u2 + v2}.


PIERE DE FERMAT
Pierre de Fermat (1601-1665) foi um Juiz francs que nasceu e viveu em Toulouse, Frana.
Ele possua uma cultura universal na poca, que cultivava a poesia, filosofia grega, direito e
principalmente matemtica. Fermat seguia uma certa tradio da poca propor problemas tipo
desafio, para outros matemticos. Ele considerado juntamente com Descartes um dos criadores da
Geometria Analtica. Suas idias sobre mtodos das tangentes contm as razes do Clculo
Diferencial. Raramente encontrasse uma publicao de Fermat como tambm ele raramente se
encontrou pessoalmente com outros grandes matemticos de sua poca. Suas comunicaes estavam
em suas correspondncias que geralmente era enviada pelo padre Marin Mersenne, e seu amigo
Pierre de Carcavy, que circulavam seus manuscritos, cartas em geral. Estes ltimos foram
colecionados em forma de livro por seu filho Samuel. Uma grande parte de seu trabalho e
problemas foram resolvido nos 200 anos seguintes sua morte. Outra grande parte dedicada as
equaes diofantinas onde ele deixou uma srie de problemas. Dentre os vrios problemas temos a
representao de primos como soma de quadrados, gerao de primos, divisibilidade por primos, e
soluo de x2 - dy2 = +1. a maioria deles foi resolvida no sculo seguinte, por Euler, Legendre,
Dirichlet e outros.
O problema conhecido como ltimo Teorema de Fermat, ou FLT, i.e., o ltimo problema de
Fermat em aberto, uma generalizao do problema das ternas pitagricas. Ele foi proposto numa

margem de uma edio do livro de Diofanto ao lado do problema 8, livro II, que o problema da
decomposio de um quadrado em soma de dois outros. Ele afirma que isto impossvel para cubos
e potncias superiores e afirma que possui uma demonstrao elegante para este fato e que no cabe
na margem. Ele tambm propem este problema para outros matemticos da poca Frenicle, Wallis,
..... O problema mostrar que a equao F[n], an + bn = cn, n inteiro maior que 2, no possui
solues no triviais no sentido de que pelo menos um dos nmeros a, b ou c zero. fcil de
verificar que basta mostrar que F[n] no tem soluo para n = 4, e para primos diferentes de 2. O
caso n = 4 foi resolvido por Fermat onde ele exibe seu mtodo a qual denomina mtodo da descida
infinita. O problema se enuncia ento mostrar que para todo primo impar p, F[p] no possui
solues no triviais. O problema pode ser dividido em dois casos: O primeiro onde se requer que
nenhum dos nmeros a, b, e c sejam divisveis por p, e o segundo o caso onde precisamente um
dos nmeros divisvel por p. Para n = 3, o cubo de 5 mais o de seis difere muito pouco do de sete.
Este problema foi proposto em 1637, e da seguiu-se uma busca para reproduzir a
demonstrao de Fermat. Como Fermat errou na soluo de outros problemas, admite-se a
possibilidade dele tambm ter errado em sua afirmao, porem no sabemos qual era sua
demonstrao.
PRIMEIRAS TENTATIVAS
As primeiras tentativas foram feitas para p = 3. Euler, no inicio do sculo passado,
apresentou a soluo neste caso. Ele cometeu enganos porem depois de um estudo minucioso da
divisibilidade de nmeros da forma a2+3.b2 ele obteve uma soluo correta. Gauss apresenta
tambm uma demonstrao para este caso. Com Gauss, 1801 nasce a Teoria dos Nmeros. O caso p
= 5, foi demonstrado por G.L.Dirichlet em 1928. O caso p = 7 de Lam (1839). Sophie Germain,
em 1823, foi a primeira a tentar uma soluo geral ela mostra que se 2p+1 tambm primo ento o
primeiro caso valido. Isto introduz um novo problema existe um numero infinito de primos para
os quais 2p + 1 tambm primo? At hoje no se sabe. Com este resultado Legendre consegue
mostrar a validade do primeiro caso para todo primo menor que 100. Lam e Cauchy apresentam
demonstraes erradas.
KUMMER
At o fim de 1993 o nico teorema geral que se conseguiu devido ao matemtico alemo
E. Kummer. Ele introduz a noo de primo regular, estes primos esto relacionados a certas
condies de divisibilidade de uns nmeros chamados nmeros de Bernouilli, Kummer demonstra
por volta de 1850, que F[p] vlido para todo primo regular. Infelizmente no se sabe se existem ou
no infinitos primos regulares, sabe-se que existe um nmero infinito de primos no regulares. Os
primos no regulares menores que 164 so 37, 59, 67, 101, 103, 131, 149, e 157. A surge com

Kummer, Dirichlet e Dedekind a Teoria de Nmeros Algbricos, teoria esta que falando
grosseiramente trata de expresses contendo radicais. A noo de nmero primo e de decomposio
estendida a estes nmeros. Isto sempre foi essencial em todos os casos demonstrados.
Em 1816, e de novo em 1850 a Academia Francesa de Cincias ofereceu uma medalha de
ouro e um prmio de 3000 Francos a quem resolve-se o problema de Fermat. A medalha foi
concedida a Kummer pelo seu brilhante trabalho neste teorema. No inicio deste Sculo, um
matemtico alemo Wohlfskehl a beira do suicdio descobriu o trabalho de Kummer sobre o
teorema de Fermat e decidiu dedicar-se a sua leitura. Ele deixou sua fortuna na Knigliche
Gesellschaft der Wissenschaften, em Gttingen, Alemanha, oferecendo um prmio de 100.000DM
pelo soluo deste problema, devido a inflao este prmio est reduzido a 10.000DM. Este prmio
leva a Academia a ter que selecionar milhares de trabalhos apresentados anualmente, dentre os quais
a grande maioria ou no fazem sentido, ou so muito elementares. Deve-se observar que Fermat era
um matemtico muito bom ele resolveu o caso n = 4 aplicando seu mtodo denominado a descente
infinita, e que qualquer soluo que se Fermat tinha em mente deveria ter contedo matemtico.
FERMAT COMPUTACIONAL
Depois de Kummer um grande nmeros de matemticos como Wagstaff, Morishima, Inkeri,
Vandiver, Eichler, Brckner, tentaram desenvolver critrios para que pudssemos verificar a
validade de F[p] via computador. Estes critrios so complicados para mencionarmos aqui. Um
deles porem bem simples Wiefrich (1909). Se p2 no divide 2p-1-1 ento o primeiro caso do
teorema de Fermat verdadeiro.
At hoje s conhecemos dois nmeros dentre todos os primos menores 3x109 para os quais
isto no verdade 1093 e 3511. A temos um novo problema existe um nmero infinito destas
excees? Desenvolveu-se tambm critrios para o estudo do caso II para primos no regulares, e
com isto Wagstaff em 1976 mostra FLT para todo p < 125000. Em 1992 Buhlen verificou a
validade de FLT para todos os primos menores que 4000000. Outra direo tomada foi de estudar o
tamanho da soluo, i.e.., se x o menor valor de uma soluo de F[p] qual um estimativa de x, ou
melhor qual a estimativa do nmero que deve-se tentar para exibir um contra exemplo? Usando-se
o resultado de Wagstaff a concluso que deveramos tentar nmeros com mais de 18x10 11
dgitos. Isto mostra que contra-exemplos est totalmente fora de nosso alcance.
TEORIA DE CURVAS
Na dcada dos 20, Mordell olha para o problema do ponto de vista de geometria algbrico
F[p] a equao de uma curva algbrica, e dizer que existe uma soluo no trivial significa dizer
que ela tem uma soluo ou ponto com coordenadas racionais, ou simplesmente uma soluo
racional. O primeiro passo seria mostrar que estas curvas teriam somente um nmero finito de

pontos racionais, desde que um dos seus invariantes, seu gnero seja maior que dois. Em 1983 o
matemtico alemo G. Faltings demonstrou esta conjectura, o que lhe valeu a Field Medal prmio
quadrienal ortografo nos Congressos Internacionais aos melhores trabalhos do perodo, equivalente
ao prmio Nobel para matemtica. Seguindo a idia de Faltings em trazer toda maquinaria moderna
da geometria algbrica para a teoria de nmeros, um grupo de matemticos associa ao problema de
Fermat uma curva algbrica especial que possui muita estrutura aritmtica a curva eltica. As curvas
de primeiro grau, equao linear em duas variveis representa uma reta no plano as do segundo grau
representam cnicas (elipse, hiprbole, parbola ou circunferncia). Uma equao do terceiro grau
representa uma cbica, ela pode ser transformada ou em y = x(x+a)(x+b) ou em y2 = x(x-a)(x+b).
Esta ltima chama-se curva eltica, ser indicada por E[a, b]. Elas tem este nome por estarem
associadas as integrais elticas, integrais estas que so duplamente peridicas. Elas tambm possuem
uma operao natural de adio de pontos. A curva no singular se a e b so distintos e diferentes
de zero.
Estudar a aritmtica da curva significa estudar os pares de inteiros que satisfazem sua
equao. Seja E uma curva eltica. Para cada primo p vamos chamar de n(p) o nmero de solues
de sua equao modulo p. Quando os nmeros n(p) possuem uma frmula aritmtica de clculo a
curva chama-se modular. Em 1955 um matemtico japons chamado Tanyama postulou que toda
curva eltica onde a e b so nmeros racionais uma curva modular.
Um matemtico alemo G. Frey em 1980 teve a idia de associar a cada soluo {a, b, c} de
F[p] a curva eltica E[an,bn] a qual tem uma estrutura algbrica muito estranha, e da a suspeita de
que tal curva no poderia existir e isto implicaria na no existncia da soluo de F[p]. Em 1986 K.
Ribet mostra que a curva associada a F[p] no modular, e da a conjectura de Tanyiama implica
em FLT !!!. No fundo o que Ribet fez foi mostrar que uma conjectura mais fraca, a de J P Serre, um
dos grandes matemticos deste sculo, se aplica a curva de Frey.
ANDREW WILES.
Andrew Wiles nasceu em 1954 na Inglaterra. Ele conta que quando tinha dez anos pegou na
biblioteca um livro onde viu este problema e desde ento se props a resolv-lo. Deixou depois de
varias tentativas amadora, o problema de lado foi estudar matemtica em Cambridge com um
grande especialista em teoria de nmeros, John Coates. Veio para Harvard onde passou dois anos e
resolveu dois problemas em aberto nesta rea. Em seguida aceita uma posio de Professor em
Princeton. Quando soube dos resultados de Ribet, ele ganhou confiana de que poderia resolve FLT
e ao contrario do matemtico comum ele decide se isolar por sete anos ate apresentar uma soluo
ao problema. Levou cinco anos para ter uma idia da soluo.

A primeira vista pode-se pensar que Wiles se trancou e ignorou o que se passa na sua rea;
isto no foi o caso, pois uma boa tcnicas e construes no existiam em 86. Sua demonstrao
apareceu volume de Maio do Annals of Mathematics de 1995. A soluo apresentada em 23 de
Junho de 1993 tem um erro que depois foi consertado por um aluno dele. O trabalho original
acertado juntamente com o material necessrio para corrigir o anterior ocupa todo este volume do
Annals. O que Wiles mostra que toda curva eltica semiestavel modular e em particular a curva
de eltica associada a uma possvel soluo de F[p] tambm modular o que contradiz ao teorema
de Ribet. Logo a cura no existe, i.e., F[p] no admite solues no triviais.
Observaes
Obs1: Babilnios e certamente Diofanto j sabiam que existe um numero infinito (em potencial) de
ternas Pitagricas.
Obs2: Atualmente o material lecionado no tanto no secundrio como nos cursos de matemtica
uma preparao para se entender mais tarde o que realmente a matemtica. A grande aventura
lgica. Idias que vem da natureza. Satisfao pessoal de resolver problemas e mais ainda de criar
nova matemtica. Neste sentido a Matemtica e nica, a cada minuto de envolvimento nos d
grande prazer, mesmo para o profissional coisas simples como novas maneiras de apresent-la so
gratificantes.
Obs3: O processo de verificao e publicao de resultados longo. Por um lado a Matemtico
requer lpis e papel somente (hoje em dias supercomputadores em alguns ramos como, por
exemplo, teoria de nmeros) o que a torna acessvel a milhares de pessoas, os matemticos hoje em
dia so da ordem de 200.000. Cada anos se produz cerca de 1.000.000 de resultados novos. Cada
um ou dois anos ela dobra o que torna impossvel a matemticos (the mathematical experience)
saberem toda a matemtica. H. Poincare foi o ultimo universal. nos 60 primeiros anos do sculo 20
A Weil era universal nas reas de Geometria Algbrica, lgebra, Teoria de Nmeros, e outras reas
relacionadas ele tinha a reputao de ter um grande faro matemtico, uma grande intuio dos
problemas que so solveis. Enunciar problemas muito fcil porem a grande maioria so de
soluo muito difcil ou ate impossvel com tcnicas atuais. Muitas vezes eles requerem um
amadurecimento de um ou vrios matemticos por dcadas. Atualmente j difcil dominar um das
centena de seus ramos e sub-ramos. O Mathematical Review a principal revista que apresenta
resenhas dos trabalhos publicados no ultimo ano, ele atualmente um volume maior que um dos
volumes de qualquer enciclopdia. Apesar de alguns problemas como o de Fermat serem simples de
enunci-los a probabilidade de se resolver novos problemas com tcnicas elementares bem
pequena. No se faz pesquisa a nvel de graduao, nem mesmo a nvel de mestrado, pois os
problemas em aberto requerem cada vez mais quantidades muito grande de conhecimento. Aquele

fato de que grande matemtica Os feita por jovens j esta passando, pois solues de grandes
problemas requerem muito conhecimento de tcnicas e de certo domnio de outros ramos, o que
leva tempo de aprendizado. Afinal de contas, Wiles tem quarenta anos.
Obs4: Ningum nunca saber qual a demonstrao que Fermat tinha em mente. Muitos matemticos
acham que ele errou em algum canto. Outros porem acham que ele pode ter descoberto uma
demonstrao deste teorema. Ele tinha uma autoreputao para cuidar.
CONFERENCIAS SOBRE FLT.
Em linhas gerais seguimos a conferencia patrocinada pelo MSRI (Mathematical Sciences
Research Institute- Berkeley, California) onde que consistiu de cinco conferencias de dez minutos,
seguida de uma mesa redonda. Em ordem falaram Osserman e ,Buhlen diretores do MRSI e os
eminente matemticos C. Rubin , Ribet e J.Conway. Osserman fala sobre a pr-histria do problema
e acrescenta dois pontos no apresentados acima. O primeiro um problema pratico Voc entra
numa Pizzaria e voc tem duas escolhas:
Comer uma pizza grande, ou uma pequena e uma media. Queremos saber qual escolha tem
mais pizza. Se cortamos as pizzas ao meio e com seus dimetros formarmos um tringulo, se este
for obtusngulo a primeira escolha a melhor. Se o tringulo for acutngulo a segunda escolha a
melhor. Os existe uma posio onde as duas escolhas so iguais quando o tringulo retngulo, e
isto uma conseqncia imediata do teorema de Pitgoras e sua recproca a lei dos cossenos.
O segundo ponto refere-se a Pitgoras. Na realidade ele era o lder de uma escola filosfica
de iniciao que se dedicava ao estudo da perfeio do universo. Eles descobriram que a natureza
traz um grande numero de fenmenos que podem ser descritos via matemtica. A seguir ele
descreve a teoria musical de Pitgoras: a busca por notas harmnicas. O fato o seguinte: se
tivermos uma corda vibrante de extremos AB e a subdividimos prendendo-a em um ponto C, e
tocarmos as duas sub-cordas sucessivamente observaremos que razes como 3:4 ou 2:3 soam
harmonicamente. Se considerarmos cordas sobre os lados de um tringulo retngulo teremos
tringulos harmnicos onde qualquer pares de lados so harmnicos. Os tringulos de lados inteiros
so harmnicos!. Dai o interesse por ternas Pitagricas. Convm mencionar que no caso do
tringulo retngulo issceles a razo 1:2. Esta razo foi conhecida na idade media como musica
diablica. O problema das construes das ternas Pitagricas esta resolvido no livro de Diofanto.
Buhlen discute o desenvolvimento do problema, os resultados obtidos nos 349 anos seguinte. Alem
do que foi dito acima ela fala um pouco mais sobre a vida de Sophie Germain, antes deste sculo
uma das trs matemticas. Ela estudou matemtica as escondidas, seu trabalho foi apresentado com
outro nome M. Leblach pois a Academia Francesa no admitia mulheres, e por fim teve ser trabalho

reconhecido por Gauss. O livro de Gauss sobre aritmtica, publicado em 1801 quando ele tinha 21
anos d a teoria de nmeros um status de rea da matemtica.
Antes ela era tratada como recreao. Gauss tambm no da grande importncia ao problema
de Fermat. A primeira pergunta que surge quando nos enfrentamos com um problema matemtico
saber como verificar a validade de nossas afirmaes, ou tentamos tambm mostrar que nossa
afirmao no valida, e para isto basta exibir um exemplo onde o teorema no valido. Tendo isto
em mente durante pelo menos um sculo ou mais tentou-se encontrar contra exemplos
procedimento este que foi abandonado nos meados deste sculo quando se teve conscincia que o
valor da menor soluo muito grande (Milnor). O caso n = 4 um dos poucos onde Fermat
apresenta sua demonstrao na margem de uma pagina do livro de Diofanto usando a descente
infinita que consiste em assumir a existncia de uma soluo, achar o menor valor da terna soluo
e por algum processo fabricar uma outra soluo que tenha menor valor que o da primeira terna.
Qualquer demonstrao de FLT feita pelo mtodo de reduo ao absurdo: parte-se de uma
suposta soluo e obtm-se uma contradio. Outro mtodo o usado por Diofanto a analise dos
divisores primos de xn = yn - zn. J que este ultimo lado direito pode ser decomposto em fatores que
em geral so nmeros complexos. Os erros de demonstrao foram na analise de divisibilidade
destes fatores. Uma analise mais critica destes fatores levou Kummer a criao da noo de
(numero) ideal, e dai a criao de uma teoria destes objetos anloga a de nmeros. Rubin comea
tratando de uma curva eltica: o grfico da equao y2 = x3 - x. Ela foi tratada por Fermat que
mostrou no ter soluo inteira. Esta equao aparece na determinao de todos os tringulos
retngulos de lados inteiros cuja rea um quadrado perfeito. Seu objetivo a noo de
modularidade. Ele explicita um procedimento muito comum na teoria de nmeros mais
especificamente quando se quer saber se uma equao tem ou no solues inteiras o que em geral
um problema bastante difcil. Reduz-se a equao modulo um primo dado observando-se que se ela
tem soluo inteira ela ter soluo modulo p para todo primo p, o que sempre fcil de se verificar
pois ficaremos com um problema de contagem. O prximo passo verificar se ela tem soluo
modulo pn para todo n. E dai tenta-se levantar esta soluo para uma soluo inteira este o
chamado principio de Hasse. No caso das curvas elticas este procedimento apresenta enormes
dificuldades. No caso particular da equao acima Gauss em 1814 apresenta uma formula para este
calculo. Ribet baseado na formula de Gauss apresenta a seguinte tabela:
primos: 2 3 5 7 11 13 17 19 23 29 31
n(p) 2 3 7 7 11 7 15 19 23 29 31
primos 1000003 1000033 1000037 1000039
n(p) 1000003 998207 998055 1000039

O calculo de n(p) para nmeros pequenos pode ser feito atravs de uma tabela contando-se o
numero de pares onde p divide y2 - x3 + x. para p = 5 a tabela contem essencialmente 25 valores. J
para 1000003 a tabela contem mais de 1012 valores o que difcil de contar. Rubin botem os
valores acima da formula de Gauss. Nos casos simples n(2)=2, e se p for um primo da forma 4k-1,
n(p) = p. No outro caso a formula mais complicada. Uma curva eltica em geral chama-se modular
se esta seqncia de inteiros n(p) admite uma formula de calculo. A curva modular desempenha um
papel fundamental na soluo de Wiles. Em 1955 Yutata Tanyama apresenta a hiptese que toda
curva eltica a coeficientes inteiros modular na poca e percebeu motivos para esta hiptese.
Tanyiama morreu em 58 mais tarde Goro Shimura professor em Princeton apresenta boas
razes porque a hiptese deveria ser verdadeira. Dai em diante o numero de especialista s na rea
que acreditavam na hiptese foi crescendo. Esta a pea principal da soluo de Wiles ele
demonstra a hiptese de Tanyiama para uma classe especial de curvas que so as semi-estaveis.
Ribet comea observando que um matemtico alemo G. Frey no inicio da dcada dos 80 comeou
a fazer clculos, e associou curva de Fermat uma curva eltica E[an, bn]. Em 1981 Frey veio a
Berkeley e trocou idias com Ribet. Na poca ele j estava percebendo a ligao de FLT e
Tanyiama, porem no estava claro. Em Janeiro de 1985, em Oberwolfach ele apresentou uma curta
comunicao onde delineava suas ideais, esta comunicao chegou a Paris onde Serre, um dos
grandes matemticos atuais que tambm possui a medalha da rea, observou que uma pequena
variao da conjectura de Tanyiama implicaria FLT. Em 1986 ele, Ribet tinha uma demonstrao da
conjectura de Serre e apresenta a Mazur que diz que parece correta. Ribet passa todo o ano de 87 no
MSRI onde apresenta seminrios semanais sobre sua demonstrao. Como o processo de publicao
lento, passa por referres onde tem-se que negociar sugestes, e somando-se o atraso nas
publicaes, seu trabalho foi publicado em 1990. A seguir ele comenta como Wiles se retirou e
observa que a soluo depende na realidade de muitas outras estruturas e trabalhos. Ribet observa
que a soluo de Wiles no um fato isolado. Ela depende dos trabalhos que muitos outros
desenvolveram nesta ultima dcada este trabalho envolve tcnica desenvolvidas nas reas:
1. Famlias de Representaes de Galois (Hida, Mazur)
2.Teoria de Iwahori (Greenberg, Rubin)
3.Sistemas de Euler(Kolyvagen, Flach)
4.Formas Modulares(Ribet)
5.Geometria Algbrica(Faltings)
6.Teoria da Representao(Langlands, Tunnel)

TRABALHO E PROBLEMAS DE FERMAT NA TEORIA DE NMEROS


Vamos a seguir apresentar uma pequena lista com comentrios sobre os problemas
enunciados por Fermat.
1. O primeiro de nossa lista o pequeno teorema de Fermat Seja p um primo dado. Ento para todo
inteiro a p divide ap-a. Este foi generalizado por Euler e sua verso geral que para todo n inteiro
positivo e todo inteiro a n divide af(n)-a, onde f(n) o numero de inteiros entre 0 e n-1 que so
relativamente primos a n.
2. Fermat estudou formulas para primos a primeira quando M(n) = 2n -1 primo. Isto leva ao
problema de saber para que valores de n, primo, M(n) primo. Estes primos chamam-se primos de
Mersenne pois foi ele que testou todos os nmeros primos menores que 100, com alguns erros. No
se sabe se existe um numero infinito de primos de Mersenne. Outro caso estudado por Fermat foi a
primalidade de F(n) = 2n -1. Aqui necessrio que n seja uma potncia de dois. Fermat afirmou que
todos estes nmeros so primos. Euler mostrou que F(5) divisvel por 641. Ate hoje no se
conseguiu outro valor maior que 5 que nos fornea um numero primo, tentou-se at 11, e alguns
valores acima tambm.
3. Fermat estudou a representao de nmeros como soma de outros particulares. Por exemplo um
primo cujo resto na diviso por 4 um uma soma de dois quadrados. Isto foi demonstrado por
Euler em 1749. Em 1977, Lagrange mostra que todo nmeros soma de quatro quadrados. Em 1801
Gauss demonstra o teorema de Fermat sobre Os de trs nmeros triangulares. Em 1813, Cauchy
demonstra os teoremas sobre cinco nmeros pentagonais e outros. Em 1840 Jacobi demonstra
outros teoremas deixados por Fermat.
4. A equao de Pell x2 - dy2 = +1, d inteiro, assim chamada erroneamente por Euler, tambm foi
estudada por Fermat. Fermat j sabia que esta equao possuiu um numero infinito de solues
quando d > 0. Legendre mostra, usando fraes continuas que todas solues so potncias de uma
delas mais tarde Dirichlet demonstra seu famoso teorema de unidades o qual engloba todos estes
resultados.

BIBLIOGRAFIA:
1) Edgard de Alencar Filho - Teoria Elementar dos Nmeros - Livraria Nobel S.A. - 1981.
2) Hygino H. Domingues - Fundamentos de Aritmtica - Atual - 1991.
3) L.H. Jacy Monteiro - lgebra Moderna - Volume I - LPM - 1963.
4) I. Vinogradov - Fundamentos de la Teoria de los Nmeros - Editorial Mir. - 1977.
5) Jos Plnio de Oliveira Santos Introduo Teoria dos Nmeros Coleo Matemtica A. J.
6) Pettofrezzo - Teoria de los Nmeros - Editorial Prentice / Hall Intercontinental - 1972.
7) Universitria - 1998 George E. Andrews Number Theory Dover Publications, Inc. - 1994.

Você também pode gostar